Sie sind auf Seite 1von 146

“Livro˙Olimpiada

i i
2006/5/27
page 1
i i

Olimpı́adas de Matemática: uma introdução

Adán J. Corcho & Fernando E. Echaiz & Krerley Oliveira

27 de maio de 2006

i i

i i
“Livro˙Olimpiada
i i
2006/5/27
page 2
i i

INTRODUÇÃO

As Olimpı́adas de Matemática têm crescido rapidamente no Brasil, com


a participação de milhões de estudantes em competições em vários nı́veis,
desde competições municipais e estaduais até as competições nacionais e in-
ternacionais, como a Olimpı́ada Brasileira de Matemática (www.obm.org.br)
e mais recentemente, a Olimpı́ada Brasileira de Matemática das Escolas
Públicas (www.obmep.org.br). Além de serem um excelente passatempo
para os estudantes, as Olimpı́adas de Matemática incentivam os mais tal-
entosos para um estudo um pouco mais profundo da matemática no ensino
básico. O resultado dessa combinação são alunos motivados e interessados
no estudo da Matemática, superando o tradicional tabu que existe em torno
da disciplina.
Apesar disso, os textos disponı́veis para o treinamento em Olimpı́adas de
Matemática são escassos, em comparação com os numerosos e quase sempre
mal formulados livros-texto para o ensino regular. Pensando nisso, na neces-
sidade dos bons alunos e professores de ensino médio no tocante a material
adicional de matemática do ensino médio, é que escrevemos esse livro. Jun-
tando as experiências didáticas vividas pelos autores em três paı́ses difer-
entes, Brasil, Cuba e Peru, esperamos tornar para o leitor a matemática
mais interessante, mostrando um pouco do imenso brilho e beleza que ela
esconde.
Agora algumas palavras de como o livro foi escrito. Os capı́tulos estão
baseados em fatos básicos da matemática e trazem alguns teoremas fun-
damentais. Como regra geral, os estudantes podem simplesmente ler as
demonstrações, sem a preocupação em entender os detalhes delas. Já os
exemplos e aplicações dos teoremas devem ser lidos com cuidado e muita
atenção. Para os estudantes que desejem treinar para olimpı́adas de mate-
mática, sugerimos que formem grupos de estudo para trabalhar os temas

i i

i i
“Livro˙Olimpiada
i i
2006/5/27
page 3
i i

individualmente, sob a orientação de um professor.


O primeiro capı́tulo é para introduzir o leitor no espı́rito do livro e
dar uma amostra do tipo de problemas e material que seguirá nos demais
capı́tulos. São propostos alguns problemas, muitos deles com soluções, e
discutimos alguns métodos importantes para uso no dia-a-dia dos estu-
dantes. Nesta discussão, incluı́mos o Método de Redução ao Absurdo e
algumas regras básicas e cuidados que devemos ter ao resolver problemas
em Matemática.
O capı́tulo seguinte trata do conceito de divisibilidade. Tentamos intro-
duzir o leitor nos principais aspectos básicos, incluindo-se a divisibilidade
com resto, máximo divisor comum e mı́nimo múltiplo comum, números pri-
mos e equações diofantinas. A organização dos exemplos tenta seguir uma
linha em ordem crescente de dificuldade e, para o melhor aproveitamento
do curso, o trabalho com os exercı́cios é parte fundamental.
No capı́tulo de contagem, começamos com noções úteis sobre conjuntos e
princı́pios básicos para contar os elementos de um conjunto. Neste capı́tulo,
estamos mais preocupados com as aplicações imediatas do que com os as-
pectos teóricos do assunto, sugerindo alguns problemas para o estudante
iniciante. Seguimos discutindo os tipos de agrupamento de elementos e
suas conseqüências.
Um dos capı́tulos mais úteis para o estudante que deseja participar de
Olimpı́adas de Matemática é o que trata do Princı́pio da Casa dos Pom-
bos. Este capı́tulo é um belo exemplo de como algo aparentemente ingênuo
pode gerar conseqüências interessantes. Alguns dos exemplos estão conec-
tados com os capı́tulos anteriores e aparentemente aplicam o princı́pio de
modo inusitado, em problemas de geometria, teoria dos números e em áreas
diversas.
Finalmente, o estudante se depara com uma arma poderosa do matemático.
O capı́tulo 5 está dedicado a Método da Indução Finita. Novamente, procu-
ramos conectar este capı́tulo com os capı́tulos anteriores, re-obtendo com o
auxı́lio do Método da Indução algumas coisas que já foram deduzidas por
outros métodos. Vários exemplos e problemas são resolvidos, alguns deles
de modo surpreendente e inesperado.
Para deixar o leitor com bastante trabalho para fazer e fixar as idéias de-
senvolvidas, dedicamos um capı́tulo a problemas de Olimpı́adas de Matemática.
De olimpı́adas regionais, nacionais e internacionais, eles apresentam um de-
safio suficiente para motivar o leitor num treinamento posterior.
Somos gratos a muitas pessoas que colaboraram com a elaboração deste

i i

i i
“Livro˙Olimpiada
i i
2006/5/27
page 4
i i

livro com sugestões e correções de muitos erros em versões iniciais. Entre


eles, citamos: Carlos Gustavo Moreira, Ali Tahzibi, Iram Gléria, Eduardo
Teixeira, Valdenberg Araújo e Chico Potiguar. Finalmente, dedicamos este
livro as nossas esposas e filhos, os quais compreenderam os sábados sacrifi-
cados em função de escrevê-lo.

i i

i i
“Livro˙Olimpiada
i i
2006/5/27
page 5
i i

Sumário

1 Primeiros Passos 8
1.1 Tarefas Impossı́veis . . . . . . . . . . . . . . . . . . . . . . . . 8
1.1.1 A morte do barbeiro . . . . . . . . . . . . . . . . . . . 8
1.1.2 A prova surpresa . . . . . . . . . . . . . . . . . . . . . 9
1.1.3 Os Irmãos Gananciosos . . . . . . . . . . . . . . . . . 10
1.2 Organizando as Idéias . . . . . . . . . . . . . . . . . . . . . . 11
1.3 Verdadeiro ou Falso? . . . . . . . . . . . . . . . . . . . . . . . 13
1.4 O Método de Redução ao Absurdo . . . . . . . . . . . . . . . 16
1.5 Dicas para Resolver Problemas . . . . . . . . . . . . . . . . . 19

2 Divisibilidade 29
2.1 Conceitos Fundamentais e Divisão com Resto . . . . . . . . . 29
2.2 Máximo Divisor Comum e Mı́nimo Múltiplo Comum . . . . . 38
2.3 Números Primos e Compostos . . . . . . . . . . . . . . . . . . 48
2.3.1 O Crivo de Eratóstenes . . . . . . . . . . . . . . . . . 50
2.3.2 O Teorema Fundamental da Aritmética . . . . . . . . 53
2.3.3 Identificando Números Compostos . . . . . . . . . . . 58
2.4 Um pouco sobre Equações Diofantinas . . . . . . . . . . . . . 63
2.5 Exercı́cios . . . . . . . . . . . . . . . . . . . . . . . . . . . . . 66

3 Contagem 69
3.1 Princı́pio Aditivo de Contagem . . . . . . . . . . . . . . . . . 69
3.2 Princı́pio Multiplicativo de Contagem . . . . . . . . . . . . . 76
3.3 Uso simultâneo dos Princı́pios Aditivo e Multiplicativo . . . . 82
3.4 Permutações Simples . . . . . . . . . . . . . . . . . . . . . . . 85
3.5 Arranjos Simples . . . . . . . . . . . . . . . . . . . . . . . . . 87

i i

i i
“Livro˙Olimpiada
i i
2006/5/27
page 6
i i

6 SUMÁRIO

3.6 Combinações Simples . . . . . . . . . . . . . . . . . . . . . . . 91


3.7 Contagem e Probabilidades . . . . . . . . . . . . . . . . . . . 94
3.8 Exercı́cios Propostos . . . . . . . . . . . . . . . . . . . . . . . 96

4 O Princı́pio da Casa dos Pombos 99


4.1 Primeiros Exemplos . . . . . . . . . . . . . . . . . . . . . . . 100
4.2 Uma Versão mais Geral . . . . . . . . . . . . . . . . . . . . . 102
4.3 Aplicações na Teoria dos Números . . . . . . . . . . . . . . . 103
4.4 Aplicações Geométricas . . . . . . . . . . . . . . . . . . . . . 105
4.5 Miscelânea . . . . . . . . . . . . . . . . . . . . . . . . . . . . 108
4.6 Exercı́cios . . . . . . . . . . . . . . . . . . . . . . . . . . . . . 110

5 Indução Matemática 111


5.1 Formulação Matemática . . . . . . . . . . . . . . . . . . . . . 111
5.1.1 Método de Indução Matemática . . . . . . . . . . . . 112
5.2 Problemas Clássicos . . . . . . . . . . . . . . . . . . . . . . . 113
5.2.1 Demonstrando Identidades . . . . . . . . . . . . . . . 113
5.2.2 Demonstrando Desigualdades . . . . . . . . . . . . . . 117
5.2.3 Indução em problemas de divisibilidade . . . . . . . . 117
5.3 Indução na Geometria . . . . . . . . . . . . . . . . . . . . . . 119
5.4 Miscelânea . . . . . . . . . . . . . . . . . . . . . . . . . . . . 123
5.5 Método Forte da Indução . . . . . . . . . . . . . . . . . . . . 125
5.6 Exercı́cios . . . . . . . . . . . . . . . . . . . . . . . . . . . . . 128

6 Problemas de Olimpı́adas 131

i i

i i
“Livro˙Olimpiada
i i
2006/5/27
page 7
i i

Sumário

i i

i i
“Livro˙Olimpiada
i i
2006/5/27
page 8
i i

Capı́tulo 1

Primeiros Passos

Neste capı́tulo, faremos uma coletânea de problemas e idéias que servirão


para ilustrar vários métodos que iremos discutir nos capı́tulos seguintes e dar
um sabor do tipo de argumento que aparece nos problemas em matemática.
Alguns dos exemplos que discutiremos serão úteis para orientar quanto ao
cuidado que devemos ter quando vamos discutir problemas em Matemática.
Começaremos com três histórias interessantes e muito intrigantes, os quais
discutiremos superficialmente.

1.1 Tarefas Impossı́veis


1.1.1 A morte do barbeiro
Num reino muito antigo e muito pequeno só havia um barbeiro, que cuidava
da aparência Real. O Rei, para agradar sua população que estava insatisfeita
com seu governo, decidiu decretar o seguinte:
Decreto Real: O Barbeiro Real barbeará somente aquelas pessoas que não
se barbeiam. Caso esse decreto seja descumprido, ordeno a morte do
barbeiro.
O sábio da corte, mago e conhecedor de Matemática, ao ver o decreto real
que iria ser publicado, correu ao encontro do Rei, clamando para que Sua
Majestade desistisse de tal idéia. O Rei espantado, achou uma impertinência
tal comportamento do mago, que prontamente lhe explicou suas razões:

i i

i i
“Livro˙Olimpiada
i i
2006/5/27
page 9
i i

[SEC. 1.1: TAREFAS IMPOSSÍVEIS 9

Majestade, caso Vossa Alteza decida publicar tal decreto, o pobre Bar-
beiro Real terá que ser morto, pois é humanamente impossı́vel cumpri-lo.
Apesar de Vossa Majestade já saber disso, vou com sua permissão tomar a
liberdade de me explicar:
“Imagine que tal decreto seja publicado e esteja em vigor em seu fabuloso
reino. Neste caso, restariam ao barbeiro duas possibilidades:

• O barbeiro não se barbeia.


Neste caso, Vossa Sabedoria obrigaria ao barbeiro a se barbear, o
que seria impossı́vel para essa pobre criatura, uma vez que ele não se
barbeia. Resultado: ele seria morto.

• O barbeiro se barbeia.
Neste caso, ele estaria descumprindo seu decreto de barbear somente
quem não se barbeia. Assim ele deveria ser morto.

Note Vossa Sabedoria, que sabe mais do que eu, que ele seria morto de
qualquer modo caso tal decreto entrasse em vigor, aumentando ainda mais
a insatisfação do povo. Assim, a humilde sugestão de seu servo é que em
sua graça e bondade, a publicação de tal decreto seja adiada.”
Depois dessas palavras sábias e precisas do Mago da Corte, a vida do
pobre barbeiro foi salva e ele viveu até os últimos dias de sua vida cortando
feliz a barba de quem se barbeasse e de quem não se barbeasse.

1.1.2 A prova surpresa


O Diretor de uma escola famosa, aborrecido com o mau desempenho dos
alunos do primeiro ano, pediu ao professor de matemática que comunicasse
aos estudantes que na semana seguinte seria realizada uma prova-surpresa,
na intenção de fazer com que os estudantes se dedicassem mais. Quando
conversou com o professor de matemática a esse respeito, o professor disse
que de acordo com a lógica, ele não deveria anunciar nada, pois se anunciasse
não havia meios de que a prova fosse surpresa. Como o diretor não aceitou
esse argumento sem uma explicação detalhada, o professor de matemática
teve que se explicar. Os dois tiveram a seguinte conversa:
- P: - Diretor, digamos que eu deseje realizar tal prova-surpresa. Neste
caso, não poderei realizá-la na sexta-feira.
- D: Mas, por quê?

i i

i i
“Livro˙Olimpiada
i i
2006/5/27
page 10
i i

10 [CAP. 1: PRIMEIROS PASSOS

- P: Caso a seja realizada na sexta, na quinta-feira os alunos vão perceber


isso, pois notarão que ela não foi realizada até a quinta.
- D: Ah! Tá certo. Mas você pode realizar a prova na quinta.
- P: Não, eu também não poderei realizá-la na quinta.
- D: Deixe-me então tentar justificar isso. Veja só, como você não pode
realizar a prova na sexta, ela terá que acontecer entre segunda-feira e quinta-
feira. Logo, se ela for ser realizada na quinta, quando chegar a quarta-feira
os alunos perceberão que não houve prova até o momento e deduzirão que
ela será na quinta, já que não pode ser na sexta!
- P: Perfeito, diretor! Você entendeu o raciocı́nio. Logo ela deve acon-
tecer segunda, terça ou quarta. Do mesmo modo, podemos argumentar e
excluir a quarta-feira.
- D: Então ela será na segunda!
- P: Exatamente! E neste caso deixará de ser surpresa.
- D: Entendi seu argumento, professor. Acho que vou ter que arrumar
outro castigo para esses alunos indisciplinados. Obrigado pela explicação!

1.1.3 Os Irmãos Gananciosos


No dia do aniversário dos gêmeos Cosme e Damião, seu pai, dono de uma
das maiores bombonieres da cidade, decidiu trazer para presentear os dois
aniversariantes uma sacola com quarenta doces. A cada minuto, eles se
alternam na escolha, podendo pegar um ou dois doces. Caso um deles
pegue dois doces, o pai pega a sacola de volta pois considera esta atitude
muito gananciosa. Todos os dois sabem disso e sabem que podem ficar com
vinte doces cada um, caso não sejam gananciosos. O primeiro a escolher
seu doce é o Cosme. O que o Cosme deve fazer?
Se você respondeu “pegar um doce somente”, vamos mostrar a você que
a lógica, pura e simples, vai nos levar na direção contrária (apesar de toda
pessoa de bom senso começar escolhendo um doce).
Para ver isso, vamos imaginar que eles já fizeram várias escolhas e os
doces estão acabando. Como o Cosme começou a retirar os doces, depois de
38 retiradas sucessivas haverá exatamente dois doces na sacola. Neste caso,
o que o Cosme faria? Naturalmente, ele pegaria os dois doces. Porém, seu
irmão Damião sabe disso e na escolha imediatamente anterior, quando hou-
vesse três doces na sacola ele teria duas opções: tiraria um doce e deixaria
seu irmão com dois doces, ou tiraria dois doces e acabaria a distribuição.

i i

i i
“Livro˙Olimpiada
i i
2006/5/27
page 11
i i

[SEC. 1.2: ORGANIZANDO AS IDÉIAS 11

Como ele conhece seu irmão e sabe que se deixasse os dois doces ele iria
tirá-los, ele acabaria o jogo quando houvesse três doces no saco.
Neste momento, argumentamos por analogia: quando houvesse quatro
doces no saco, Cosme saberia que se tirasse um doce, Damião tiraria dois e
acabaria com a distribuição. Assim, ele tiraria dois doces e acabaria a dis-
tribuição quando houvesse quatro doces na sacola. Por sua vez, o Damião...
Argumentando de trás-para-frente, concluı́mos que o Cosme deveria tirar
na sua primeira escolha dois doces e acabar com a distribuição. Entendeu?

1.2 Organizando as Idéias


Nesta seção vamos começar a montar os alicerces de nossa construção em
Matemática. Para isso, precisamos ter bem claros em nossa mente alguns
conceitos que serão úteis na hora de distinguir entre o que devemos provar
e o que estamos assumindo como verdade em uma afirmação matemática.
É sobre isso que falaremos agora. Começaremos observando as seguintes
afirmações:

(a) Se como muitos doces, então passo mal;


(b) Se estudo bastante, então passo no exame de inglês.

Em ambas, estamos assumindo alguma coisa como verdadeira e a partir


disso, afirmamos que alguma outra coisa é válida. Na primeira afirmação,
estamos assumindo que comemos muitos doces e afirmamos que, com isto,
acabamos passando mal. Em nosso estudo da Matemática, ter em mente o
que estamos assumindo como verdadeiro e o que desejamos provar é muito
importante para evitar confusões e erros em nossos argumentos. Para facil-
itar essa divisão, vamos definir o que entendemos por proposição:

Definição 1.1. Uma proposição é uma afirmação composta de hipótese e


tese. A hipótese é tudo aquilo que assumimos como verdadeiro. A tese é
tudo aquilo que queremos mostrar.

Em nossa segunda afirmação, estamos assumindo que estudamos bas-


tante. A hipótese neste caso é:

• Hipótese: Estudo bastante.


Queremos mostrar que passaremos no exame de inglês. A tese é então:

i i

i i
“Livro˙Olimpiada
i i
2006/5/27
page 12
i i

12 [CAP. 1: PRIMEIROS PASSOS

• Tese: Passo no exame de inglês.

Forma Direta: Vamos chamar o modo em que apresentamos uma proposição


de forma direta. Por exemplo, quando enunciamos a proposição “Se x é uma
laranja, então x é uma fruta”, estamos enunciando sua forma direta. Vamos
descrever agora como podemos obter novas proposições a partir da forma
direta de uma proposição:

Forma Negativa: Para obter a forma negativa basta que neguemos a


hipótese e neguemos a tese:

Forma Direta Forma Negativa


Hipótese x é uma laranja x não é uma laranja
Tese x é uma fruta x não é uma fruta

Logo a forma negativa fica sendo:

Se x não é uma laranja, então x não é uma fruta.

Forma Recı́proca: Para construirmos a forma recı́proca, temos que trocar


a hipótese pela tese (e vice-versa) na forma direta. No nosso exemplo:

Forma Direta Forma Recı́proca


Hipótese x é uma laranja x é uma fruta
Tese x é uma fruta x é uma laranja

i i

i i
“Livro˙Olimpiada
i i
2006/5/27
page 13
i i

[SEC. 1.3: VERDADEIRO OU FALSO? 13

Assim a Forma Recı́proca é:

Se x é uma fruta, então x é uma laranja.

Forma Contra-recı́proca: Para obtermos a forma contra-recı́proca a par-


tir da forma direta, temos que trocar a hipótese pela tese e negar a hipótese
e a tese.

Forma Direta Forma Contra-recı́proca


Hipótese x é uma laranja x não é uma fruta
Tese x é uma fruta x não é uma laranja

Logo a forma contra-recı́proca fica sendo:

Se x não é uma fruta, então x não é uma laranja.

1.3 Verdadeiro ou Falso?


Uma das coisas que distingue a Matemática das demais Ciências Naturais é
o fato de que um tema de Matemática é discutido utilizando-se a lógica pura
e, por conta disso, uma proposição em Matemática uma vez demonstrada
através de argumentos lógicos corretos, é aceita como verdade e não sofre
grandes alterações ao longo dos tempos.
Como conseqüência desse fato é que os resultados provados em Matemática
às vezes podem tornar seus descobridores ilustres e se aplicam através dos
séculos. Por exemplo, o Teorema de Pitágoras é utilizado até hoje do mesmo
modo que foi descoberto há mais de 2.000 anos e seguirá do mesmo jeito
nos próximos 2.000 anos.
Vamos ilustrar melhor essa diferença com um exemplo em Geografia:
Hoje, todos nós sabemos que a Terra tem aproximadamente o formato de
uma laranja, um pouco achatada nos pólos. Porém, na época de Pitágoras,
um dos grandes temores dos navegadores era encontrar o fim-do-mundo.
No pensamento de alguns destes aventureiros, a Terra tinha o formato de
um cubo, e uma vez chegando em um dos seus extremos, o navio despen-
caria no vazio. Esse é um dos muitos exemplos de como a concepção da

i i

i i
“Livro˙Olimpiada
i i
2006/5/27
page 14
i i

14 [CAP. 1: PRIMEIROS PASSOS

natureza mudou ao longo do tempo, transformando uma proposição ver-


dadeira num perı́odo da humanidade em algo completamente falso em outra
época. Porém, para nossa felicidade, isso não acontece na Matemática.
Uma proposição ou é verdadeira ou é falsa e permanecerá deste modo pelos
milênios.
Mas como saber se uma proposição é verdadeira ou falsa? A primeira
coisa que devemos fazer é tomar muito cuidado. As aparências enganam
ou, como diziam nossos avós, “nem tudo que reluz é ouro.”O leitor, avisado
disso, pense agora na seguinte pergunta:
• Qual é a chance de que pelo menos duas pessoas num ônibus com 44
passageiros façam aniversário no mesmo dia do ano?
Como já avisamos, o leitor deve ter cuidado ao responder a pergunta
acima, pois podemos nos enganar muito facilmente. Por exemplo, podemos
formular o seguinte argumento errado: o ano tem 365 dias e, como estou
escolhendo um grupo de 44 pessoas ao acaso, é claro que a chance de que
num grupo de 44 pessoas pelo menos duas delas façam aniversário no mesmo
dia do ano é pequena, pois 44 é um número muito pequeno com respeito
a 365. À primeira vista isso pode até parecer verdadeiro, mas com uma
análise mais cuidadosa veremos que é completamente falso. Na verdade,
a chance de que pelo menos duas pessoas do ônibus façam aniversário no
mesmo dia do ano é de cerca de 80%!
Quem não acreditar nisto pode fazer duas coisas: primeiro, ir a sua
sala de aula ou no seu ônibus escolar, que deve ter pelo menos 40 pessoas,
e fazer o experimento ao vivo. Muito provavelmente você deve conseguir
duas pessoas que fazem aniversário no mesmo dia do ano. Se você verifica
que existiam duas pessoas que faziam aniversário no mesmo dia do ano, não
foi por acaso, pois a chance de isso acontecer era muito alta. Mas, cuidado!
Isso não é uma prova matemática para este fato. Para provar que este fato é
verdadeiro você deve verificar que se escolhermos ao acaso um grupo de 44
pessoas então com 80% de chance pelo menos duas delas fazem aniversário
no mesmo dia do ano.
Porém, se você faz o experimento e não encontra duas pessoas que fazem
aniversário no mesmo dia do ano, não se desespere. Lembre-se de que se
trata de algo que acontece com chance de 80% e que pode não acontecer
quando fazemos um teste. Em qualquer um dos casos, para ter a certeza de
que a proposição é verdadeira o leitor deve passar para o Capitulo 3, onde
demonstraremos matematicamente que o que afirmamos é verdade.

i i

i i
“Livro˙Olimpiada
i i
2006/5/27
page 15
i i

[SEC. 1.3: VERDADEIRO OU FALSO? 15

Vamos analisar agora outro fato aparentemente óbvio:


• Num campeonato de futebol onde cada time joga a mesma quantidade
de jogos, cada vitória vale três pontos, o empate vale um ponto e a
derrota nenhum ponto. Em caso de empate, o critério de desempate
entre as equipes era o seguinte:
- A melhor equipe é aquela que tem mais vitórias.
Os organizadores decidiram passar a adotar o critério:
- A melhor equipe é aquela que tem mais derrotas.
Você acha que este critério é justo?
Com respeito a esta pergunta, a resposta do leitor deve ter sido que a
mudança de critério é totalmente injusta (acertamos a sua resposta?), pois
um time que perdeu mais é pior que um que perdeu menos. Na verdade, não
houve mudança nenhuma de critério! Para ver isso rapidamente, lembre-se
de que se a equipe A perdeu mais que a equipe B e ainda assim empataram,
então ela deve ter ganho mais, para que no fim do campeonato a equipe A
ainda assim conseguisse empatar com a equipe B. Vamos mostrar isso
precisamente. Sejam a1 , a2 , a3 o número de derrotas, empates e vitórias,
respectivamente, da equipe A. Do mesmo modo, sejam b1 , b2 , b3 o número de
derrotas, empates e vitórias, respectivamente, da equipe B. Nossa hipótese
é de que a equipe A obteve mais vitórias que a equipe B, ou seja, que
a3 > b3 . Como cada equipe jogou o mesmo número de jogos, temos que
a1 + a2 + a3 = b1 + b2 + b3 . (1.1)
Por outro lado, note que o número de pontos obtidos pela equipe A é a2 +
3a3 . Do mesmo modo, o número de pontos obtidos pela equipe B é igual a
b2 + 3b3 . Como as duas empataram, temos que:
a2 + 3a3 = b2 + 3b3 .
Ou ainda,
b2 − a2
3(a3 − b3 ) = b2 − a2 ou b3 − a3 = − .
3
Como a3 − b3 > 0 temos que b2 − a2 > 0. Reescrevendo a equação (1.1),
temos que:
b2 − a2 2
a1 − b1 = b2 − a2 + (b3 − a3 ) = b2 − a2 − = (b2 − a2 ).
3 3

i i

i i
“Livro˙Olimpiada
i i
2006/5/27
page 16
i i

16 [CAP. 1: PRIMEIROS PASSOS

Logo, temos que a1 − b1 > 0, pois b2 − a2 > 0. Isso significa que A teve
mais derrotas que B, como querı́amos demonstrar.
Assim, como estes dois exemplos mostram, ao depararmos com um prob-
lema em Matemática, devemos ter cuidado ao tirar conclusões apressadas
para evitar que cometamos algum engano. Pode acontecer que uma situação
que é claramente falsa para um observador menos atento, se mostre ver-
dadeira quando fazemos uma análise mais criteriosa.

1.4 O Método de Redução ao Absurdo


“Um detetive está investigando um caso de roubo de um objeto que ocorreu
numa sala onde havia somente duas pessoas. Naturalmente, o criminoso
se encontra entre as duas pessoas que estavam na sala. Após investigar
criteriosamente a primeira pessoa, ele chega à conclusão de que esta pessoa
é inocente. Logo, o crime foi cometido pela segunda pessoa.”
Apesar de sua simplicidade, o método descrito no parágrafo anterior é
uma das ferramentas mais poderosas da Matemática. Nós conhecemos este
método pelo nome de Método de Redução ao Absurdo ou Método do Terceiro
Excluı́do. Resumidamente, se uma afirmação não é verdadeira, ela só pode
ser falsa. Para escrever o método em termos de proposições, devemos seguir
o seguinte roteiro:

• Assumindo a Hipótese como verdade, tentamos mostrar que a nossa


Tese é verdadeira.

• Se não conseguimos isso diretamente, supomos que a Tese é falsa.

• Após argumentarmos corretamente, deduzimos que a Hipótese tem


que ser falsa. Isso é um absurdo, pois estamos assumindo que a
Hipótese é verdadeira.

• Deste modo, não podemos supor que a tese é falsa, onde ela deve ser
verdadeira.

Em termos de proposições, podemos concluir que o Método de Redução


ao Absurdo pode ser formulado do seguinte modo:

• Para mostrar que uma proposição é verdadeira, basta mostrar


que sua forma contra-recı́proca é verdadeira.

i i

i i
“Livro˙Olimpiada
i i
2006/5/27
page 17
i i

[SEC. 1.4: O MÉTODO DE REDUÇÃO AO ABSURDO 17

Em resumo, podemos dizer que o método da redução ao absurdo é:

Forma Direta ⇔ Forma Contra-recı́proca

O sı́mbolo ⇔ significa que a Forma Direta de uma proposição é verdadeira


se, e somente se, a sua Forma Contra-recı́proca também o for.
Vamos mostrar como esse método funciona na prática. Dizemos que um
número natural n é par, se existir outro número natural k tal que n = 2k.
No caso contrário, este número é chamado de ı́mpar e se escreve da forma
2k + 1 para algum número natural k.
Considere a seguinte proposição:

Proposição 1.2. Se n é par, então n2 é par.

• Hipótese: n é par.

• Tese: n2 é par.

Apesar de parecer óbvia, esta proposição merece uma prova matemática


rigorosa. Vamos dar essa prova:

Demonstração. Se n é par, então pela definição de número par, deve existir


um número natural k tal que

n = 2k.

Logo, n2 = (2k)(2k) = 2(2k 2 ). Denotando q = 2k 2 , temos que n2 = 2q é


par também.

Vamos agora considerar uma outra proposição:

Proposição 1.3. Se n2 é par, então n é par.

• Hipótese: n2 é par.

• Tese: n é par.

i i

i i
“Livro˙Olimpiada
i i
2006/5/27
page 18
i i

18 [CAP. 1: PRIMEIROS PASSOS

n2 n
4 2
16 4
36 6
64 8
100 10
144 12

Desafiamos o leitor a tentar mostrar esta proposição partindo da hipótese e


tentando concluir a tese. Note que podemos verificar que nossa proposição
é verdadeira para vários valores de n2 como na tabela abaixo, mas isso não
é uma prova matemática da nossa proposição.

Mesmo verificando para um bilhão de valores de n2 , sempre nos restariam


números para serem verificados. Como nossas tentativas de provar a forma
direta desta proposição estão sendo frustradas, neste momento iremos apelar
para o Método de Redução ao Absurdo, ou seja, para mostrar a Proposição 1.3,
iremos mostrar a sua forma contra-recı́proca:

• Negativa da Tese: n não é par.

• Negativa da Hipótese: n2 não é par.

Demonstração. Como estamos assumindo que n não é par, logo n tem que
ser ı́mpar, ou seja, existe k, número natural, tal que n = 2k + 1. Logo,

n2 = (2k + 1)(2k + 1)
= 4k 2 + 2k + 2k + 1
= 4k 2 + 4k + 1
= 2(2k 2 + 2k) + 1,

chamando q = 2k 2 + 2k, temos que:

n2 = 2q + 1

Logo, n2 é ı́mpar, de onde concluı́mos a prova.

i i

i i
“Livro˙Olimpiada
i i
2006/5/27
page 19
i i

[SEC. 1.5: DICAS PARA RESOLVER PROBLEMAS 19

Isto mostra um pouco do poder do Método de Redução ao Absurdo.


Antes de usá-lo, não tı́nhamos a menor idéia de como mostrar a proposição 2.
Aplicando o método, construı́mos uma proposição equivalente a proposição
2, cuja prova conseguimos obter sem maiores dificuldades.

1.5 Dicas para Resolver Problemas


Nesta seção, estudaremos algumas “regras gerais”que devemos ter em mente
na hora de resolver um problema de matemática. Aplicaremos estas “re-
gras”a alguns problemas interessantes para ilustrar sua importância. Vamos
à primeira regra:

• Regra 1: Ler bem o enunciado do problema e utilizar todas as in-


formações disponı́veis.

Problema 1.4. Ao encontrar uma velha amiga, um matemático tem a


seguinte conversa:

M: Como vão os três filhos da senhora?


S: Vão bem, obrigada!
M: Qual a idade deles mesmo?
S: Vou lhe dar dicas. O produto das idades deles é 36.
M: Só com essa dica é impossı́vel!
S: A soma das idades é igual ao número daquela casa (apontou para uma
casa em frente aos dois).
M: Ainda não sei!
S: O mais velho toca piano!
M: Agora eu sei!
Você é capaz de descobrir as idades dos três filhos da senhora?

Solução. É muito importante neste problema tirar o máximo de informação


das dicas da senhora. Vamos à primeira dica: o produto das idades é 36.
Digamos que as idades dos filhos sejam 0 6 x 6 y 6 z 6 36. Como
xyz = 36, temos as seguintes possibilidades:

i i

i i
“Livro˙Olimpiada
i i
2006/5/27
page 20
i i

20 [CAP. 1: PRIMEIROS PASSOS

x y z
1 1 36
1 2 18
1 3 12
1 4 9
1 6 6
2 2 9
2 3 6
3 3 4

Vamos agora calcular a soma das idades:

x y z x+y+z
1 1 36 38
1 2 18 21
1 3 12 16
1 4 9 14
1 6 6 13
2 2 9 13
2 3 6 11
3 3 4 10

Observe que não conhecemos a princı́pio o número da casa. Porém, sabe-


mos que após a segunda dica, o matemático ainda não conseguiu deduzir as
idades das crianças.
Por que ele não conseguiu? Imagine que o número da casa fosse 14. Ora,
de acordo com nossa tabela, só existe um terno de números cujo produto é
36 e a soma é 14, que é o terno (1,4,9). Assim, se o número da casa fosse
14 o matemático teria dado a resposta após a segunda dica. Como ele ficou
em dúvida, olhando a tabela 2, chegamos à conclusão de que o número da
casa só pode ser igual a 13.
Última dica: O mais velho toca piano.
No inı́cio essa dica parecia inútil, mas agora ela é fundamental para resolver-
mos o problema. De fato, como o mais velho toca piano, isso significa que

i i

i i
“Livro˙Olimpiada
i i
2006/5/27
page 21
i i

[SEC. 1.5: DICAS PARA RESOLVER PROBLEMAS 21

existe um mais velho, o que descarta o caso (1,6,6). Assim, as idades são 2,
2, e 9.

Problema 1.5. Numa cesta encontram-se 9 moedas idênticas, sendo que


oito delas têm o mesmo peso e uma moeda é mais leve que as demais.
Usando duas vezes uma balança de dois pratos, encontrar a moeda mais
leve.

Solução. Este é o tipo de problema que a primeira vista pode parecer difı́cil,
mas que quando usamos todas as informações do seu enunciado se torna
fácil. A idéia é dividir as moedas em três grupos de três moedas cada, que
chamaremos grupos A, B e C. Colocaremos na balança os grupos A e B e
deixaremos o grupo C fora. Podem acontecer duas coisas:

(a) Os pratos ficam equilibrados;

(b) Os pratos ficam desequilibrados.

No caso 1, temos que os grupos A e B têm o mesmo peso. Logo, a moeda


mais leve deve estar no grupo C. No caso 2, um dos grupos ficou mais leve,
o que significa que a moeda mais leve está neste grupo. Assim, utilizando
a balança apenas uma vez conseguiremos descobrir qual é o grupo em que
a moeda mais leve está. Digamos que este grupo seja o grupo A. Para
achar a moeda mais leve, procedemos de modo semelhante ao que fizemos
anteriormente: separamos as três moedas do grupo A colocando uma em
cada prato e deixando a terceira de fora. Podem acontecer duas coisas:

(a) Os pratos ficam desequilibrados e assim a moeda mais leve está no


prato mais leve;

(b) Os pratos ficam equilibrados, logo a moeda mais leve foi a que ficou
fora.

No final, usamos a balança exatamente duas vezes.

• Regra 2: Fazer casos particulares ou casos mais simples de problemas


similares, para adquirir familiaridade com o problema.

Problema 1.6. Numa pequena ilha existem 5 pessoas de olhos azuis e 5


pessoas de olhos verdes. Existe um grande tabu nesta ilha que é o seguinte:

i i

i i
“Livro˙Olimpiada
i i
2006/5/27
page 22
i i

22 [CAP. 1: PRIMEIROS PASSOS

se uma pessoa descobre que possui olhos azuis ela se suicida à meia-noite
do dia em que descobriu, pulando do alto da prefeitura. Por conta disso,
ninguém conversa sobre o assunto, olha para espelhos ou vê seu reflexo na
água. Todos se cruzam diariamente e conhecem os olhos de seus amigos.
Numa manhã, um estrangeiro chegou a ilha e reuniu as dez pessoas para o
seguinte pronunciamento:

“Nesta ilha, existe uma pessoa de olhos azuis.”

Pergunta-se:

1. O que aconteceu com os habitantes da ilha?

2. Que informação nova o estrangeiro trouxe?

Dicas:

1. Fazer o caso uma pessoa com olhos azuis e uma com olhos verdes;

2. Fazer o caso de duas pessoas de olhos azuis e duas de olhos verdes.

Solução. Como em muitos problemas de matemática, abordar casos mais


simples do problema pode ajudar bastante na solução. Assim, vamos imagi-
nar o seguinte caso mais simples: na Ilha existe somente uma pessoa de olhos
azuis e a outra de olhos verdes. Pensando neste caso, a pessoa que tinha
olhos azuis só via as que tinham olhos verdes. Quando o estrangeiro afirmou
que existia uma pessoa de olhos azuis, ela descobriu que tinha olhos azuis,
pois as outras pessoas tinham olhos verdes. Assim, à meia-noite ela subiu
na prefeitura e pulou. Com isso, a pessoa que tinha olhos verdes descobriu
que tinha olhos verdes, pois se ela tivesse olhos azuis sua companheira não
se suicidaria no dia anterior.

Vamos agora dar um passo crucial na solução do nosso problema original,


considerando o caso onde existem duas pessoas de olhos azuis e duas pessoas
de olhos verdes na ilha. Vamos chamar as pessoas de olhos azuis de A e
B e as pessoas de olhos verdes de C e D. No dia em que o estrangeiro
fez o seu pronunciamento, nada aconteceu, pois as pessoas C e D viam as
pessoas A e B com olhos azuis e a pessoa A via a pessoa B com olhos azuis
e vice-versa. Já no segundo dia, a pessoa A teve o seguinte pensamento:

i i

i i
“Livro˙Olimpiada
i i
2006/5/27
page 23
i i

[SEC. 1.5: DICAS PARA RESOLVER PROBLEMAS 23

“Se eu tivesse olhos verdes, a pessoa B teria descoberto que tinha


olhos azuis ontem, pois ela veria três pessoas de olhos verdes.
Como ela não se suicidou ontem, eu tenho olhos azuis.”

Pensando da mesma forma, a pessoa B descobriu que também tinha


olhos azuis. Por isso, à meia noite do segundo dia, as pessoas A e B se
suicidaram.
O que aconteceu depois? As pessoas C e D ainda tinham a dúvida da
cor de seus olhos. Para chegar à conclusão de que seus olhos são verdes, no
terceiro dia, a pessoa C pensou assim:

Bem, se eu tivesse olhos azuis, as pessoas A e B veriam cada


uma duas pessoas com olho azul. Logo, elas não teriam se sui-
cidado no segundo dia, pois não conseguiriam deduzir a cor de
seus olhos. Logo, tenho olhos verdes. Ufa!

Do mesmo modo, a pessoa D conseguiu descobrir a cor de seus olhos.


Analisando de modo semelhante, conseguiremos deduzir que no pro-
blema original as cinco pessoas de olhos azuis descobrirão que possuem
olhos azuis e juntas se suicidarão no quinto dia após o pronunciamento do
estrangeiro.
Agora vamos descobrir a resposta da segunda pergunta do enunciado:
que informação nova o estrangeiro trouxe? Aparentemente nada de novo
foi acrescentado pela frase do estrangeiro, pois cada pessoa estava vendo
alguma pessoa com olhos azuis. Mas isso não é verdade.
Para ver isso e descobrir qual é a nova informação que o estrangeiro
trouxe, vamos voltar ao caso de somente duas pessoas na Ilha, uma de olhos
azuis e outra de olhos verdes. Neste caso, a pessoa de olhos azuis somente
vê uma pessoa de olhos verdes. Com a informação de que existe uma pessoa
de olhos azuis ela pode descobrir a cor de seus olhos. Note que a pessoa de
olhos verdes já sabia que existia pelo menos uma pessoa de olhos azuis. Mas
ela não sabia que a pessoa de olhos azuis tinha conhecimento de que na Ilha
existia alguém com olhos azuis. Essa é a nova informação que o estrangeiro
trouxe.

Problema 1.7. Um viajante deseja se hospedar durante 31 dias num hotel.


Entretanto, percebe que está sem dinheiro e que a única coisa que possui é
uma corrente com 31 elos de ouro (veja figura). Para pagar sua conta, ele

i i

i i
“Livro˙Olimpiada
i i
2006/5/27
page 24
i i

24 [CAP. 1: PRIMEIROS PASSOS

acertou com o gerente pagar um elo por dia, sem atrasar ou adiantar o
pagamento, durante os 31 dias. O gerente pode dar troco em elos. Depois
ele deseja recuperar a corrente e por isso ele quer pagar a conta cortando
a corrente no menor número de pedaços. Quantos cortes você conseguiria
dar e pagar a conta?

Solução. Uma primeira solução é cortar a corrente 30 vezes, separando todos


os elos. Porém, essa não é a melhor solução, como veremos a seguir. Vamos
iniciar nossa análise observando que para pagar o primeiro dia precisamos
dar um corte na corrente. Assim, o gerente receberá um elo. O “pulo do
gato”do problema vem agora: para pagar o 2◦ dia, vamos cortar a corrente
de modo a separar dois elos de uma vez. Assim, daremos dois elos ao gerente
e ele devolverá um elo de troco. Com este elo pagaremos o terceiro dia. Note
que pagamos três dias fazendo dois cortes na corrente, como mostra a tabela:

Gerente Viajante
Elos 1, 2 28

Note que o número 2 denota o pedaço que contém 2 elos. Para pagar o
4◦ dia, cortaremos a corrente de modo a obter um pedaço com quatro elos.
Entregamos ao gerente este pedaço e recebemos de troco um elo solto e um
pedaço com dois elos. Com o elo solto, pagamos o 5◦ dia. Assim, no 5◦ dia
teremos o seguintes grupos de elos:

Gerente Viajante
Elos 1, 4 2, 24

Assim, pagamos o 6◦ dia com o pedaço que contém dois elos e rece-
beremos o elo solto de troco. Finalmente pagaremos o 7◦ dia com o elo
solto. Note que foi possı́vel pagar 7 dias com apenas três cortes na corrente.
A continuação do procedimento está quase revelada. Para pagar o 8◦ dia,
cortaremos um pedaço com oito elos. Daremos este pedaço e receberemos
de troco 7 elos, sendo um elo solto, um pedaço com 4 e um pedaço com dois
elos. Repetindo o procedimento anterior, pagaremos os 7 dias seguintes, pa-
gando até o 15◦ dia sem precisar de cortes adicionais. Ou seja, para pagar

i i

i i
“Livro˙Olimpiada
i i
2006/5/27
page 25
i i

[SEC. 1.5: DICAS PARA RESOLVER PROBLEMAS 25

os 15 primeiros dias, precisamos de 4 cortes na corrente. Neste momento, a


corrente está distribuı́da do seguinte modo:

Gerente Viajante
Elos 1, 2, 4, 8 16

Para pagar o 16◦ dia, entregaremos ao gerente o pedaço com os 16 elos


restantes, recebendo 15 elos divididos em pedaços de 1, 2, 4 e 8 elos. Se
repetirmos o processo, pagaremos o hotel até o 31◦ dia sem precisar de novos
cortes. Assim, o mı́nimo número de cortes é 4.

• Regra 3: Mudar a representação do problema, transformando-o em


um problema equivalente.

Problema 1.8. Sabendo que em cada jogada o movimento do cavalo con-


siste em se deslocar duas casas na horizontal e uma na vertical ou duas
na vertical e uma na horizontal, decidir se é possı́vel sair do tabuleiro 1 e
chegar ao tabuleiro 2 sem que em algum momento existam dois cavalos na
mesma casa.

(a) (b)

Figura 1.1::

Solução: Para resolver este problema vamos usar a estratégia de mudar a


representação. O que significa isso? Vamos reescrever o problema com

i i

i i
“Livro˙Olimpiada
i i
2006/5/27
page 26
i i

26 [CAP. 1: PRIMEIROS PASSOS

outros ingredientes, porém sem alterar em nada sua essência. Primeira-


mente, enumere as casas do tabuleiro com os números 1, 2, . . . , 9, como na
Figura 1.5.

1 2 3

4 5 6

7 8 9

Figura 1.2::

Vamos agora associar ao tabuleiro, um conjunto de nove pontos também


enumerados com os números 1,2, . . . , 9. Se for possı́vel sair de uma casa i e
chegar à casa j com apenas uma jogada do cavalo, colocaremos um segmento
ligando os pontos i e j. Por exemplo, é possı́vel, saindo da casa 1 chegar
à casa 6 e a casa 8. Deste modo, o ponto com número 1 está ligado com
o ponto com número 8. Se analisarmos todas as possı́veis ligações entre os
pontos obteremos a seguinte figura:

9
2• •
•4

7 • • • 3

• •
6 • 8
1

Figura 1.3::

Assim, se colocarmos os cavalos como no tabuleiro 1, teremos a situação


descrita na Figura 1.4. Deste modo, fica evidente que não podemos trocar a

i i

i i
“Livro˙Olimpiada
i i
2006/5/27
page 27
i i

[SEC. 1.5: DICAS PARA RESOLVER PROBLEMAS 27

posição dos cavalos branco e preto sem que em algum momento eles ocupem
a mesma casa.
CP
9
2• •
•4

CP 7 • • • 3 CB

• •
6 • 8
1
CB

Figura 1.4: CB=cavalo branco e CP=cavalo preto.

• Regra 4: Use sua imaginação pesquisando caminhos alternativos.


Extrapole seus limites!

Para exemplificar como o enunciado de um problema pode limitar nossa


imaginação, vamos considerar o problema abaixo:
Exemplo 1.9. Mostre que podemos cobrir os 9 pontos no reticulado abaixo
traçando 4 segmentos de reta sem tirar o lápis do papel.

• • •

• • •

• • •

Figura 1.5: Cobrir os 9 pontos com quatro segmentos

O problema acima é o tı́pico problema de bar. Vou explicar o porquê


da terminologia estranha: quando você encontra um conhecido que você
não vê há muito tempo e fala que está estudando Matemática, ele vem logo

i i

i i
“Livro˙Olimpiada
i i
2006/5/27
page 28
i i

28 [CAP. 1: PRIMEIROS PASSOS

com um desses desafios, imaginando que você deve ser uma máquina de
resolver problemas. Resolva isso, afirma ele, esperando que você não consiga
imaginar a solução. Para se vingar desses inoportunos, daremos a solução
desse problema e complementaremos a receita dando uma generalização no
exemplo 5.11 do Capı́tulo 5.
A solução desse problema passa por extrapolar o convencional. Podemos
tentar várias vezes cobrir os nove pontos com quatro segmentos sem tirar o
lápis do papel, mas só iremos de fato conseguir fazer isso se libertarmos as
nossas idéias e nos permitirmos sair um pouco do convencional. Se você é
curioso e não quer tentar por si só resolver esse problema, veja a solução no
exemplo 1.9 do Capı́tulo 5. Recomendamos que você tente resolver sozinho
o desafio.

i i

i i
“Livro˙Olimpiada
i i
2006/5/27
page 29
i i

Capı́tulo 2

Divisibilidade

A Teoria dos Números é o ramo da matemática encarregado de desvendar


os mistérios dos números e teve sua origem na antiga Grécia. Os belı́ssimos
problemas ligados a esta área constituem, até hoje, uma das principais fontes
inspiradoras dos amantes da Matemática.
Apesar de se terem passado muitos anos, ainda hoje persistem muitas
questões naturais e simples sem resposta e outras, da mesma natureza, só
têm sido resolvidas recentemente. Isto faz desta área um grande atrativo
para os matemáticos do mundo todo.
Este capı́tulo será dedicado ao estudo de algumas propriedades básicas
relativas aos números inteiros.

2.1 Conceitos Fundamentais e Divisão com


Resto
Denotamos por Z o conjunto dos números inteiros formado pelo conjunto dos
números naturais N = {1, 2, 3, · · · } munido do zero e dos inteiros negativos,
ou seja, Z = {· · · , −3, −2, −1, 0, 1, 2, 3, · · · }.
Começamos observando que a soma, diferença e produto de números
inteiros também serão números inteiros. Entretanto, o quociente de dois
inteiros pode ser um inteiro ou não.

29

i i

i i
“Livro˙Olimpiada
i i
2006/5/27
page 30
i i

30 [CAP. 2: DIVISIBILIDADE

Uma das propriedades fundamentais dos números naturais que utilizare-


mos ao longo do texto é o conhecido Princı́pio da Boa Ordenação. Este
princı́pio pode ser enunciado do seguinte modo:
Princı́pio da Boa Ordenação: Todo subconjunto não-vazio A ⊆ N possui
um elemento menor que todos os outros elementos deste.
Por exemplo, se A é o conjunto dos números pares, o menor elemento de
A é o número 2. Por outro lado, observemos que o conjunto dos números
inteiros não goza da boa ordenação.
Apesar do Princı́pio da Boa Ordenação parecer inocente e natural, muitos
resultados importantes a respeito dos números naturais decorrem do mesmo,
como veremos ao longo de todo este capı́tulo.

Definição 2.1. Sejam a e b inteiros. Dizemos que a divide b, ou a é divisor


de b, ou b é múltiplo de a, se existe um inteiro q tal que b = aq.

Usaremos a notação: a | b para representar todas as frases equivalentes


ditas anteriormente. Se a não for divisor de b, então escreveremos a ∤ b.

Exemplo 2.2. 7 | 21 pois 21 = 7 · 3. Por outro lado 3 ∤ 8 pois considerando


o conjunto M = {3k, k ∈ N} = {3, 6, 9, 12, · · · } dos múltiplos positivos de
3 vemos que 8 não pertence ao mesmo.

A seguinte proposição é um bom exercı́cio para entender os conceitos


enunciados acima.

Proposição 2.3. Sejam a, b e c números inteiros. Então

(a) Se a | b e b | c então a | c;

(b) Se a | b e a | c então a | (b + c) e a | (b − c);

(c) Se a e b são positivos e a | b então 0 < a ≤ b;

(d) Se a | b e b | a então a = b ou a = −b.

Demonstração. Se a | b e b | c então existem inteiros q1 e q2 tais que

b = aq1 (2.1)

e
c = bq2 . (2.2)

i i

i i
“Livro˙Olimpiada
i i
2006/5/27
page 31
i i

[SEC. 2.1: CONCEITOS FUNDAMENTAIS E DIVISÃO COM RESTO 31

Substituindo (2.1) em (2.2) temos que

c = a q1 q 2 , (2.3)
|{z}
q∈Z

provando isto a afirmação feita em (a).


Agora provaremos (b). Com efeito, se a | b e a | c valem as igualdades

b = aq1 , q1 ∈ Z (2.4)

e
c = aq2 , q2 ∈ Z. (2.5)
Operando com os ambos lados das igualdades (2.4) e (2.5) temos que

b + c = a(q1 + q2 ) e b − c = a(q1 − q2 ),
| {z } | {z }
r∈Z s∈Z

obtendo assim o resultado desejado.


Continuamos agora com a prova de (c). De fato, se a | b, sendo ambos
positivos, então b = aq com
q ≥ 1. (2.6)
Logo, multiplicando por a ambos lados de (2.6) temos (como a é positivo)
que
b = aq ≥ a > 0,
como esperávamos.
Finalmente, provaremos (d). Com este propósito observamos que se
a | b e b | a então |a| divide |b| e |b| divide |a|. Portanto, pelo item (c)
temos que |a| ≤ |b| e |b| ≤ |a|, ou seja, |a| ≤ |b| ≤ |a|. Logo, |a| = |b| e
conseqüentemente a = b ou a = −b.

Exemplo 2.4. Prove que o número N = 545362 − 7 não é divisı́vel por 5.

Solução. Vamos mostrar isso utilizando o método do absurdo. Se este


número fosse divisı́vel por 5, então 545362 − 7 = 5q. Logo, 7 = 545362 − 5q,
ou seja, 7 seria divisı́vel por 5, o que é um absurdo.

A próxima proposição apresenta alguns dos critérios de divisibilidade que


são estudados nas lições de matemática do colégio. A prova que daremos

i i

i i
“Livro˙Olimpiada
i i
2006/5/27
page 32
i i

32 [CAP. 2: DIVISIBILIDADE

para estes está baseada na representação de um inteiro positivo na base


decimal. Por exemplo, 345 escreve-se na base decimal da seguinte forma

345 = 300 + 40 + 5 = 3 · 102 + 4 · 10 + 5,

assim como 2768 se escreve da forma

2768 = 2000 + 700 + 60 + 8 = 2 · 103 + 7 · 102 + 6 · 10 + 8.

De modo geral, se denotamos por N = bn bn−1 · · · b1 b0 o número inteiro


positivo formado pelos algarismos bn , bn−1 , · · · , b1 e b0 , nessa ordem, então
N se escreve na base decimal da forma
N = bn 00 · · · 0} + bn−1 00
| {z · · · 0} + · · · + b1 0 + b0
| {z
n vezes n−1 vezes (2.7)
= bn 10n + bn−1 10n−1 + · · · b1 10 + b0

Proposição 2.5 (Critérios de divisibilidade). Seja N um inteiro posi-


tivo, então:
(a) N é divisı́vel por 3 ou por 9, se a soma dos seus dı́gitos é divisı́vel por
3 ou por 9, respectivamente;
(b) N é divisı́vel por 5, se seu último dı́gito for 0 ou 5.
Demonstração. Faremos uso da seguinte propriedade que é muito fácil de
verificar. Dado qualquer número m ∈ N vale que

10m = |99 {z
· · · 9} +1.
m vezes

Como já foi observado, se N = bn bn−1 · · · b0 , então

N = bn 10n + bn−1 10n−1 + · · · + b1 10 + b0


= bn (99
| {z · · · 9} +1) + · · · + b1 (9 + 1) + b0
· · · 9} +1) + bn−1 ( |99 {z
n vezes n−1 vezes

= bn · |99 {z
· · · 9} +bn−1 · |99 {z
· · · 9} + · · · + b1 · 9 + bn + bn−1 + · · · + b0
n vezes n−1 vezes
³ ´
· · · 1} +bn−1 · |11 {z
= 9 bn · |11 {z · · · 1} + · · · + b1 + bn + bn−1 + · · · + b0 .
| {z }
n vezes n−1 vezes S

i i

i i
“Livro˙Olimpiada
i i
2006/5/27
page 33
i i

[SEC. 2.1: CONCEITOS FUNDAMENTAIS E DIVISÃO COM RESTO 33

Então, aplicando o item (b) da Proposição 2.3 temos que se 9 | S, então


também dividirá a N . Reciprocamente, suponhamos que 9 | N e observemos
que S = N − 9k, com

k = a1 · |11 {z
· · · 1} +a2 · |11 {z
· · · 1} + · · · + an−1 .
n−1 vezes n−2 vezes

Então, usando novamente o item (b) da Proposição 2.3 temos que 9 | S.


É óbvio que a prova para o caso da divisibilidade por 3 segue de maneira
idêntica, logo fica provado o item (a).
A prova do item (b) segue de maneira muito semelhante e deixamos a
mesma a cargo do leitor.

Exemplo 2.6. Prove sem fazer muitas contas que o número

N = 13424136 + 1234567890

é divisı́vel por 3.

Solução. Note que não precisamos fazer a soma dos números anteriores.
Para mostrar isso, basta aplicar o item (ii) da Proposição 2.3 e o item (i)
da Proposição 2.5, observando que cada um dos números acima é divisı́vel
por 3, pois a soma de seus dı́gitos é um múltiplo de 3.

O próximo passo de nossa discussão é ver o que acontece quando um


número não é divisı́vel por outro. Por exemplo, analisemos se 31 é divisı́vel
por 7; para isto listamos a diferença entre 31 e os múltiplos positivos de 7,
ou seja,

r1 = 31 − 7 · 1 = 24,
r2 = 31 − 7 · 2 = 17,
r3 = 31 − 7 · 3 = 10,
r4 = 31 − 7 · 4 = 3,
r5 = 31 − 7 · 5 = −4,
r6 = 31 − 7 · 6 = −11,
..
.

i i

i i
“Livro˙Olimpiada
i i
2006/5/27
page 34
i i

34 [CAP. 2: DIVISIBILIDADE

Claramente 31 não é divisı́vel por 7, pois caso contrário terı́amos que alguma
das diferenças acima seria igual a zero, o que é impossı́vel pois as diferenças
rq := 31 − 7q com 1 ≤ q ≤ 4 são todas positivas e com q ≥ 5 são todas
negativas. Entretanto, notamos que dentre as diferenças positivas a única
que é menor que 7 corresponde ao caso q = 4. O resultado seguinte nos
diz o que acontece no caso geral da divisão de um inteiro b por um inteiro
positivo a.
Teorema 2.7 (Algoritmo da Divisão). Dados dois inteiros a e b, com
a > 0, existem únicos inteiros q e r tais que

b = aq + r, 0 ≤ r < a.

Se a ∤ b, então r satisfaz a desigualdade estrita 0 < r < a.


Demonstração. Por simplicidade, suporemos que b é positivo. Se b < a,
basta tomar q = 0 e r = b. Se b = a, então tomamos q = 1 e r = 0. Assim,
assumiremos também que b > a > 0. Consideremos o conjunto

R = {b − aq ∈ Z; b − aq ≥ 0} ⊆ N ∪ {0} (2.8)
Notemos que o conjunto R é não-vazio, pois b − a ∈ R, já que b − a > 0.
Deste modo, pelo Princı́pio da Boa Ordenação temos que R admite um
menor elemento, que denotaremos por r. Claramente r = b − aq, para
algum q ≥ 0; assim como r ≥ 0, pois foi escolhido com essa condição. Além
disso, r < a pois caso contrário

r = b − aq ≥ a ⇒ b − a(q + 1) ≥ 0. (2.9)

Aliás,
a > 0 ⇒ b − a(q + 1) < b − aq. (2.10)
Das desigualdades (2.9) e (2.10) segue que

0 ≤ b − a(q + 1) < b − aq,

contradizendo o fato de que r = b − aq é o menor elemento não-negativo de


R.
Agora provaremos que de fato r e q escolhidos desta forma são únicos.
Com efeito, suponhamos que existem outros inteiros r1 e q1 tais que

b = aq1 + r1 , 0 ≤ r1 < a.

i i

i i
“Livro˙Olimpiada
i i
2006/5/27
page 35
i i

[SEC. 2.1: CONCEITOS FUNDAMENTAIS E DIVISÃO COM RESTO 35

Então resulta que aq + r = aq1 + r1 . Logo,

(r − r1 ) = (q1 − q)a; (2.11)

sendo assim, r − r1 é múltiplo de a. Mas em virtude de −a < r − r1 < a, o


único valor que r − r1 pode tomar, sendo este múltiplo de a, é r − r1 = 0.
Portanto, r = r1 , de onde se deduz diretamente de (2.11) que q = q1 .

Os números q e r no enunciado do teorema acima são chamados, respec-


tivamente, de quociente e resto da divisão de b por a.
Os exemplos a seguir apresentam a utilidade do Teorema 2.7.

Exemplo 2.8. Encontrar o último dı́gito do número 22043 .

Solução. É claro que este número é muito grande e seria muito tedioso
multiplicar 2043 vezes o número 2 para descobrir o último dı́gito de 22043 .
Entretanto, se calculamos as primeiras potências vemos que existe regular-
idade nos resultados, como mostra a tabela a seguir.

21 =2 22 = 4 23 =8 24 = 16
25 = 32 26 = 64 27 = 128 28 = 256
29 = 512 210 = 1024 211 = 2048 212 = 4096
213 = 8192 214 = 16384 215 = 32768 216 = 65536

Agora observamos que:

• na primeira coluna da tabela só aparecem potências da forma 24k+1 ,


com k = 0, 1, 2, 3, · · · , e todas elas terminam em 2;

• na segunda coluna da tabela só aparecem potências da forma 24k+2 ,


com k = 0, 1, 2, 3, · · · , e todas elas terminam em 4;

• na terceira coluna da tabela só aparecem potências da forma 24k+3 ,


com k = 0, 1, 2, 3, · · · , e todas elas terminam em 8;

• na quarta coluna da tabela só aparecem potências da forma 24k , com


k = 1, 2, 3, · · · , e todas elas terminam em 6.

Portanto, só resta saber que resto deixa 2043 na divisão por 4. Como
2043 = 510 · 4 + 3, o número 22043 aparecerá na terceira coluna da tabela,
logo o último dı́gito deste é 8.

i i

i i
“Livro˙Olimpiada
i i
2006/5/27
page 36
i i

36 [CAP. 2: DIVISIBILIDADE

O próximo exemplo, como veremos, motiva a procura de caminhos efi-


cientes para encontrar o resto que deixa um número quando é dividido por
outro.
Exemplo 2.9. Um turista brasileiro chega a Cuba e troca parte de seu
dinheiro na casa de câmbio, recebendo 175 notas de 50 pesos e 213 notas
de 20 pesos. Ele decide trocar este dinheiro pela maior quantidade possı́vel
das famosas moedas de 3 pesos cubanos, devido às mesmas terem gravadas
a imagem do guerrilheiro Che Guevara. Quanto sobrou do dinheiro depois
de fazer a troca pelas moedas?
Solução. O problema é fácil de resolver. Basta achar o resto que deixa o
número N = 175·50+213·20 quando é dividido por 3. Entretanto, queremos
destacar que não é preciso fazer os produtos e a soma envolvidos no número
N . Em lugar de fazer isto substituı́mos cada número que aparece em N
pelo resto que este deixa na divisão por 3, formando assim um novo número
N1 , ou seja,
N1 = 1 · 2 + 0 · 2 = 2.
Agora procuramos o resto que N1 deixa na divisão por 3, que obviamente
é 2. A surpresa é que este resto é o mesmo que deixa N na divisão por 3.
Logo, sobraram 2 pesos depois de fazer a troca.
A solução do exemplo anterior é uma aplicação particular do seguinte
lema que é de muita utilidade na resolução de problemas.
Lema 2.10 (Lema dos Restos). A soma e o produto de quaisquer dois
números naturais deixa o mesmo resto que a soma e o produto dos seus
restos, na divisão por um inteiro positivo a.
Demonstração. Sejam N1 , N2 ∈ Z. Fazendo a divisão com resto de ambos
números por a temos que

N1 = aq1 + r1 e N2 = aq2 + r2 ,

com 0 ≤ r1 , r2 < a. Então


N1 N2 = (aq1 + r1 )(aq2 + r2 )
= a2 q1 q2 + aq1 r2 + aq2 r1 + r1 r2
(2.12)
= a(aq1 q2 + q1 r2 + q2 r1 ) + r1 r2
= aq + r1 r2 ,

i i

i i
“Livro˙Olimpiada
i i
2006/5/27
page 37
i i

[SEC. 2.1: CONCEITOS FUNDAMENTAIS E DIVISÃO COM RESTO 37

onde q = aq1 q2 + q1 r2 + q2 r1 . Agora dividimos r1 r2 por a para obtermos

r1 r2 = ap + r, p ∈ Z, 0 ≤ r < a. (2.13)

Das igualdades (2.12) e (2.13) segue que

N1 N2 = aq + ap + r = a(p + q) + r, 0 ≤ r < a. (2.14)

Portanto, de (2.13) e (2.14) concluı́mos que os restos que deixam N1 N2 e


r1 r2 na divisão por a são iguais, ficando provado o resultado para o produto.
A prova para a soma é análoga.
A vantagem do lema é que em certos problemas que envolvem números
muito grandes podemos substituir estes por números muito menores e mais
confortáveis para trabalhar. Vejamos isto através dos seguintes exemplos.
Exemplo 2.11. Prove que o produto de dois números naturais consecutivos
é sempre divisı́vel por 2.
Solução. Se n ∈ N temos que provar que N = n(n + 1) é divisı́vel por 2.
Quando fazemos a divisão de n por 2 temos duas possibilidades para o resto:
r = 0 ou r = 1. Analisemos os dois casos por separado.
• [r = 0] Neste caso o resto que deixa N na divisão por 2 é o mesmo
que o resto que deixa 0(0+1)=0, logo N é divisı́vel por 2.
• [r = 1] Neste caso podemos substituir N por 1(1+1)=2 e o resto
que este último deixa quando é dividido por 2 é 0, logo N também é
divisı́vel por 2.

Mostraremos agora como utilizar o exemplo anterior pra resolver um dos


problemas da 1a Olimpı́ada Brasileira de Matemática.
Exemplo 2.12. Prove que se n é ı́mpar, então n2 − 1 é múltiplo de 8.
Solução. Como n é ı́mpar, podemos escrever n = 2k + 1, para algum k ∈ Z.
Logo
n2 − 1 = (2k + 1)2 − 1 = 4k 2 + 4k + 1 − 1 = 4k 2 + 4k.
Assim,
n2 − 1 = 4k(k + 1).

i i

i i
“Livro˙Olimpiada
i i
2006/5/27
page 38
i i

38 [CAP. 2: DIVISIBILIDADE

Observe que de acordo com o exemplo 2.11, k(k + 1) é múltiplo de 2. Por-


tanto, k(k + 1) = 2q para algum q ∈ Z, de aonde

n2 − 1 = 4k(k + 1) = 4 · 2q = 8q,

como querı́amos demonstrar.

Exemplo 2.13. Prove que em qualquer triângulo retângulo com lados in-
teiros, pelo menos um deles é múltiplo de 3.

Solução. Comecemos analisando quais são os restos possı́veis para a divisão


por 3 de um número que é quadrado. De acordo com o Lema dos Restos
temos a seguinte tabela para os restos de n e n2 , na divisão por 3:

n n2
0 0
1 1
2 1

Resumindo, se um número não é múltiplo de 3 então o resto da divisão de


seu quadrado por 3 deve ser igual a 1.
Agora denotemos por a e b os catetos e por c a hipotenusa. Suponhamos
que nenhum deles é divisı́vel por 3. Então a2 e b2 deixam resto 1 na divisão
por 3. Logo, a2 + b2 deixa resto 12 + 12 = 2 na divisão por 3; mas isto é
uma contradição pois, pelo Teorema de Pitágoras, a2 + b2 = c2 e c2 deixa
resto 1 quando é dividido por 3.

2.2 Máximo Divisor Comum e Mı́nimo Múltiplo


Comum
Nesta seção estudaremos dois conceitos fundamentais, que aparecem natu-
ralmente em vários problemas de divisibilidade, assim como a relação exis-
tente entre eles.
O primeiro destes conceitos está relacionado com os inteiros positivos
que dividem simultaneamente a dois inteiros pré-fixados e é denominado
Máximo Divisor Comum.
Daqui por diante só consideraremos os divisores positivos dos números.

i i

i i
“Livro˙Olimpiada
i i
2006/5/27
page 39
i i

[SEC. 2.2: MÁXIMO DIVISOR COMUM E MÍNIMO MÚLTIPLO COMUM 39

Definição 2.14 (Máximo Divisor Comum). Sejam a e b inteiros difer-


entes de zero. O máximo divisor comum, resumidamente m.d.c, entre a e b
é o número d que satisfaz as seguintes condições:
(a) d é um divisor comum de a e b, isto é, d | a e d | b;
(b) d é o maior inteiro positivo com a propriedade (a).
Neste caso, denotamos o m.d.c entre a e b por d = m.d.c(a, b) ou por
d = (a, b). Se (a, b) = 1, então dizemos que a e b são primos entre si.
Exemplo 2.15. Observando que 12 = 6·2, 18 = 6·3 temos que m.d.c.(12, 18) =
6. Por outro lado, m.d.c.(4, 15) = 1, logo os números 4 e 15 são primos entre
si.
Vejamos agora algumas das propriedades mais importantes dos divisores
comuns de dois inteiros.
Proposição 2.16. Sejam a e b dois inteiros. Então valem as seguintes
afirmações.
(a) Se a é múltiplo de b, então (a, b) = b;
(b) Se a = bq + c, c 6= 0, então o conjunto dos divisores comuns dos
números a e b coincide com o conjunto dos divisores comuns dos
números b e c. Particularmente, (a, b) = (b, c).
Demonstração. Começamos com a prova de (a). Com efeito, todo divisor
comum dos números a e b é um divisor de b. Reciprocamente, usando que
a é múltiplo de b, todo divisor de b é também um divisor de a, ou seja,
um divisor comum dos números a e b. Portanto, o conjunto dos divisores
comuns dos números a e b é igual ao conjunto dos divisores de b. Como o
maior divisor de b é ele mesmo, resulta que (a, b) = b.
Vejamos (b). Usando o item (b) da Proposição 2.3 temos que todo
divisor comum de a e b também divide c e, conseqüentemente, é um divisor
de b e c. Pela mesma razão todo divisor comum de b e c também divide a
e, conseqüentemente, é um divisor de a e b. Portanto os divisores comuns
de a e b são os mesmos que os divisores comuns de b e c. Particularmente,
também coincidem os maiores divisores comuns, ou seja, (a, b) = (b, c).
O teorema a seguir é uma das ferramentas básicas na resolução de prob-
lemas que envolvem o m.d.c entre dois números. O mesmo é devido ao
matemático francês Étienne Bézout (1730-1783).

i i

i i
“Livro˙Olimpiada
i i
2006/5/27
page 40
i i

40 [CAP. 2: DIVISIBILIDADE

Teorema 2.17 (Teorema de Bézout). Se d é o m.d.c de a e b, então


existem números inteiros x0 e y0 tais que d = (a, b) = ax0 + by0 .
Demonstração. Considere a combinação linear ax+by, onde x e y percorrem
todos os inteiros. Este conjunto de inteiros, denotado por

C = {ax + by; x, y ∈ Z},

inclui valores positivos e negativos. Além disso, escolhendo x = y = 0,


vemos que C também contém o zero.
Pelo Princı́pio da Boa Ordenação, podemos escolher x0 e y0 tais que
λ = ax0 + by0 seja o menor número inteiro positivo contido no conjunto C.
Agora mostraremos que λ | a e λ | b. Provaremos que λ | a e o outro
segue analogamente. Usaremos para este propósito o método de redução ao
absurdo, ou seja, vamos supor que λ ∤ a e obteremos uma contradição.
De λ ∤ a segue que existem inteiros q e r tais que a = λq + r com
0 < r < λ (pelo Teorema 2.7). Portanto,

r = a − λq = a − q(ax0 + by0 ) = a(1 − qx0 ) + b(−qy0 )

e assim r está no conjunto C, o que contradiz a hipótese de λ ser o menor


elemento positivo contido em C.
Uma vez que λ divide a e b só resta provar que λ = d. Com efeito, desde
que d = (a, b), podemos escrever a = da1 , b = db1 e

λ = ax0 + by0 = d(a1 x0 + b1 y0 ).

Assim d | λ. Logo pela parte (c) da Proposição 2.3, concluı́mos que d ≤ λ.


Agora d < λ é impossı́vel pois d = m.d.c(a, b), e portanto d = λ = ax0 +
by0 .
A seguinte proposição resume algumas conseqüências importantes da
demonstração dada ao Teorema de Bézout.
Proposição 2.18. Sejam d, k ∈ N e a, b, c ∈ Z. Então valem as seguintes
afirmações:
(a) Se d | a e d | b, então d | (a, b);
(b) O m.d.c.(a, b) é o menor valor positivo de ax+by, onde x e y percorrem
todos os números inteiros;

i i

i i
“Livro˙Olimpiada
i i
2006/5/27
page 41
i i

[SEC. 2.2: MÁXIMO DIVISOR COMUM E MÍNIMO MÚLTIPLO COMUM 41

(c) (ka, kb) = k(a, b);


(d) Se d | a e d | b, então ( ad , db ) = d1 (a, b). Conseqüentemente,
µ ¶
a b
, = 1;
(a, b) (a, b)

(e) Se (a, c) = (b, c) = 1, então (ab, c) = 1;


(f) Se c | ab e (b, c) = 1, então c | a.
Demonstração. A prova de (a) é conseqüência imediata da igualdade (a, b) =
ax0 + by0 anunciada no Teorema de Bézout; assim como (b) segue direta-
mente da demonstração dada a este teorema.
Para provar (c), primeiro observamos que

(ka)x + (kb)y = k(ax + by) onde x, y ∈ Z.

Usando o item (a) e o fato de k ser positivo, da igualdade acima segue que

(ka, kb) = menor valor positivo de (ka)x + (kb)y, onde x, y ∈ Z


= k · {menor valor positivo de ax + by, onde x, y ∈ Z}
= k(a, b).

A afirmação feita em (d) segue diretamente de (c), observando que


µ ¶ µ ¶
a b a b
(a, b) = d , d =d , .
d d d d

Continuamos com a prova de (e). De (a, c) = (b, c) = 1, temos que


existem inteiros xj e yj , j = 1, 2, tais que

ax1 + cy1 = 1,
bx2 + cy2 = 1.

Multiplicando lado-a-lado as igualdades obtemos

(x1 x2 ) ab + (ax1 y2 + y1 bx2 + cy1 y2 ) c = 1.


| {z } | {z }
x y

Então, usando o item (a) e a igualdade acima resulta que (ab, c) = 1.

i i

i i
“Livro˙Olimpiada
i i
2006/5/27
page 42
i i

42 [CAP. 2: DIVISIBILIDADE

Finalmente, provaremos (f). Das hipóteses temos que existem inteiros


x0 e y0 tais que
bx0 + cy0 = 1.
Multiplicamos a igualdade acima por a em ambos lados para obtermos

abx0 + acy0 = a.

Por outro lado, ab = cq para algum inteiro q. Usando esta condição na


última igualdade temos que

cqx0 + acy0 = c(qx0 + ay0 ) = a,

logo c | a.
Apesar de conhecermos propriedades teóricas do m.d.c entre dois in-
teiros, encontrá-lo de fato pode ser uma tarefa complicada, sem auxı́lio das
ferramentas corretas. Lembrando o seu significado, o leitor talvez pudesse
pensar que devemos calcular todos os divisores de a, todos os divisores de
b e descobrir qual é o maior elemento comum aos dois conjuntos.
Na prática, para achar o m.d.c se faz uso de um importante método
denominado Algoritmo de Euclides.
Teorema 2.19 (Algoritmo de Euclides). Dados dois inteiros positivos,
a e b, aplicamos sucessivamente o algoritmo da divisão para obter a seguinte
seqüência de igualdades


 b = aq1 + r1 , 0 ≤ r1 < a,



 a = r1 q2 + r2 , 0 ≤ r2 < r1 ,


r = r q + r , 0 ≤ r3 < r2 ,
1 2 3 3
(2.15)

 ··· ··· ··· ··· ···



rn−2 = rn−1 qn + rn , 0 ≤ rn < rn−1 ,



rn−1 = rn qn+1 .

O m.d.c.(a, b) = rn , ou seja, é o último resto não-nulo no processo de divisão


anterior.
Demonstração. Começamos observando que o processo de divisão (2.15)
é finito. Com efeito, a seqüência de números inteiros rk é estritamente
decrescente e está contida no conjunto {r ∈ Z, 0 ≤ r < a}, portanto não

i i

i i
“Livro˙Olimpiada
i i
2006/5/27
page 43
i i

[SEC. 2.2: MÁXIMO DIVISOR COMUM E MÍNIMO MÚLTIPLO COMUM 43

pode conter mais do que a inteiros positivos. Examinando as igualdades


(2.15) de cima para baixo e usando a Proposição 2.16 temos que

(a, b) = (a, r1 ) = (r1 , r2 ) = · · · = (rn−1 , rn ) = rn .

Observação 2.20. Notemos que o Teorema de Bézout também pode ser


obtido como conseqüência do processo de divisão (2.15). Com efeito, pode-
mos escrever
rn = rn−2 − rn−1 qn o
⇒ rn = rn−2 − (rn−3 − rn−2 qn−1 )qn .
rn−1 = rn−3 − rn−2 qn−1

Logo, conseguimos escrever rn em termos de rn−2 e rn−3 . Utilizando a


expressão rn−2 = rn−4 −rn−3 qn−2 podemos escrever rn como combinação de
rn−3 e rn−4 . Repetindo este processo várias vezes, concluimos que existem
x, y ∈ Z tais que
d = rn = xr1 + yr2 .
Ora, como r1 = b−aq1 e r2 = a−r1 q2 = a(1+q1 q2 )−bq2 , então, substituindo
estes valores na última igualdade obtemos o Teorema de Bézout.
Exemplo 2.21. Achar o máximo divisor comum dos números 471 e 1176.
Solução. Aplicando o algoritmo de Euclides obtemos a seguinte seqüência
de divisões com resto:
1176 = 471 · 2 + 234,
471 = 234 · 2 + 3,
234 = 78 · 3,

então o m.d.c(471, 1176) = 3.


2n+8
Exemplo 2.22. Provar que a fração 4n+15 é irredutı́vel para todo número
natural n.
Solução. Usando o algoritmo de Euclides temos que

4n + 15 = (2n + 8) · 1 + 2n + 7,
2n + 8 = (2n + 7) · 1 + 1,
2n + 7 = (2n + 7) · 1.

i i

i i
“Livro˙Olimpiada
i i
2006/5/27
page 44
i i

44 [CAP. 2: DIVISIBILIDADE

Então o m.d.c.(4n + 15, 2n + 8) = 1 e portanto 4n + 15 e 2n + 8 são primos


entre si para qualquer valor de n.

| ·{z
Exemplo 2.23. Achar o m.d.c.(111 · · 111}, 11
| ·{z
· · 11})
100vezes 60 vezes

Solução. Primeiro escrevemos os números na base decimal, isto é,

· · 111} = 1099 + 1098 + · · · + 1,


| ·{z
111
100 vezes
· · 11} = 1059 + 1058 + · · · + 1.
| ·{z
11
60 vezes

Aplicamos agora o algoritmo de Euclides para obter as seguintes igualdades

· · 111} = (1059 + 1058 + · · · + 1)1040 + 1039 + 1038 + · · · + 1,


| ·{z
111
100 vezes
1059 + 1058 + · · · + 1 = (1039 + 1038 + · · · + 1)1020 +
+ 1019 + 1018 + · · · + 1,
1039 + 1038 + · · · + 1 = (1019 + 1018 + · · · + 1)1020 +
+ 1019 + 1018 + · · · + 1.

Disto resulta que

| ·{z
m.d.c.(111 · · 111}, 11 · · 11}) = 1019 + 1018 + · · · + 1 = |11 ·{z
| ·{z · · 11} .
100 vezes 60 vezes 20 vezes

Agora passamos ao segundo conceito importante desta seção. O mesmo


está relacionado com os inteiros positivos que são simultaneamente múltiplos
de dois inteiros pré-fixados e é denominado Mı́nimo Múltiplo Comum.

Definição 2.24 (Mı́nimo Múltiplo Comum). Sejam a e b inteiros difer-


entes de zero. O mı́nimo múltiplo comum, resumidamente m.m.c, entre a e
b é o inteiro positivo m que satisfaz as seguintes condições:

(a) m é um múltiplo comum de a e b, isto é, a | m e b | m;

(b) m é o menor inteiro positivo com a propriedade (a).

i i

i i
“Livro˙Olimpiada
i i
2006/5/27
page 45
i i

[SEC. 2.2: MÁXIMO DIVISOR COMUM E MÍNIMO MÚLTIPLO COMUM 45

Neste caso, denotamos o m.m.c entre a e b por m = m.m.c(a, b) ou por


m = [a, b].
Resumimos a seguir algumas das propriedades fundamentais do m.m.c
de dois inteiros.
Proposição 2.25. Sejam a, b, c ∈ Z e k ∈ Z. Então valem as seguintes
afirmações:
(a) Se c é múltiplo comum de a e b, então [a, b] | c;
(b) [ka, kb] = k[a, b];
(c) |ab| = [a, b] · (a, b).
Demonstração. Começamos com a prova de (a). A divisão com resto de c
por [a, b] nos dá
c = [a, b]q + r, 0 ≤ r < [a, b]. (2.16)
Da igualdade acima, basta provar que r = 0 para obter o resultado desejado.
Suponhamos, pelo contrário, que 0 < r < [a, b]. Notemos que tanto a como
b dividem c e [a, b]. Logo, pelo item (b) da Proposição 2.3 e a igualdade
(2.16), temos que a e b também dividem r, ou seja, r é múltiplo comum de
a e b e não pode ser menor que [a, b], contradizendo nossa suposição.
Prosseguimos com a prova de (b). Observemos que k[a, b] é múltiplo
comum de ka e kb, logo pelo item (i) vale que
[ka, kb] ≤ k[a, b]. (2.17)
Por outro lado, [ka, kb] = q1 ka = q2 kb, para alguns inteiros q1 e q2 ; logo,
[ka,kb]
k é um múltiplo comum de a e b. Portanto,
[ka, kb]
[a, b] ≤ ⇔ k[a, b] ≤ [ka, kb]. (2.18)
k
Das igualdades (2.17) e (2.18) segue que
k[a, b] ≤ [ka, kb] ≤ k[a, b],
de onde vem diretamente o resultado.
Para provar (c) podemos supor sem perda de generalidade que a e b são
positivos devido às igualdades
[a, b] = [a, −b] = [−a, b] = [−a, −b].

i i

i i
“Livro˙Olimpiada
i i
2006/5/27
page 46
i i

46 [CAP. 2: DIVISIBILIDADE

Dividiremos a prova em dois casos:

Caso 1: (a, b) = 1.
Sabemos que b | [a, b] e [a, b] = qa, para algum q ∈ N. Então b | qa e
além disso (a, b) = 1. Logo, pelo item (v) da Proposição 2.18 temos que
b | q. Portanto, b ≤ q e conseqüentemente
ab ≤ aq = [a, b]. (2.19)
Entretanto, da definição de [a, b] vale que
[a, b] ≤ ab. (2.20)
Das desigualdades (2.19) e (2.20) segue que ab ≤ [a, b] ≤ ab. Assim, ab =
[a, b] = [a, b] · 1 = [a, b] · (a, b).

Caso 2: (a, b) > 1. ³ ´


a b
Da parte (c) da Proposição 2.18 sabemos que (a,b) , (a,b) = 1. Apli-
cando o caso anterior vale que
· ¸ µ ¶
a b a b a b
· = , · , .
(a, b) (a, b) (a, b) (a, b) (a, b) (a, b)
Multiplicamos esta última igualdade por (a, b)2 e usamos o item (b) provado
anteriormente, assim como a parte (d) da Proposição 2.18 para obter
· ¸ µ ¶
a b a b
ab = (a, b) , (a, b) , = [a, b] · (a, b).
(a, b) (a, b) (a, b) (a, b)

Exemplo 2.26. Dois amigos passeiam de bicicleta, na mesma direção, em


torno a uma pista circular. Para dar uma volta completa um deles demora
15 minutos e o outro demora 18 minutos. Eles partem juntos e combinam
interromper o passeio quando os dois se encontrarem pela primeira vez no
ponto de partida. Quantas voltas deu cada um?
Solução. Denotemos por N1 e N2 , respectivamente, o número de voltas que
dá cada um dos amigos. Notemos que o tempo total da corrida é o menor
valor positivo de T que satisfaz as igualdades
T = 15N1 = 18N2 ,

i i

i i
“Livro˙Olimpiada
i i
2006/5/27
page 47
i i

[SEC. 2.2: MÁXIMO DIVISOR COMUM E MÍNIMO MÚLTIPLO COMUM 47

ou seja
15 · 18
T = [15, 18] = = 90.
3
Portanto, N1 = 6 e N2 = 5.

Finalizamos esta seção com um exemplo que nos fornece uma bela in-
terpretação geométrica do mı́nimo múltiplo comum. O mesmo foi proposto
na Olimpı́ada Brasileira de Matemática.

Exemplo 2.27. Um retângulo de lados inteiros AB = m e CD = n, é


dividido em quadrados de lado 1. Em cada um dos vértices ele possui um
pequeno orifı́co. Um raio de luz entra no retângulo por um dos vértices, na
direção da bissetriz do ângulo reto, e é refletido sucessivamente nos lados
do retângulo. Quantos quadrados são atravessados pelo raio de luz?

Solução. Se fizermos alguns testes preliminares dando valores a m e n, ver-


emos que em cada caso a resposta coincidirá com o m.m.c.(m,n). Provemos
que isto de fato vale para m e n quaisquer. Para realizar a prova nos aux-
iliaremos da seguinte figura

D C

A B
Figura 2.1::

Primeiramente, notemos que cada vez que o raio de luz atravessa um


quadrado ele avança uma unidade tanto na direção horizontal como na
direção vertical. Usando este fato fazemos as seguintes observações

• Se o raio entra pelo vértice A, terá que atravessar m quadrados até


chegar ao lado BC, imediatamente mais m para chegar ao lado AD,

i i

i i
“Livro˙Olimpiada
i i
2006/5/27
page 48
i i

48 [CAP. 2: DIVISIBILIDADE

depois mais m para chegar novamente ao lado BC, e assim sucessi-


vamente. Além disto, depois do raio percorrer pm quadrados, com
p ∈ N, estará batendo no lado BC ou no lado AD.

• Analogamente o raio baterá no lado AB ou no lado DC se, e somente


se, atravessar qn quadrados, com q ∈ N.

• Somente nos vértices B, C e D do retângulo pode acontecer que o raio


incidente saia do retângulo, terminando assim o processo de reflexão.

Usando as observações acima é fácil ver que o raio chegará a um vértice


quando chegar simultaneamente a dois lados perpendiculares do retângulo.
Portanto, deve ter atravessado um número x de quadrados tal que x =
pm = qn, ou seja, x deverá ser um múltiplo comum de m e n. É claro que a
primeira vez que o raio chega a um vértice o número x é o menor múltiplo
comum de m e n, isto é, x = [m, n].
Finalmente, observamos que nenhum dos quadrados é atravessado duas
vezes no percurso do raio de A até bater no primeiro vértice, pois como
vemos na figura numa das direções os quadrados atravessados serão todos
cinzas e na outra direção, serão todos brancos.

2.3 Números Primos e Compostos


Ao longo da História da matemática, os números primos foram protago-
nistas de célebres problemas que motivaram o desenvolvimento de teorias
e técnicas pelas mentes mais férteis, como Fermat, Euler e Gauss. Até
hoje muitos destes problemas, simples de enunciar, que envolvem números
primos são desafios intelectuais para toda a humanidade.
Esta seção será dedicada ao estudo de propriedades básicas dos números
primos.
O número 1 tem um único divisor, precisamente ele mesmo. Isto faz
dele um número bem peculiar dentro dos números naturais. Todo número
natural N maior do que 1 tem pelo menos 2 divisores, claramente 1 e N .
Isto motiva a seguinte definição.

Definição 2.28 (Números Primos e Compostos). Um inteiro positivo


N é dito primo se os únicos divisores que ele tem são 1 e ele próprio; caso
contrário, é dito composto.

i i

i i
“Livro˙Olimpiada
i i
2006/5/27
page 49
i i

[SEC. 2.3: NÚMEROS PRIMOS E COMPOSTOS 49

Exemplo 2.29. Os números 1, 3, 5, 7, e 11 são primos e os números 10, 15, 35,


e 348 são compostos.
Proposição 2.30. Seja N > 1 um número inteiro. Então
(a) O menor divisor de N diferente de 1 é um número primo;
(b) Se
√ N é composto, o seu menor divisor diferente de 1 não é maior que
N;
Demonstração. Começamos provando (a). Seja p o menor divisor de N ,
diferente de 1. Se p fosse composto teria algum divisor q tal que 1 < q < p;
mas
q | p e p | N,
o que nos diz que q | N , e isto contradiz a hipótese levantada sobre p.
Para provar (b) denotamos por p o menor divisor de N , diferente de 1.
Portanto, N = pq com q ≥ p. Multiplicando ambos lados da desigualdade
por p obtemos
N = pq ≥ p2 ,

e conseqüentemente vale N ≥ p.
Agora vamos enunciar um dos resultados mais clássicos da matemática,
que garante a existência de infinitos números primos. Até onde se conhece,
a demonstração a seguir foi a primeira demonstração escrita utilizando o
método de redução ao absurdo, e é devida a Euclides (300 a.C.).
Teorema 2.31 (Teorema de Euclides). A quantidade de números pri-
mos é infinita.
Demonstração. Faremos a prova por redução ao absurdo. Suponha que
existe uma quantidade finita de números primos e denotemos estes por
p1 , p2 , p3 , . . . , pk .
Consideremos o número
N = p1 p2 p3 · · · pk + 1
e chamemos de q o seu menor divisor primo. Obviamente q não coincide
com nenhum dos números pi , 1 ≤ i ≤ k, pois caso contrário, como ele divide
N , teria que dividir 1, o que é impossı́vel. Logo, temos uma contradição à
hipótese de termos uma quantidade finita de primos.

i i

i i
“Livro˙Olimpiada
i i
2006/5/27
page 50
i i

50 [CAP. 2: DIVISIBILIDADE

Os números primos também podem ser caracterizados da seguinte maneira:


Proposição 2.32. Um inteiro positivo p é primo se, e somente se, satisfaz
a seguinte propriedade:
p | ab =⇒ p | a ou p|b (2.21)
onde a, b ∈ Z.
Demonstração. Primeiramente, suponhamos que p é primo e que p ∤ b, logo
(p, b) = 1. Então, pelo item (f) da Proposição 2.18 temos que p | a.
Reciprocamente, suponhamos que, a propriedade 2.21 é válida e além
disto vamos supor, pelo absurdo, que p não é primo. Então,
p = d1 d2 , com 1 < d1 < p, 1 < d2 < p. (2.22)
De (2.21) segue que p | d1 ou p | d2 ; conseqüentemente
p ≤ d1 , ou p ≤ d2 , (2.23)
contradizendo isto o afirmado em (2.22).

2.3.1 O Crivo de Eratóstenes


Apesar de sabermos que existem infinitos números primos, determinar se
um número é primo nem sempre é uma tarefa rápida. Uma das princi-
pais ferramentas utilizadas para esse fim é um método simples, denominado
Crivo de Eratóstenes, que nos permite achar todos os números primos que
não são maiores que um inteiro N dado. O método consiste nos seguintes
passos: Escrevemos os números de forma ordenada a partir de 2, isto é,
2, 3, 4, 5, 6, 7, 8, 9, 10, 11, 12, 13, 14, 15, 16, 17, . . . , N (2.24)
• Observamos que o primeiro primo que aparece em (2.24) é 2 e ime-
diatamente apagamos da lista (2.24) todos os múltiplos de 2 maiores
que ele, por serem compostos; resta assim a seguinte lista
2, 3, 5, 7, 9, 11, 13, 15, 17 . . .

• O primeiro número não apagado que aparece na lista restante é 3, que


também é primo. Imediatamente apagamos da lista todos os múltiplos
de 3 maiores que ele, por serem compostos; resta agora a lista
2, 3, 5, 7, 11, 13, 17, . . .

i i

i i
“Livro˙Olimpiada
i i
2006/5/27
page 51
i i

[SEC. 2.3: NÚMEROS PRIMOS E COMPOSTOS 51

• O primeiro número não apagado que aparece na lista que restou do


passo anterior é 5, que também é primo. Imediatamente apagamos da
lista todos os múltiplos de 5 maiores que ele, por serem compostos.

Se repetirmos este processo até o maior número menor que N , os
números que sobram são exatamente os números primos.

Por exemplo, se N = 40, temos que 40 = 6, 324555 . Então, aplicando
o método:

2 3 4 5 6 7 8 9 10
11 12 13 14 15 16 17 18 19 20
21 22 23 24 25 26 27 28 29 30
31 32 33 34 35 36 37 38 39 40

Passo 1: Ordenamos os números

2 3 5 7 9
11 13 15 17 19
21 23 25 27 29
31 33 35 37 39

Passo 2: Tiramos os múltiplos de 2

2 3 5 7
11 13 17 19
23 25 29
31 35 37

Passo 3: Tiramos os múltiplos de 3

2 3 5 7
11 13 17 19
23 29
31 37

Passo 4: Tiramos os múltiplos de 5

Como 72 = 49 > 40, paramos agora.

i i

i i
“Livro˙Olimpiada
i i
2006/5/27
page 52
i i

52 [CAP. 2: DIVISIBILIDADE

Note que ao começar a apagar os múltiplos de um número primo p


podemos começar a apagar a partir de p2 , pois se supomos que existe um
número composto N1 não apagado menor que p2 , temos que N1 = p1 q1 ,
sendo √
p1 seu menor divisor primo. Então, pelo item (b) da Proposição 2.30,
p1 < N1 < p, logo N1 deveria ter sido apagado pois é múltiplo de um
primo menor que p.

2 3 5 7 11 13 17 19 23 29
31 37 41 43 47 53 59 61 67 71
73 79 83 89 97 101 103 107 109 113
127 131 137 139 149 151 157 163 167 173
179 181 191 193 197 199 211 223 227 229
233 239 241 251 257 263 269 271 277 281
283 293 307 311 313 317 331 337 347 349
353 359 367 373 379 383 389 397 401 409
419 421 431 433 439 443 449 457 461 463
467 479 487 491 499 503 509 521 523 541

Tabela 2.1: Os primeiros 100 números primos

Os números primos além de belos e desafiadores do ponto de vista


matemático, são extremamente importantes para as atividades usuais de
nosso dia-a-dia. Por exemplo, nenhuma transação bancária ou pela inter-
net estaria segura sem o uso de números primos muito grandes. Assim,
surge naturalmente a pergunta de como podemos produzı́-los em grandes
quantidades. Essa pergunta sempre intrigou os matemáticos e continua sem
solução até os dias atuais. Apesar deles serem abundantes, em quantidade
infinita de acordo com o Teorema 2.31, não existe nenhum método razoável
de produção de números primos, mesmo tendo em mãos a alta tecnologia
de hoje em dia. Porém, ao longo do tempo algumas fórmulas se mostraram
úteis para a descoberta de números primos. Entre estas fórmulas, destaca-se
o que chamamos de primos de Mersenne.
Marin Mersenne (1588-1648) foi um monge francês que nasceu na cidade
de Maine e foi um dos grandes influenciadores da matemática francesa nos
séculos XVI e XVII. Apaixonado pelos números, teve entre seus correspon-
dentes Descartes, Fermat, Pascal e Galileu. Entre suas várias descobertas,

i i

i i
“Livro˙Olimpiada
i i
2006/5/27
page 53
i i

[SEC. 2.3: NÚMEROS PRIMOS E COMPOSTOS 53

ele estudou os números da forma:

Mn = 2n − 1

Observe que vale o seguinte fato a respeito desses números:


Proposição 2.33. Se Mn é primo, então n é primo.
Demonstração. Provar essa proposição equivale a mostrar que a sua forma
contra-recı́proca vale. Ou seja, que se n é composto, digamos n = a.b, com
a ≥ b > 1, então Mn também é composto. De fato, podemos decompô-lo
do seguinte modo:
¡ ¢¡ ¢
Ma.b = 2ab − 1 = 2a(b−1) − 2a(b−2) + · · · + 2a + 1 2b − 1 .

Porém, não é verdade a recı́proca da afirmação acima. Por exemplo,


Hudalricus Regius mostrou em 1536 que M11 = 211 − 1 = 2047 não é primo,
já que 2047 = 23 · 89.
Em 1643, Mersenne afirmou que para

n = 2, 3, 5, 7, 13, 17, 19, 31, 67, 127 e 257,

os valores de Mn são todos primos e para todos os outros valores de n


menores que 257, Mn é composto.
Hoje sabemos que Mersenne errou na sua afirmação, esquecendo três
valores de n onde Mn é primo: 61, 89 e 107 e incluindo M67 e M257 como
números primos. Esses fatos só foram mostrados em 1947.
O maior número primo conhecido até 15 de Dezembro de 2005 é M30402457
que possui 9.152.052 de dı́gitos! Para descobrir esse número, o grupo de
pesquisa liderado por Cooper, Boone, Woltman e Kurowski (GIMPS) pre-
cisou do auxı́lio de mais de 700 computadores! Para mais informações veja
a página: www.mersenne.org.

2.3.2 O Teorema Fundamental da Aritmética


Os números primos são as células dos números naturais, no sentido de que
qualquer número natural é produto de números primos. Por exemplo,

560 = 56 · 10 = 7 · 8 · 5 · 2 = 7 · 2 · 2 · 2 · 5 · 2,

i i

i i
“Livro˙Olimpiada
i i
2006/5/27
page 54
i i

54 [CAP. 2: DIVISIBILIDADE

onde cada um dos fatores que aparecem no produto são números primos.
Perguntamo-nos, o que acontece se começamos com uma outra fatoração
inicial de 560, por exemplo, 560 = 28 · 20. Vejamos:
560 = 28 · 20 = 14 · 2 · 10 · 2 = 7 · 2 · 2 · 5 · 2 · 2.
Surpreendentemente chegamos à mesma representação anterior, salvo a or-
dem dos fatores.

2 7
5
2

Figura 2.2: O número 560 é composto de 4 células do tipo 2 , uma célula do tipo
7 e uma célula do tipo 5.

O fato observado acima vale para qualquer número natural maior que
1. Especificamente, temos o seguinte resultado conhecido como Teorema
Fundamental da Aritmética.
Teorema 2.34 (Teorema Fundamental da Aritmética). Todo número
natural N maior que 1 pode ser escrito como um produto
N = pα1 α2 α3 αm
1 · p2 · p3 · · · pm , (2.25)
onde m ≥ 1 é um número natural, αi ∈ N e pi é primo para todo 1 ≤ i ≤ m
. Além disso, a fatoração em (2.25) é única se exigirmos que p1 < p2 <
· · · < pm .
Demonstração. Seja N um inteiro maior que 1. Denotando por p1 seu menor
divisor primo tem-se que
N = p1 β1 , 1 ≤ β1 < N.
Se β1 = 1, então N1 = p1 e a fatoração desejada é obtida. Caso contrário,
denotando por p2 o menor divisor primo de β1 tem-se que
N = p1 p2 β2 , 1 ≤ β2 < β1 .

i i

i i
“Livro˙Olimpiada
i i
2006/5/27
page 55
i i

[SEC. 2.3: NÚMEROS PRIMOS E COMPOSTOS 55

Se β2 = 1, então N = p1 p2 e novamente chegamos à fatoração desejada.


Caso contrário, denotando por p3 o menor divisor primo de β2 tem-se que
N = p1 p2 p3 β3 , 1 ≤ β3 < β2 .
Continuando este processo sucessivamente obtemos então uma seqüência
estritamente decrescente de números naturais αn , ou seja,
N > β1 > β2 > β2 > · · · > βn > βn+1 > · · · ≥ 1,
Então, pelo princı́pio da boa ordem, só pode existir uma quantidade finita
de indices n tais que βn > 1 e conseqüentemente βn+1 = 1, de onde segue
que
N = p1 p2 · · · pn .
Notemos que na representação acima os pi podem-se repetir, resultando
finalmente a representação desejada em (2.25).
Provaremos agora a unicidade de tal fatoração. Com efeito, suponha
que existem duas fatorações:
β1 β2 β3
pα α2 α3 αm βs
1 · p2 · p3 · · · pm = N = q1 · q2 · q3 · · · qs
1

Pela Proposição 2.32 temos que cada pi divide algum qj , logo pi = qj ,


por serem primos. Portanto, cada pi aparece no lado direito da igualdade
acima, e, um argumento análogo nos dá que cada qj também aparece no
lado esquerdo da igualdade. Então, como os pi s e os qj s são diferentes dois
a dois e organizados crescentemente, temos m = s e a igualdade se reduz a
β1 β2 β3
pα α2 α3 αm βm
1 · p2 · p3 · · · pm = p1 · p2 · p3 · · · pm .
1

Suponhamos agora que α1 seja diferente de β1 ; sem perda de generali-


dade vamos supor que α1 < β1 . Portanto,
β1 −α1
pα α3 αm
2 · p3 · · · pm = p1
2
· pβ2 2 · pβ3 3 · · · pβmm ,
e como β1 − α1 > 0 então, pela Proposição 2.32 temos que p1 divide algum
pj , com j > 1, o que é impossı́vel. Portanto, α1 = β1 . Similarmente
provamos que αi = βi , com i = 1, · · · , n.
Observação 2.35. O Teorema Fundamental da Aritmética foi enunciado
precisamente por Gauss (1777-1855). Seus antecessores, Fermat, Euler, La-
grange e Legendre, utilizavam este teorema sem a preocupação de tê-lo
enunciado ou demonstrado com precissão.

i i

i i
“Livro˙Olimpiada
i i
2006/5/27
page 56
i i

56 [CAP. 2: DIVISIBILIDADE

Uma prova alternativa deste teorema será apresentada no Capı́tulo 5,


usando o método de indução.
Exemplo 2.36. Prove que um número N é par se, e somente se, o número
2 aparece na fatoração de N em fatores primos.
Solução. Obviamente, se 2 aparece na fatoração em primos de N , então N
é par. Ora, se N é par temos que N = 2q. Por outro lado q e N se fatoram,
respeitivamente, como
q = q1α1 q2α2 · · · qm
αm
e N = pβ1 1 pβ2 2 · · · pβs s .
Logo,
2 · q1α1 q2α2 · · · qm
αm
= pβ1 1 pβ2 2 · · · pβs s .
Pela unicidade da fatoração, para algum i, com 1 ≤ i ≤ s, o correspondente
pi deve ser igual a 2. Portanto, 2 aparece na fatoração de n.
Exemplo 2.37. Seja A = {1, 2, 3, 4, 5, 6, 7}. É possı́vel decompor o con-
junto A em dois subconjuntos disjuntos tais que o produto dos elementos
de um seja igual ao produto dos elementos do outro?
Solução. Mostraremos que é impossı́vel fazer esta decomposição. Com
efeito, suponha que existem tais conjuntos, A1 = {p1 , p2 , · · · , pr } e A2 =
{q1 , q2 , · · · , qs }. Então

p1 p2 · · · pr = q1 q2 · · · qs
| {z } | {z }
α β

e além disso, como os conjuntos A1 A2 são disjuntos, temos que o número 5


aparece no produto α ou no produto β, mas não em ambos simultaneamente.
Por outro lado, o Teorema 2.34 nos diz que a fatoração em primos de α é
igual à fatoração em primos de β, logo o número 5 deveria aparecer tanto no
produto α como no produto β, contradizendo isto o fato anterior. Portanto
não existe uma decomposição com as condições exigidas.
Exemplo 2.38. Encontre todos os números inteiros e positivos n com a
propriedade de que o conjunto
A = {n, n + 1, n + 2, n + 3, n + 4, n + 5}
pode ser particionado em dois subconjuntos tais que o produto dos elementos
de um dos subconjuntos seja igual ao produto dos elementos do outro.

i i

i i
“Livro˙Olimpiada
i i
2006/5/27
page 57
i i

[SEC. 2.3: NÚMEROS PRIMOS E COMPOSTOS 57

Demonstração. Digamos que seja possı́vel essa decomposição para algum n


e vamos denotar os conjuntos que obtemos com a decomposição por A1 e
A2 . Observando a decomposição dos elementos dos subconjuntos em fatores
primos, temos que todo fator primo de A1 também deverá pertencer a A2 .
No conjunto dos seis números só podemos ter um múltiplo de 7, por isso
não podemos tomar n como múltiplo deste primo. Analogamente para
primos maiores que 7. Analisando o primo 5, concluı́mos que n e n + 5 são
múltiplos de 5, pois se não, cairı́amos na análise anterior. Assim, os números
n + 1, n + 2, n + 3 e n + 4 são da forma 2α 3β . Como entre eles existem dois
ı́mpares, logo teremos duas potências de 3 cuja diferença é 2, um absurdo.
Assim, não existe n que satisfaz as condições do enunciado.

Finalizamos esta seção com um exemplo que mostra como podemos com-
binar os fatos estudados para resolver problemas mais difı́ceis

Exemplo 2.39. Encontre todos os números que são formados por 4 algar-
ismos da forma aabb e que sejam quadrados perfeitos.

Solução. Como o número aabb é um quadrado perfeito, significa que:

N 2 =aabb ;
¡ ¢
N 2 =103 a + 102 a + 10b + b = 103 + 102 · a + (10 + 1) · b ;
N 2 =1100 · a + 11 · b ;
¡ ¢ ¡ ¢
N 2 =11 100a + b = 11 99a + a + b ;

Como 11 é primo é fácil ver, usando a Proposição 2.32, que 112 | N 2 .


Segue-se então que 11 | (99a + a + b). Portanto, 11 | (a + b). Como
aabb tem 4 algarismos, segue-se que a 6= 0; portanto a ∈ {1, 2, 3, · · · , 9} e
b ∈ {0, 1, 2, · · · , 9}. De onde a + b ≤ 18. Logo, necessariamente devemos
ter a + b = 11. Podemos observar que a 6= 1, pois se a = 1 então b = 10.
Analogamente, b 6= 0, 1. Portanto,

a ∈ {2, 3, 4, · · · , 9} e b ∈ {2, 3, 4, · · · , 9}.

Como em todo número quadrado perfeito o algarismo das unidades somente


pode acabar em 0, 1, 4, 5, 6 e 9. Segue-se que

b ∈ {4, 5, 6, 9}.

i i

i i
“Livro˙Olimpiada
i i
2006/5/27
page 58
i i

58 [CAP. 2: DIVISIBILIDADE

Certamente b 6= 5, pois todo número que acaba em 5 quando é elevado ao


quadrado sempre acaba em 25. Assim,
b ∈ {4, 6, 9}.
• Se b = 4, então a = 7. Neste caso o número seria 7744 que é um
quadrado perfeito;
• Se b = 6, então a = 5. Neste caso o número seria 5566 que não é um
quadrado perfeito;
• Se b = 9, então a = 2. Neste caso o número seria 2299 que não é um
quadrado perfeito.
Finalmente, a única solução possı́vel é aabb = 7744 = 882 .

2.3.3 Identificando Números Compostos


Não existe uma regra geral que nos permita determinar imediatamente
se um número inteiro é primo ou composto. Entretanto, existem alguns
critérios, baseados fundamentalmente em identidades algébricas, que nos
ajudam nessa difı́cil tarefa.

Critério 1. Todo número que é diferença de quadrados de dois inteiros


positivos, com diferença distinta de um, é composto.
Demonstração. Com efeito, se N = a2 − b2 , então N = a2 − b2 = (a − b)(a +
b). Logo, como a − b =
6 1, temos que N é composto.
Exemplo 2.40. Prove que o número N = 220 − 254 é composto.
Solução. Escrevemos N de outra forma, com o objetivo de facilitar nosso
trabalho. Com efeito, observemos que
N = (210 )2 − (252 )2 = 10242 − 6252 ,
logo é composto por ser diferença de quadrados. Além disso,
N = 10242 − 6252 ,
= (1024 − 625)(1024 + 625),
(2.26)
= 399 · 1649,
= 3 · 133 · 1649.

i i

i i
“Livro˙Olimpiada
i i
2006/5/27
page 59
i i

[SEC. 2.3: NÚMEROS PRIMOS E COMPOSTOS 59

Portanto, podemos concluir que 3 | N .


Notemos que neste exemplo não é muito prático aplicar o critério de
divisibilidade, estudado na Proposição 2.5, para ver se o mesmo é divisı́vel
por 3.

O segundo critério é uma generalização do primeiro.

Critério 2. Sejam a, b inteiros positivos, com a − b ≥ 2, e n ∈ N, com


n ≥ 2. Então a − b divide an − bn e, portanto, an − bn é composto.

Demonstração. Os casos n = 2 e n = 3 seguem das clássicas fatorações:

• a2 − b2 = (a − b)(a + b), [diferença de quadrados]

• a3 − b3 = (a − b)(a2 + ab + b2 ), [diferença de cubos].

Estas igualdades sugerem a seguinte fatoração para o caso geral

¡ ¢
an − bn = (a − b) an−1 + an−2 b + an−3 b2 + · · · + abn−2 + bn−1 ,

de onde segue diretamente o resultado desejado. Para provar esta igualdade,


denotamos por β o lado direito, ou seja,
¡ ¢
β = (a − b) an−1 + an−2 b + an−3 b2 + · · · + abn−2 + bn−1

e desenvolvemos o mesmo para obtermos


¡ ¢
β = an + an−1 b + an−2 b2 + · · · + a2 bn−2 + abn−1
¡ ¢
− bn − an−1 b + an−2 b2 + · · · + a2 bn−2 + abn−1
= an − bn ,

terminando assim a prova.

Exemplo 2.41. Prove que N = (999994)1234567890 − 1 é divisı́vel por


333331.

Solução. Escolhendo a = 999994, b = 1 e n = 1234567890 pelo critério 2


temos que
a − 1 = 999993 | N. (2.27)

i i

i i
“Livro˙Olimpiada
i i
2006/5/27
page 60
i i

60 [CAP. 2: DIVISIBILIDADE

Além disso, 999993 = 3 · 333331; portanto

333331 | 999993. (2.28)

Então, usando o item (a) da Proposição 2.3 e as afirmações feitas em (2.27)


e (2.28), temos que 333331 | N .

Outro critério muito valioso é a Identidade de Sophie Germain, devido à


matemática francesa Sophie Germain(1776-1831), que enunciamos a seguir.

Critério 3. (Identidade de Sophie Germain). Dados a, b ∈ R, vale a


igualdade
a4 + 4b4 = (a2 + 2b2 + 2ab)(a2 + 2b2 − 2ab).

Demonstração. A prova segue das seguintes igualdades:

a4 + 4b4 = a4 + 4a2 b2 + 4b4 − 4a2 b2


= (a2 + 2b2 )2 − 4a2 b2
= (a2 + 2b2 + 2ab)(a2 + 2b2 − 2ab).

Como aplicação desta identidade vejamos os seguintes exemplos.

Exemplo 2.42. Mostre que para qualquer inteiro positivo n > 1 o número
qn = n4 + 4n nunca é primo.

Solução. O conjunto dos números naturais é particionado em duas classes


disjuntas: o conjunto dos números pares e o conjunto dos números ı́mpares.
Estudaremos cada classe por separado. Assim,

• se n é um número par, então n = 2k para algum inteiro positivo k ≥ 1.


Deste modo,

n4 + 4n = (2k)4 + 42k = 16k 4 + 24k ,


¡ ¢
= 2 8k 4 + 24k−1 .

Portanto, neste caso, n4 + 4n é um número par maior do que 2. Logo,


se n > 1 é qualquer número inteiro positivo par temos que n4 + 4n
não é um número primo.

i i

i i
“Livro˙Olimpiada
i i
2006/5/27
page 61
i i

[SEC. 2.3: NÚMEROS PRIMOS E COMPOSTOS 61

• se n é um número ı́mpar, então n = 2k + 1 para algum inteiro positivo


k ≥ 1. Assim,
n4 + 4n = (2k + 1)4 + 42k+1 = (2k + 1)4 + 4 · 42k
4 ¡ ¢4
= (2k + 1)4 + 4 · 24k = (2k + 1) + 4 · 2k .
Logo, tomando a = 2k + 1 e b = 2k , o resultado é uma conseqüência
direta da identidade de Sophie Germain.

Exemplo 2.43. O número N = 42005 + 20054 é primo?


Solução. Notemos que
¡ ¢4
42005 + 20054 = 4 · 42004 + 20054 = 4 · 4501 + 20054 .
Na Identidade de Sophie Germain considerando a = 2005 e b = 4501 , temos
que

N = (20052 + 2 · 41002 + 2 · 2005 · 4501 ) · (20052 + 2 · 41002 − 2 · 2005 · 4501 ).


Logo, o número 42005 + 20054 não é primo.

Também podemos resolver problemas deste tipo de várias formas.


Exemplo 2.44. Mostre que o número 520 + 230 é composto.
Solução 1. Escrevemos
¡ ¢4 ¡ ¢4
520 + 230 = 55·4 + 22 · 228 = 55 + 4 · 27 ,
de onde podemos usar a Identidade de Sophie Germain com a = 55 e b = 27
para comprovar que o número 520 + 230 é composto.
Solução 2. As seguintes igualdades são válidas:
¡ ¢2 ¡ ¢2
520 + 230 = 510 + 215
¡ ¢2 ¡ ¢2
= 510 + 2 · 510 · 215 + 215 − 2 · 510 · 215
¡ ¢2
= 510 + 215 − 2 · 510 · 215
¡ ¢2
= 510 + 215 − ·510 · 216
¡ ¢¡ ¢
= 510 + 215 + 55 · 28 510 + 215 − 55 · 28 .

i i

i i
“Livro˙Olimpiada
i i
2006/5/27
page 62
i i

62 [CAP. 2: DIVISIBILIDADE

A última identidade mostra que o número 520 + 230 é composto.

Finalizamos esta seção com mais alguns exemplos interessantes.


5125 −1
Exemplo 2.45. Mostre que o inteiro positivo 525 −1 é composto.

Solução. Sabemos que x5 − 1 = (x − 1)(x4 + x3 + x2 + x + 1), de onde

x5 − 1
= x4 + x3 + x2 + x + 1 = (x2 + 3x + 1)2 − 5x(x + 1)2 .
x−1

Tomando x = 525 obtemos

5125 − 1 ¡ 50 ¢2 ¡ ¢2
25
= 5 + 3 · 525 + 1 − 5 · 525 525 + 1 ,
5 −1
¡ ¢2 ¡ ¢2
= 550 + 3 · 525 + 1 − 526 525 + 1 ,
© ª2 © ¡ ¢ª2
= 550 + 3 · 525 + 1 − 513 525 + 1 ,

que é composto por ser uma diferença de quadrados.

Exemplo 2.46. Demonstre que o número 1 000


| {z. . . 00} 1 é composto.
2006 zeros

Solução. Observemos que

1 000
| {z. . . 00} 1 =102007 + 1;
2006 zeros
¡ ¢3
= 10669 + 1;

da identidade a3 + 1 = (a + 1)(a2 − a + 1) segue-se que:


¡ 669 ¢ ¡ 1338 669
¢
1 000 .
| {z }. . 00 1 = 10 + 1 10 − 10 + 1 .
2006 zeros

Esta última igualdade mostra que o número 1 |000 {z


. . . 00} 1 é composto.
2006 zeros

i i

i i
“Livro˙Olimpiada
i i
2006/5/27
page 63
i i

[SEC. 2.4: UM POUCO SOBRE EQUAÇÕES DIOFANTINAS 63

x y

Figura 2.3::

2.4 Um pouco sobre Equações Diofantinas


Começamos esta seção com uma brincadeira interessante.
João, ao sair da aula de matemática do professor Peitágoras, encontrou
Pedro e lhe propôs a seguinte brincadeira:
- Pense numa peça de dominó, Pedro. Vou adivinhar que peça é essa
usando uma fórmula mágica.
- Ok, João. Pode começar, já pensei.
- Escolha um dos números na peça e multiplique por 5. Depois disso
some três a esse resultado. Multiplique agora o número que você obteve por
dois. Some isto com o outro número da peça. Qual foi o resultado?
- Foi 40.
- Então a peça que você escolheu foi a 3 com 4!
- Como você acertou? Me ensina!
Claro que de mágico João não tinha nada e decidiu contar seu segredo
a Pedro.
O jogo funciona assim: cada parte da peça de dominó pode ser consider-
ada como um dos dı́gitos de um número de 2 algarismos, o qual denotamos
por N = xy = 10x + y (veja a Figura 2.3). Acompanhando os passos de
João, temos que:

(5x + 3)2 + y = 40 ⇔ 10x + y = 34, (2.29)

que claramente, tem por soluções: x = 3 e y = 4, usando a representação


de 34 na base decimal.

A equação (2.29) é um caso particular da seguinte forma mais geral:

ax + by = c, (2.30)

i i

i i
“Livro˙Olimpiada
i i
2006/5/27
page 64
i i

64 [CAP. 2: DIVISIBILIDADE

onde a, b, c ∈ Z, com a 6= 0 e b 6= 0.
A equação (2.30) é chamada de equação diofantina linear e uma solução
desta é qualquer par de inteiros (x, y) que satisfaçam (2.30). É conhecido
que todos os pontos do plano, com coordenadas (x, y), que satisfazem a
igualdade (2.30) representam, geometricamente, uma reta. Logo, as soluções
de uma equação diofantina linear são os pontos de coordenadas inteiras
(também conhecidos como pontos latices) que estão dispostos sobre a reta
que esta representa. Por exemplo, os pontos (−1, −2) e (1, 1) são soluções
da equação diofantina 3x − 2y = 1, veja a Figura 2.4.
3
y
2
L

1

0
x
-1

-2

-3
-3 -2 -1 0 1 2 3

Figura 2.4: A equação da reta L é 3x − 2y = 1.

Naturalmente nos perguntamos: É sempre possı́vel achar soluções para


uma equação diofantina linear ? A resposta é não; o próximo resultado nos
diz quando isto é possı́vel. Além disso, se uma equação diofantina linear
tem uma solução na verdade ela tem uma infinidade de soluções.
Proposição 2.47. A equação diofantina linear

ax + by = c, a, b, c ∈ Z, com a 6= 0 e b 6= 0, (2.31)

tem solução se, e somente se, d | c, onde d = (a, b). Além disso, se (x0 , y0 )
é uma solução, então o conjunto de soluções de (2.31) é constituı́do por
todos os pares de inteiros (x, y) da forma:

x = x0 + t db e y = y0 − t ad , t ∈ Z. (2.32)

Demonstração. Primeiramente suponhamos que (x0 , y0 ) é uma solução de


(2.31), logo ax0 + by0 = c. Usando que d = (a, b) sabemos que existem

i i

i i
“Livro˙Olimpiada
i i
2006/5/27
page 65
i i

[SEC. 2.4: UM POUCO SOBRE EQUAÇÕES DIOFANTINAS 65

inteiros q1 e q2 , tais que dq1 = a e dq2 = b. Portanto, se verifica a


igualdade
dq1 x0 + dq2 y0 = d(q1 x0 + q2 y0 ) = c,
de onde segue obviamente que d | c.
Reciprocamente, suponhamos que d | c e portanto c = qd com q inteiro.
O Teorema de Bézout nos garante a existência de dois inteiros, x0 e y0 , tais
que ax0 + by0 = d. Multiplicando ambos os lados desta última igualdade
por q temos que
ax0 q + by0 q = c,
logo o par (x1 , y1 ), com x1 = x0 q e y1 = y0 q, é solução da equação dio-
fantina.
Resta provar agora que temos infinitas soluções da forma (2.32). Com
efeito, sendo (x, y) uma outra solução qualquer além de (x0 , y0 ), vale que
ax0 + by0 = c = ax + by, de onde ax0 + by0 = ax + by. Desta igualdade
obtemos a(x − x0 ) = b(y0 − y) e dividimos esta última por d para obtermos
a b
(x − x0 ) = (y0 − y).
d d
Como o ( ad , db ) = 1, então temos que ad | (y0 − y) e db | (x − x0 ). Logo, existe
inteiro t tal que
x = x0 + t db e y = y0 − t ad .
Por outro lado, é fácil verificar que para qualquer inteiro t as expressões
achadas acima para x e y resolvem a equação diofantina.
A seguir damos um exemplo de como proceder para resolver equações
diofantinas.
Exemplo 2.48. Achar todas as soluções inteiras da equação

12x + 33y = 27.

Solução. Observemos que (12, 33) = 3 e que 3 | 27, logo a equação tem
infinitas soluções. Como sabemos, basta achar uma delas e teremos as
restantes. Para achar esta solução particular podemos trabalhar de duas
maneiras, que descrevemos a seguir:
Alternativa 1: Reduzimos a equação à forma equivalente

4x + 11y = 9,

i i

i i
“Livro˙Olimpiada
i i
2006/5/27
page 66
i i

66 [CAP. 2: DIVISIBILIDADE

e por tentativa e erro vemos que x0 = 5 e y0 = −1 solucionam a mesma.


Então pela Proposição 2.47 temos que

x = 5 + 11t e y = 4t − 1, t ∈ Z,

esgotam todas as soluções que procuramos.


Alternativa 2: Aplicamos o Algoritmo de Euclides para achar o m.d.c (12, 33),
obtendo os seguintes resultados:

33 = 12 · 2 + 9,
12 = 9 · 1 + 3,
9 = 3 · 3 + 0.

Da segunda e primeira igualdades temos, respectivamente, que

3 = 12 − 9 · 1 e 9 = 33 − 12 · 2.

Usando estas duas obtemos


3 = 12 − (33 − 12 · 2) · 1
= 12 − 33 + 12 · 2
= 3 · 12 − 1 · 33,

ou seja, achamos x0 = 3 e y0 = −1, garantidos pelo Teorema de Bézout,


que validam 3 = 12x0 + 33y0 . Multiplicamos por 9 esta última igualdade
para obter
27 = 12(9x0 ) + 33(9y0 ).
Portanto, x e0 = 9x0 = 27 e ye0 = 9y0 = −9 resolvem, particularmente, a
equação diofantina. Analogamente, como na alternativa, anterior podemos
escrever a solução geral da forma:

x = 27 + 11s e y = 4s − 9, s ∈ Z.

2.5 Exercı́cios
1. Encontre o resto que deixa

i i

i i
“Livro˙Olimpiada
i i
2006/5/27
page 67
i i

[SEC. 2.5: EXERCÍCIOS 67

(a) 2001 · 2002 · 2003 · 2004 + 20052 quando é dividido por 7;


(b) 2100 quando é dividido por 3;
¡ ¢28
(c) 1237156 + 34 quando é dividido por 111.

2. Provar que o número n5 + 4n é divisı́vel por 5 para qualquer número


natural n.

3. Provar que o número n3 − n é divisı́vel por 24 para qualquer número


natural n ı́mpar.

4. Mostre que se n é ı́mpar, então n2 − 1 é divisı́vel por 8.

5. Prove que se a é ı́mpar, então a2 + (a + 2)2 + (a + 4)2 + 1 é divisı́vel


por 12.

6. O número 21093 − 2 é divisı́vel por 10932 ?

7. Utilizando o fato de que o resto de um quadrado quando dividido


por 4 só pode ser 0 ou 1, dê uma outra solução para o problema do
Exemplo 2.39.

8. Dados 3 inteiros tais que x2 +y 2 = z 2 , mostre que x e y não são ambos


ı́mpares e que xy é múltiplo de 6.

9. Demonstre que o quadrado de um inteiro é da forma 8n ou 8n + 1 ou


8n + 4.

10. Três números primos p, q e r, maiores que 3, formam uma progressão


aritmética, ou seja, q = p + d e r = p + 2d. Prove que d é divisı́vel por
6.

11. Demonstrar que existem infinitos números primos da forma 4m + 3 e


da forma 6m + 5, onde m ∈ Z.

12. Encontrar o último dı́gito dos números

(a) 19892005 ;
(b) 777777 + 250 ;
(c) 1 + 22 + 32 + · · · + 20052 .

i i

i i
“Livro˙Olimpiada
i i
2006/5/27
page 68
i i

68 [CAP. 2: DIVISIBILIDADE

13. Prove que a soma dos quadrados de cinco números consecutivos não
é um quadrado perfeito.
14. Prove que 1 |00 ·{z
· · 00} 5 00
| ·{z
· · 00} 1 não é um cubo perfeito.
100−zeros 100−zeros

15. Use o Lema dos Restos para dar uma prova diferente para a Proposição 2.5.
16. Dados n ∈ N e aj ∈ Z, j = 0, 1, · · · , n; an > 0. Prove que o conjunto
© ª
Ln = an xn + an−1 xn−1 + · · · + a0 , x ∈ Z

contém infinitos números compostos.


17. Prove que os números
1 1
(a) αn = 1 + 2 + 3 + · · · + n1 , com n > 1,
1 1 1
(b) βn = 3 + 5 + ··· + 2n+1 , com n > 0,
não são inteiros.
18. Dizemos que um conjunto An formado por n inteiros positivos escritos
no sistema binário (base 2) é regular se, para qualquer s inteiro não
negativo a quantidade de números de An que contemplam 2s na rep-
resentação binária é par. Dizemos que An é irregular se, pelo menos
para algum s este número é ı́mpar. Demonstre que um sistema ir-
regular pode se converter em regular excluindo-se apenas um único
elemento do mesmo, e, um sistema regular pode se converter em ir-
regular excluindo-se qualquer um dos seus elementos.
19. Seja n um inteiro positivo. Demonstrar que todos os coeficientes do
desenvolvimento do binômio de Newton (a + b)n são ı́mpares se, e
somente se, n é da forma 2s − 1.
20. Prove que se (x0 , y0 ) é uma solução da equação diofantina linear ax −
by = 1, então a área do triângulo cujos vértices são (0, 0), (b, a) e
(x0 , y0 ) é 1/2.
21. Qual é a menor distância possı́vel entre dois pontos (x1 , y1 ) e (x2 , y2 ),
com coordenadas inteiras, situados sobre a reta definida pela equação
diofantina ax − by = c ?

i i

i i
“Livro˙Olimpiada
i i
2006/5/27
page 69
i i

Capı́tulo 3

Contagem

Neste capı́tulo discutiremos problemas envolvendo a contagem de elementos


de um conjunto dado. Por exemplo, responderemos perguntas do tipo: de
quantos modos podemos distribuir 32 seleções nacionais de futebol em seis
grupos de quatro times cada?
Para tanto, utilizaremos como ferramentas básicas os princı́pios aditivo
e multiplicativo da contagem. Veremos também que o uso simultâneo destes
princı́pios será muito útil para resolver problemas com certos nı́veis de com-
plexidade. Além disto, serão abordados os conceitos de permutações, arran-
jos e combinações, sendo estes de muita importância por serem os alicerces
de um ramo da matemática denominado Combinatória.
Antes de prosseguirmos daremos algumas definições e notações que serão
úteis ao longo de todo o capı́tulo. Dado um conjunto A denotamos por |A|
a quantidade de elementos que este possui. O produto cartesiano de n
conjuntos A1 , A2 , . . . , An−1 e An é o conjunto definido por
© ª
A1 × A2 × · · · × An := (a1 , a2 , . . . , an ); ai ∈ Ai , i = 1, 2, . . . , n ,
onde cada elemento (a1 , a2 , . . . , an ) é chamado de n-upla ordenada. Deno-
taremos o conjunto vazio com o sı́mbolo ∅.

3.1 Princı́pio Aditivo de Contagem


O princı́pio aditivo da contagem garante que dados dois conjuntos que não
têm elemento em comum, o número de elementos da união é exatamente a

69

i i

i i
“Livro˙Olimpiada
i i
2006/5/27
page 70
i i

70 [CAP. 3: CONTAGEM

soma do número de elementos de cada um, ou seja, se A1 e A2 são disjuntos


(isto é, A1 ∩ A2 = ∅), então

|A1 ∪ A2 | = |A1 | + |A2 |.

Apesar de sua simplicidade, muitos problemas podem ser resolvidos uti-


lizando esse simples princı́pio. A seguir enunciamos uma extensão deste
princı́pio para um número finito qualquer de conjuntos.

Princı́pio Aditivo da Contagem: Dados os conjuntos A1 , A2 , . . . ,


An dois a dois disjuntos (isto é, Ai ∩ Aj = ∅ , ∀ i 6= j), temos que

|A1 ∪ A2 ∪ · · · ∪ An | = |A1 | + |A2 | + · · · + |An |.

Exemplo 3.1. Em Maceió entraram em cartaz 4 filmes distintos e 2 peças


de teatro. Se Pedro Vı́tor só tem dinheiro para assistir a um filme ou a uma
peça de teatro, diga quantos são os possı́veis programas de Pedro Vı́tor.
Solução. Denotemos por f1 , f2 , f3 e f4 os quatro filmes que estão em cartaz
e por t1 e t2 as duas peças de teatro. Agora, representemos pelo par (i, j),
com 0 ≤ i ≤ 4 e 0 ≤ j ≤ 2, o programa que consiste em assistir ao filme
fi e à peça tj (caso i = 0 ou j = 0 isso significa que não será assistido a
nenhum filme ou a nenhuma peça, respectivamente).
Pelas limitações econômicas do Pedro Vı́tor temos que ele só pode es-
colher um programa dentro dos seguintes conjuntos disjuntos:
© ª © ª
A1 = (1, 0), (2, 0), (3, 0), (4, 0) e A2 = (0, 1), (0, 2) .

Logo, no total são |A1 ∪ A2 | = |A1 | + |A2 | = 6 programas distintos, entre


os quais Pedro Vı́tor terá que escolher um.
Exemplo 3.2. Numa reunião havia um certo número de pessoas e todos os
presentes apertaram as mãos entre si. Sabendo-se que ao todo foram feitos
66 cumprimentos, calcule o número de pessoas presentes à reunião.
Solução. Vamos enumerar as pessoas com os números do conjunto P =
{1, 2, . . . , n}. A cada aperto de mão associaremos um par (i, j), significando
que a pessoa i apertou a mão da pessoa j. Assim, os apertos de mão
envolvendo a pessoa 1 foram:

A1 = {(1, 2), (1, 3), . . . , (1, n)}.

i i

i i
“Livro˙Olimpiada
i i
2006/5/27
page 71
i i

[SEC. 3.1: PRINCÍPIO ADITIVO DE CONTAGEM 71

Do mesmo modo, definimos os apertos de mão envolvendo a pessoa 2 que


não envolvem a pessoa 1, como:
A2 = {(2, 3), (2, 4), . . . , (2, n)}.
Note que o aperto (2, 1) é o mesmo que o aperto (1, 2), já que se 1 aperta a
mão de 2, então 2 aperta a mão de 1. Analogamente,
Ai = {(i, i + 1), (i, i + 2), . . . , (i, n)}, para 1 ≤ i ≤ n.
Note que Ai ∩ Aj = ∅ para i 6= j. Observe também que todos os apertos
aparecem em um dos conjuntos Ai . Assim, A1 ∪ · · · ∪ An contém todos os
apertos de mão. Logo, pelo Princı́pio Aditivo:
|A1 ∪ A2 ∪ · · · ∪ An | = |A1 | + |A2 | + . . . |An |
= (n − 1) + (n − 2) + · · · + 2 + 1
(n − 1)n
= = 66.
2
Resolvendo em n, temos que n = 12.

Vimos que o princı́pio aditivo nos fornece o número de elementos de


qualquer união de conjuntos dois a dois disjuntos. Discutiremos agora uma
extensão do princı́pio para qualquer união de conjuntos, não necessaria-
mente dois a dois disjuntos.
Proposição 3.3. Considere A1 e A2 dois conjuntos arbitrários, então:
|A1 ∪ A2 | = |A1 | + |A2 | − |A1 ∩ A2 |.
Demonstração. Observe que
A1 ∪ A2 = (A1 − A2 ) ∪ A2
onde a união é dois a dois disjunta. Pelo principio aditivo, temos que
|A1 ∪ A2 | = |A1 − A2 | + |A2 |. (3.1)

Analogamente, aplicando novamente este pricı́pio, temos que

|A1 | = |A1 − A2 | + |A1 ∩ A2 |; (3.2)


A Proposição segue imediatamente combinando as igualdades (3.1) e (3.2).

i i

i i
“Livro˙Olimpiada
i i
2006/5/27
page 72
i i

72 [CAP. 3: CONTAGEM

Para chegar a uma expressão análoga à do princı́pio aditivo, vamos fazer


mais um caso, considerando agora três conjuntos.

Corolário 3.4. Considere A1 , A2 e A3 três conjuntos quaisquer. Então,

|A1 ∪ A2 ∪ A3 | =|A1 | + |A2 | + |A3 |


¡ ¢
− |A1 ∩ A2 | + |A1 ∩ A3 | + |A2 ∩ A3 |
+ |A1 ∩ A2 ∩ A3 |.

Demonstração. Pela Proposição 3.3 temos que,

|A1 ∪ (A2 ∪ A3 )| = |A1 | + |A2 ∪ A3 | − |A1 ∩ (A2 ∪ A3 )|,

de onde,

|A1 ∪ A2 ∪ A3 | = |A1 | + |A2 ∪ A3 | − |(A1 ∩ A2 ) ∪ (A1 ∩ A3 )|.

Novamente, pela Proposição 3.3 temos que,

|A1 ∪ A2 ∪ A3 | = |A1 | + |A2 | + |A3 | − |A2 ∩ A3 | − |(A1 ∩ A2 ) ∪ (A1 ∩ A3 )|.

Aplicando mais uma vez a Proposição 3.3 temos que,

|(A1 ∩ A2 ) ∪ (A1 ∩ A3 )| = |A1 ∩ A2 | + |A1 ∩ A3 | − |(A1 ∩ A2 ) ∩ (A1 ∩ A3 ).

Combinando as duas últimas igualdades obtemos

|A1 ∪ A2 ∪ A3 | =|A1 | + |A2 | + |A3 |


¡ ¢
− |A1 ∩ A2 | + |A1 ∩ A3 | + |A2 ∩ A3 |
+ |A1 ∩ A2 ∩ A3 | ,

como desejávamos.

Para facilitar nossa escrita, vamos denotar por A1 A2 . . . Ak o conjunto


A1 ∩ A2 ∩ · · · ∩ Ak . Assim, outra forma de enunciar o Corolário 3.4 é a
seguinte:
¯ 3 ¯
¯[ ¯ X 3 X X
¯ ¯
¯ Ai ¯ = |Ai | − |Ai1 Ai2 | + |Ai1 Ai2 Ai3 |.
¯ ¯
i=1 i=1 1≤i1 <i2 ≤3 1≤i1 <i2 <i3 ≤3

i i

i i
“Livro˙Olimpiada
i i
2006/5/27
page 73
i i

[SEC. 3.1: PRINCÍPIO ADITIVO DE CONTAGEM 73

De forma geral, dados conjuntos A1 , A2 , . . . , An , as expressões anteriores


nos levam a definir os números:
n
X
S1 = |Ai |
i=1
X
S2 = |Ai1 Ai2 |,
1≤i1 <i2 ≤n
..
.
X
Sk = |Ai1 Ai2 . . . Aik |,
1≤i1 <i2 <···<ik ≤n
..
.
Sn = |A1 A2 . . . An |.

Assim, a versão mais geral do princı́pio aditivo, também conhecida como


princı́pio de inclusão e exclusão, é:

Princı́pio Aditivo - Versão Geral: Dados quaisquer conjuntos


A1 , A2 . . . , An , vale:
¯ n ¯
¯[ ¯
¯ ¯
¯ Ai ¯ = S1 − S2 + S3 − S4 + · · · + (−1)n−1 Sn .
¯ ¯
i=1

Não iremos provar essa versão, mas o leitor pode (e deve!) mostrá-la
como exercı́cio, repetindo os argumentos anteriores. Vamos agora a uma
aplicação:

Exemplo 3.5. Num colégio foram entrevistados 78 estudantes. Destes, 32


estavam fazendo um curso de francês; 40 um curso de fı́sica; 30 um curso
de matemática; 23 um curso de história; 19 francês e fı́sica; 13 francês e
matemática; 15 fı́sica e matemática; 2 francês e história; 15 fı́sica e história;
14 matemática e história; 8 francês, fı́sica e matemática; 8 francês, fı́sica
e história; 2 francês, matemática e história; 6 fı́sica, matemática e história
e 2 estavam fazendo todos os quatro cursos. Quantos estudantes estavam
fazendo pelo menos 1 curso nas 4 áreas mencionadas?

i i

i i
“Livro˙Olimpiada
i i
2006/5/27
page 74
i i

74 [CAP. 3: CONTAGEM

Solução. Denotemos por A1 , A2 , A3 , e A4 os conjuntos dos estudantes


que fazem francês, fı́sica, matemática e história, respectivamente. Observe-
mos que as igualdades

|A1 | = 32,
|A2 | = 40,
|A3 | = 30,
|A4 | = 23,

4
X
nos dão que S1 = |Ai | = 125; as igualdades
i=1

|A1 A2 | = 19,
|A1 A3 | = 13,
|A1 A4 | = 2,
|A2 A3 | = 15,
|A2 A4 | = 15,
|A3 A4 | = 14,
X
nos dão que S2 = |Ai1 Ai2 | = 78; as igualdades
1≤i1 <i2 ≤4

|A1 A2 A3 | = 8,
|A1 A2 A4 | = 8,
|A1 A3 A4 | = 2,
|A2 A3 A4 | = 6,

X
nos dão que S3 = |Ai1 Ai2 Ai3 | = 24; assim como que S4 =
1≤i1 <i2 <i3 ≤4
|A1 A2 A3 A4 | = 2. ¯ ¯
¯[4 ¯
¯ ¯
Segue-se então, do princı́pio aditivo, que ¯ Ai ¯ = 125 − 78 + 24 − 2 =
¯ ¯
i=1
69.

i i

i i
“Livro˙Olimpiada
i i
2006/5/27
page 75
i i

[SEC. 3.1: PRINCÍPIO ADITIVO DE CONTAGEM 75

Definição 3.6. Definimos o complementar do conjunto A em relação ao


conjunto U como sendo um subconjunto de U dado por
© ª
Ac = x ∈ U; x ∈/A .

Figura 3.1: A área pintada de verde corresponde a Ac e o conjunto U é repre-


sentado por todo o retângulo.

Neste caso é fácil verificar que os conjuntos A e Ac são disjuntos e que


U = A ∪ Ac . Segue-se do princı́pio aditivo que |U| = |A| + |Ac |; portanto,

|Ac | = |U| − |A|.

Analogamente, dados dois conjuntos A1 ⊂ U e A2 ⊂ U, temos que A1 ∪ A2


e (A1 ∪ A2 )c são disjuntos e, aliás, U = (A1 ∪ A2 ) ∪ (A1 ∪ A2 )c . Novamente,
pelo do princı́pio aditivo, vale que

|U| = |A1 ∪ A2 | + |(A1 ∪ A2 )c |;

e conseqüentemente temos que

|(A1 ∪ A2 )c | = |U| − (|A1 | + |A2 |) + |A1 A2 |.

Similarmente, dados três conjuntos A1 ⊂ U, A2 ⊂ U e A3 ⊂ U podemos


demonstrar que

|(A1 ∪ A2 ∪ A3 )c | = |U| − (|A1 | + |A2 | + |A3 |)


+ (|A1 A2 | + |A1 A3 | + |A2 A3 |)
− |A1 A2 A3 |.

Então, usando a notação S0 = |U|, temos a seguinte proposição:

i i

i i
“Livro˙Olimpiada
i i
2006/5/27
page 76
i i

76 [CAP. 3: CONTAGEM

Proposição 3.7. Para toda famı́lia de subconjuntos Ai ⊂ U, i = 1, 2, . . . , n,


vale a relação:
¯Ã n !c ¯
¯ [ ¯ ¡ ¢
¯ ¯
¯ Ai ¯ = S0 − S1 − S2 + S3 − S4 − · · · + (−1)n−1 Sn
¯ ¯
i=1
= S0 − S1 + S2 − S3 + S4 − · · · + (−1)n Sn ,

ou resumidamente,
¯Ã n !c ¯
¯ [ ¯ n
X
¯ ¯
¯ Ai ¯ = |Ac1 Ac2 · · · Acn | = (−1)j Sj .
¯ ¯
i=1 j=0

Observação 3.8. Observemos que na última relação da proposição usamos


a conhecida lei de Morgan: o complementar da união de uma familia finita
conjuntos, em relação a um conjunto U, é a intersecção dos complementares
de cada um deles.

3.2 Princı́pio Multiplicativo de Contagem


Começamos esta seção discutindo um problema relacionado com o apaixo-
nante jogo de xadrez. O mesmo consiste no seguinte: queremos saber de
quantas maneiras diferentes podemos colocar duas torres num tabuleiro de
xadrez de forma tal que nenhuma ataque a outra. Uma situação como a
que procuramos é mostrada na Figura 3.2, pois lembramos que torres só
se movimentam na direção horizontal ou na direção vertical do tabuleiro.
Antes de prosseguir deixamos claro o seguinte: se na Figura 3.2 trocamos
a posição da torre a com a torre b consideraremos isto como uma situação
diferente.
Notemos o seguinte: uma vez que coloquemos uma das torres numa
casa do tabuleiro não podemos colocar a segunda torre na mesma linha ou
coluna em que esta se encontra, pois ela seria ameaçada. Como cada linha e
cada coluna contém 8 casas do tabuleiro, sendo uma delas comum a ambas,
então temos 15 posições proibidas para colocar a segunda torre, ou seja, ela
só pode ser colocada em 64 − 15 = 49 posições diferentes. Resumindo, por
cada uma das 64 possı́veis posições para a torre a temos 49 possibilidades
diferentes para colocar a torre b, totalizando 64·49 = 3136 formas diferentes
de colocar ambas as torres no tabuleiro sem que elas se ataquem.

i i

i i
“Livro˙Olimpiada
i i
2006/5/27
page 77
i i

[SEC. 3.2: PRINCÍPIO MULTIPLICATIVO DE CONTAGEM 77

Figura 3.2::

O exemplo acima traz a essência do que é chamado princı́pio multiplica-


tivo da contagem: se um evento A1 pode ocorrer de m maneiras distintas
e, se para cada uma dessas m maneiras possı́veis de A1 ocorrer, um outro
evento A2 pode ocorrer de n maneiras distintas, então o número de maneiras
de ocorrerem sucessivamente os eventos A1 e A2 é m · n. Na linguagem
matemática: dados dois conjuntos A1 e A2 , temos que

|A1 × A2 | = |A1 | |A2 |.

Uma extensão deste princı́pio para um número finito qualquer de con-


juntos é a seguinte:

Princı́pio Multiplicativo da Contagem: Dados os conjuntos A1 , A2 ,


. . . , An temos que

|A1 × A2 × · · · × An | = |A1 | · |A2 | · · · |An |.

Note que neste princı́pio, não é necessária nenhuma hipótese adicional


sobre os conjuntos Ai . Vamos agora dar alguns exemplos de como aplicar
esse princı́pio.
Exemplo 3.9. Em Maceió entraram em cartaz 4 filmes distintos e 2 peças
de teatro. Se agora o Pedro Vı́tor tem dinheiro para assistir exatamente a
um filme e a uma peça de teatro, diga quantos são os possı́veis programas
que Pedro Vı́tor pode fazer.

i i

i i
“Livro˙Olimpiada
i i
2006/5/27
page 78
i i

78 [CAP. 3: CONTAGEM

Solução. Denotemos por f1 , f2 , f3 e f4 os quatro filmes que estão em cartaz


e por t1 e t2 as duas peças de teatro. Definamos os conjuntos

A1 = {f1 , f2 , f3 , f4 } e A2 = {t1 , t2 }.

Neste caso, as condições econômicas do Pedro Vı́tor permitem que ele


escolha um elemento do conjunto A1 e outro elemento do conjunto A2 . Este
tipo de escolha representa-se pelo conjunto
© ª
A1 × A2 = (fi , tj ); 1 ≤ i ≤ 4 e 1 ≤ j ≤ 2 ,
onde cada par (fi , tj ) representa o programa que consiste em assistir ao filme
fi e à peça tj . Logo, no total são |A1 × A2 | = |A1 | · |A2 | = 8 programas
distintos.

Exemplo 3.10. Se numa loja de doces existem 9 tipos distintos de balas


e 5 tipos distintos de chiclete, diga quantas escolhas podemos fazer para
comprar somente uma bala e um chiclete.

Solução. Denotemos por b1 , b2 , b3 , b4 , b5 , b6 , b7 , b8 e b9 os nove tipos


distintos de balas e por c1 , c2 , c3 , c4 e c5 os cinco tipos distintos de chicletes.
Definamos os conjuntos

B = {b1 , b2 , b3 , b4 , b5 , b6 , b7 , b8 , b9 } e C = {c1 , c2 , c3 , c4 , c5 }.

Como precisamos comprar simultaneamente um elemento do conjunto B


e um elemento do conjunto C, então o conjunto B × C me dá o conjunto
de todas as escolhas possı́veis. Logo, o número de escolhas possı́veis para
comprar simultaneamente um tipo de bala e um tipo de chiclete é |B × C| =
9 · 5 = 45.

Exemplo 3.11. O alfabeto Português é formado por 5 vogais e 18 con-


soantes. Suponha que uma sı́laba do alfabeto esteja formada por uma única
consoante seguida de uma única vogal. Diga quantas sı́labas tem nosso
alfabeto.

Solução. Considere os conjuntos,

V ={ a, e, i, o, u },
C ={ b, c, d, f, g, h, j, l, m, n, p, q, r, s, t, v, x, z }.

i i

i i
“Livro˙Olimpiada
i i
2006/5/27
page 79
i i

[SEC. 3.2: PRINCÍPIO MULTIPLICATIVO DE CONTAGEM 79

Uma sı́laba, pela definição dada no nosso problema, é formada por uma
única consoante seguida de uma única vogal. Assim, o conjunto de todas
as sı́labas de nosso alfabeto é formado pelo conjunto C × V. Como |C| = 18
e |V| = 5, segue-se que o número de sı́labas possı́veis é 18 · 5 = 90.
Exemplo 3.12. De quantas maneiras 2 pessoas podem estacionar seus
carros numa garagem com 10 vagas?
Solução. Observando que a primeira pessoa pode estacionar seu carro de
10 formas distintas e que a segunda pessoa pode estacionar seu carro de 9
formas distintas, temos pelo princı́pio multiplicativo que existem 9 · 10 = 90
formas possı́veis nas quais duas pessoas podem estacionar seus carros numa
garagem com 10 vagas.
Exemplo 3.13. Dado o número 720, diga
(a) quantos divisores inteiros e positivos ele possui;
(b) dentre seus divisores inteiros e positivos, quantos são pares;
(c) dentre seus divisores inteiros e positivos, quantos são ı́mpares;
(d) dos divisores acima aludidos quantos são quadrados perfeitos.
Solução. Pelo Teorema Fundamental da Aritmética, todo número inteiro
positivo é primo ou produto de primos. Observe que a decomposição de 720
em fatores primos vem dada por:

720 = 24 · 32 · 51 . (3.3)

Agora definamos os seguintes conjuntos:

A ={ todos os divisores de 720 que são da forma 2k , onde k ∈ Z+ },


B ={ todos os divisores de 720 que são da forma 3m , onde m ∈ Z+ },
C ={ todos os divisores de 720 que são da forma 5n , onde n ∈ Z+ }.

Observemos que 0 ≤ k ≤ 4, pois se k > 4 então pelo menos a potência


25 deveria estar presente em (3.3); como isto não acontece segue-se que
0 ≤ k ≤ 4, de modo que
© ª
A = 20 , 21 , 22 , 23 , 24 ,

i i

i i
“Livro˙Olimpiada
i i
2006/5/27
page 80
i i

80 [CAP. 3: CONTAGEM

seguindo o mesmo raciocı́nio, podemos demonstrar que 0 ≤ m ≤ 2 e que


0 ≤ n ≤ 1. Assim,
© ª © ª
B = 30 , 31 , 32 e C = 50 , 51 .

(a) O conjunto de todos os possı́veis divisores de 720 vem dado pelo con-
junto A × B × C. De onde o número de divisores inteiros e positivos de
720 é |A × B × C|. Porém, o Princı́pio Multiplicativo nos garante que
|A × B × C| = |A| · |B| · |C|. Portanto, o número de divisores inteiros e
positivos de 720 é 5 × 3 × 2 = 30, pois |A| = 5, |B| = 3 e |C| = 2.

(b) Para obter o conjunto de todos os divisores pares de 720 devemos


remover o elemento 20 do conjunto A. Assim, o conjunto de todos os
0
divisores pares e positivos de 720 vem dado pelo ¯conjunto
¡ A
¢ − {2 }¯ ×
¯B×C. O0 princı́pio
¯
¯
multiplicativo nos garante que A−{2 } ×B×C ¯ =
0

¯A − {2 }¯ · |B| · |C|. Portanto, o número de divisores pares e positivos


¯ ¯
de 720 é 4 × 3 × 2 = 24, pois ¯A − {20 }¯ = 4, |B| = 3 e |C| = 2.

(c) Para obter o conjunto de todos os divisores ı́mpares de 720 devemos


remover os elementos 21 , 22 , 23 e 24 do conjunto A. Assim, o con-
junto de todos os divisores ı́mpares e positivos de 720 vem dado pelo
conjunto ¡ ¢
A − {21 , 22 , 23 , 24 } × B × C.
O princı́pio multiplicativo nos garante que
¯¡ ¢ ¯ ¯ ¯
¯ A − {21 , 22 , 23 , 24 } × B × C ¯ = ¯A − {21 , 22 , 23 , 24 }¯ · |B| · |C|.

Portanto,
¯ o número de divisores ¯ ı́mpares e positivos de 720 é 1×3×2 =
6; pois ¯A − {21 , 22 , 23 , 24 }¯ = 1, |B| = 3 e |C| = 2.

(d) Para obter o conjunto de todos os divisores de 720 que são quadrados
perfeitos devemos ficar com as potências pares nos conjuntos A, B e
C, respectivamente. Portanto, devemos remover os elementos 21 , 23
do conjunto A. Também devemos remover o elemento 31 do conjunto
B. Finalmente do conjunto C devemos remover o elemento 51 . Logo, o
conjunto de todos os divisores quadrados perfeitos e positivos de 720
vem dado pelo conjunto
¡ ¢ ¡ ¢ ¡ ¢
D := A − {21 , 23 } × B − {31 } × C − {51 } .

i i

i i
“Livro˙Olimpiada
i i
2006/5/27
page 81
i i

[SEC. 3.2: PRINCÍPIO MULTIPLICATIVO DE CONTAGEM 81

O princı́pio multiplicativo nos garante que


¯ ¯ ¯ ¯ ¯ ¯ ¯ ¯
¯D¯ = ¯A − {21 , 23 }¯ · ¯B − {31 }¯ · ¯C − {51 }¯.

Portanto, o número¯ de divisores¯quadrados ¯ perfeitos


¯ e positivos
¯ de
¯ 720
é 3 · 2 · 1 = 6; pois ¯A − {21 , 23 }¯ = 3, ¯B − {31 }¯ = 2 e ¯C − {31 }¯ = 1.
Observe que {1, 4, 9, 16, 36, 144} é o conjunto dos divisores de 720 que
são quadrados perfeitos.

Exemplo 3.14. Se um número natural n se fatora como

n = pk11 · pk22 · · · pkr r , (3.4)

onde os pi são números primos distintos e cada ki ∈ Z+ , então o número de


divisores positivos de n,denotado por d(n) é

d(n) = (k1 + 1)(k2 + 1) . . . (kr + 1).

Solução. Defina o conjunto

A1 ={ todos os divisores de n que são da forma pm +


1 , onde m ∈ Z },
1

e em geral, defina

Ai ={ todos os divisores de n que são da forma pm +


i , onde t ∈ Z }.
i

Observemos que mi ≤ pi , pois se mi > pi , então pelo menos a potência


pki i +1 deveria estar presente em (3.4); como isto não acontece segue-se que
mi ≤ pi , de modo que
© ª
Ai = p0i , p1i , p2i , . . . , pki i , para i = 1, 2, 3, . . . , ki .
¯ ¯
É imediato ver que ¯Ai ¯ = ki + 1.
O conjunto de todos os possı́veis divisores de n vem dado pelo conjunto
A1 × A2 × · · · × Ar . De onde se conclui que o número de divisores inteiros
e positivos de n é

d(n) = |A1 × A2 × · · · × Ar | = |A1 | · |A2 | · · · |Ar |,

i i

i i
“Livro˙Olimpiada
i i
2006/5/27
page 82
i i

82 [CAP. 3: CONTAGEM

onde na última igualdade usamos o Princı́pio Multiplicativo. Portanto, o


número de divisores inteiros e positivos de n é

d(n) = (k1 + 1)(k2 + 1) . . . (kr + 1).

Exemplo 3.15. De quantas maneiras podemos escolher dois inteiros de 1


a 20 de forma que a soma seja ı́mpar?

Solução. Observemos que

• A soma de dois números inteiros pares é um número par. Com efeito,


para quaisquer a, b ∈ Z temos que 2a + 2b = 2(a + b);

• A soma de dois números inteiros ı́mpares é um número par. Com


efeito, para quaisquer a, b ∈ Z temos que (2a + 1) + (2b + 1) = 2(a +
b + 1);

• A soma de um número inteiro par com qualquer outro inteiro ı́mpar


sempre é um inteiro ı́mpar. Com efeito, para quaisquer a, b ∈ Z temos
que 2a + (2b + 1) = 2(a + b) + 1.

Isto nos sugere definir os conjuntos

P = {2, 4, 6, 8, 10, 12, 14, 16, 18, 20},


I = {1, 3, 5, 7, 9, 11, 13, 15, 17, 19},

onde P × I são todas as formas possı́veis de somar um número inteiro par


com outro ı́mpar. O princı́pio multiplicativo nos garante que nossa resposta
é |P × I| = |P| · |I| = 100, pois |P| = |I| = 10.

3.3 Uso simultâneo dos Princı́pios Aditivo e


Multiplicativo
Aproveitamos esta seção para apresentar problemas um pouco mais difı́ceis
que os tratados nas seções anteriores. Nestes problemas, precisaremos em-
pregar simultaneamente o princı́pio aditivo e o princı́pio multiplicativo. Va-
mos ao primeiro deles:

i i

i i
“Livro˙Olimpiada
i i
2006/5/27
page 83
i i

[SEC. 3.3: USO SIMULTÂNEO DOS PRINCÍPIOS ADITIVO E MULTIPLICATIVO 83

Exemplo 3.16. Sabemos que no inı́cio da premiação da 1a fase da Olimpı́ada


Alagoana de Matemática existem 10 livros diferentes de Álgebra, 7 livros
diferentes de Combinatória e 5 livros diferentes de Geometria para home-
nagear os vencedores. Danielle é a primeira a pegar o prêmio que consiste
em 2 livros, com a condição de que estes não podem ser da mesma matéria.
Diga quantas escolhas Danielle pode fazer para pegar seu prêmio.
Solução. Denotemos por
A = {a1 , . . . , a10 }, C = {c1 , . . . , c7 } e G = {g1 , . . . , g5 }, os conjuntos de
livros de Álgebra, Combinatória e Geometria, respectivamente. Observemos
que |A| = 10, |C| = 7 e |G| = 5 e Danielle tem as seguintes possibilidades
de escolha:
• Escolher um livro de A e um livro de C. Neste caso, Danielle tem
|A × C| = |A| · |C| = 70 escolhas possı́veis (devido ao Princı́pio Multi-
plicativo).
• Escolher um livro de A e um livro de G. Neste caso, Danielle tem
|A × G| = |A| · |G| = 50 escolhas possı́veis (devido ao Princı́pio Multi-
plicativo) ou
• Escolher um livro de C e um livro de G. Neste caso, Danielle tem
|C × G| = |C| · |G| = 35 escolhas possı́veis (devido ao Princı́pio Multi-
plicativo).
Agora o Princı́pio aditivo nos garante que o número total de escolhas que
Danielle pode fazer é 70 + 50 + 35 =155.

Exemplo 3.17. Há 18 moças e 12 rapazes, onde 5 deles são irmãos (3


moças e 2 rapazes) e os restantes não possuem parentesco. Diga quantos
casamentos são possı́veis naquela turma (sabendo que irmãos não se casam).
Solução. Observemos que 15, dentre as 18 moças, não têm parentesco
nenhum com os 12 rapazes, logo, pelo Princı́pio Multiplicativo temos que é
possı́vel efetuar 15 · 12 = 180 casamentos diferentes entre eles. Por outro
lado, as 3 moças restantes podem efetuar casamento com 10 dos 12 rapazes,
pois 2 deles são seus irmãos. Novamente, pelo Princı́pio Multiplicativo é
possı́vel realizar 3 · 10 = 30 casamentos diferentes neste caso. Finalmente, o
Princı́pio Aditivo nos dá que podem ser realizados um total de 180+30 = 210
casamentos.

i i

i i
“Livro˙Olimpiada
i i
2006/5/27
page 84
i i

84 [CAP. 3: CONTAGEM

Exemplo 3.18. Quantas palavras de 5 caracteres podem ser formadas com


as letras α, β e γ de modo que em cada palavra não falte nenhuma dessas
letras?

Solução. Definamos os seguintes conjuntos,

U :={ palavras de 5 caracteres só com as letras α, β e γ};


Aα :={ palavras que estão em U e onde não aparece a letra α};
Aβ :={ palavras que estão em U e onde não aparece a letra β};
Aγ :={ palavras que estão em U e onde não aparece a letra γ}.

Por exemplo,

• a palavra γγγγγ ∈ Aα ∩ Aβ ;

• a palavra γααγα ∈ Aβ ;

• a palavra βαβββ ∈ Aγ .

Primeiramente, notemos que cada caracter de U pode ser escolhido de


3 formas distintas. Segue-se então do Princı́pio Multiplicativo que existem
35 formas de escrever uma palavra de 5 caracteres usando um alfabeto de 3
letras, isto é,
S0 = |U| = 35 = 243.
Calculemos agora |Aα |, isto é, o número de palavras onde não aparece a
letra α. Para isto, observemos que cada caracter em Aα pode ser escolhido
de 2 formas. Logo, o Princı́pio Multiplicativo nos garante que existem 25
palavras em Aα , ou seja, |Aα | = 25 . Analogamente, podemos mostrar que
|Aβ | = |Aγ | = 25 . Portanto,

S1 = |Aα | + |Aβ | + |Aγ | = 25 + 25 + 25 = 96.

Prosseguimos com o cálculo de |Aα Aβ |, isto é, do número de palavras


onde não aparecem as letras α e β; portanto, cada caracter em Aα Aβ pode
ser escolhido de 1 forma. Logo, o Princı́pio Multiplicativo nos garante que
existem 15 = 1 palavra em Aα Aβ , ou seja, |Aα Aβ | = 1. Similarmente,
podemos mostrar que |Aα Aγ | = |Aβ Aγ | = 1. Portanto,

S2 = |Aα | + |Aβ | + |Aγ | = 3.

i i

i i
“Livro˙Olimpiada
i i
2006/5/27
page 85
i i

[SEC. 3.4: PERMUTAÇÕES SIMPLES 85

Por fim, achamos |Aα Aβ Aγ |, que nos dá o número de palavras onde não
aparecem as letras α, β e γ; mas cada palavra em Aα Aβ Aγ tem que usar
pelo menos um dos caracteres proibidos. Logo,

S3 = |Aα Aβ Aγ | = 0.

Finalmente, observamos que o conjunto das palavras de 5 caracteres que


podem ser formadas com as letras α, β e γ de modo que em cada palavra
não falte nenhuma dessas letras é exatamente o conjunto Acα Acβ Acγ . Usando
a Proposição 3.7, temos:

|Acα Acβ Acγ | =S0 − S1 + S2 − S3


=243 − 96 + 3 − 0
=150.

3.4 Permutações Simples


Definição 3.19. Uma permutação simples de n objetos distintos é qualquer
agrupamento ordenado desses n objetos. Denotaremos por Pn o número de
todas as permutações simples de n objetos dados.

Por exemplo, todas as permutações dos 3 elementos do conjunto A =


{a1 , a2 , a3 } são:

σ1 = (a1 , a2 , a3 ),
σ2 = (a1 , a3 , a2 ),
σ3 = (a2 , a1 , a3 ),
σ4 = (a2 , a3 , a1 ),
σ5 = (a3 , a1 , a2 ),
σ6 = (a3 , a2 , a1 ).

Proposição 3.20. Seja n ≥ 1. O número total de permutações simples de


n objetos O = {o1 , o2 , . . . , on } é dado por Pn = n!

i i

i i
“Livro˙Olimpiada
i i
2006/5/27
page 86
i i

86 [CAP. 3: CONTAGEM

Demonstração. É claro que a fórmula vale para n = 1. Vejamos agora que


existe a seguinte relação entre Pn e Pn−1 para n ≥ 2:

Pn = nPn−1 . (3.5)

Para comprovar isto, para cada i definamos Ai como sendo as permutações


dos n − 1 objetos {o1 , . . . , oi−1 , oi+1 , . . . , on }. Note que |Ai | = Pn−1 , para
cada i = 1, 2, . . . , n. Assim, para obtermos uma permutação dos n objetos,
basta que fixemos o objeto inicial oi e tomemos um elemento do conjunto Ai ,
que é uma permutação dos n − 1 objetos restantes. Pelo princı́pio aditivo,
temos que:
Pn = |A1 | + |A2 | + · · · + |An | = nPn−1 .
Como nossa equação (3.5) é válida para todo n ≥ 2, podemos aplicá-la para
n − 1, obtendo:
Pn−1 = (n − 1)Pn−2 ,
de onde vem que
Pn = n(n − 1)Pn−2 .
Repetindo este argumento, obtemos que

Pn = n(n − 1)(n − 2) · · · 3 · 2 · 1 = n!,

como querı́amos demonstrar.


Exemplo 3.21. De quantas maneiras podemos formar uma fila com 4 pes-
soas?
Demonstração. Observe que se enumeramos os lugares da fila e enumer-
amos as pessoas, pa , pb , pc , pd , cada distribuição vai corresponder a uma per-
mutação do conjunto {1, 2, 3, 4}. Por exemplo, a distribuição (pc , pa , pb , pd )
corresponde à permutação (3, 1, 2, 4). Assim, o número de distribuições na
fila é 4! = 24.
Exemplo 3.22. De quantas maneiras k moças e k rapazes podem formar
pares para uma dança?
Solução. Estando as moças em uma fila e os rapazes em outra, podemos
enumerá-los com números de 1, 2, . . . , k. A uma permutação desses números,
digamos (a1 , a2 , . . . , ak ) com ai ∈ {1, 2, . . . , k} faremos uma associação da
mulher i com o rapaz ai . Por exemplo, a permutação (2, 1, 3, . . . , k) significa

i i

i i
“Livro˙Olimpiada
i i
2006/5/27
page 87
i i

[SEC. 3.5: ARRANJOS SIMPLES 87

que a moça 1 dançará com o rapaz 2, a moça 2 com o rapaz 1, e a moça i


com o rapaz i, para i ≥ 3.
Observe que toda associação de k moças e k rapazes produz uma per-
mutação, de modo que o número de associações possı́veis das moças com os
rapazes é igual ao número de permutações do conjunto {1, 2, 3, . . . , k}. Pela
Proposição 3.20 existem k! modos diferentes de combinar as moças com os
rapazes.

3.5 Arranjos Simples


Definição 3.23. Consideremos n objetos e p um inteiro positivo tal que
0 < p ≤ n. Um arranjo simples de classe p dos n objetos dados é uma
seleção de p objetos distintos dentre estes que diferem entre si pela ordem
de colocação ou pela natureza de cada um, isto é, o que importa é quem
participa ou o lugar que ocupa. Denotaremos por Apn o número de arranjos
simples de classe p de n objetos.

Por exemplo, dados os objetos o1 , o2 e o3 todos os arranjos possı́veis de


classe 2 são: A1 = (o1 , o2 ), A2 = (o2 , o1 ), A3 = (o1 , o3 ), A4 = (o3 , o1 ),
A5 = (o2 , o3 ) e A6 = (o3 , o2 ).

Observação 3.24. Notemos que um arranjo simples de classe n de n


objetos dados não é mais que uma permutação desses n objetos. Logo,
Pn = Ann = n!.

Proposição 3.25. Seja n ≥ 1. O número total de arranjos simples de


n!
classe p de n objetos O = {o1 , o2 , . . . , on } é dado por Apn = (n−p)! .

Demonstração. Para n = 1 a fórmula é obviamente válida. Similarmente


ao caso das permutações, primeiramente provaremos que para n ≥ 2 vale a
seguinte igualdade:
Apn = nAp−1
n−1 . (3.6)
Agora definimos os conjuntos Ai como sendo os arranjos simples de classe
p − 1 dos n − 1 objetos {o1 , . . . , oi−1 , oi+1 , . . . , on }. Note que |Ai | = Ap−1
n−1 ,
para cada i = 1, 2, . . . , n. Assim, para obtermos um arranjo simples de
classe p dos n objetos, basta que fixemos o objeto inicial oi e tomemos um

i i

i i
“Livro˙Olimpiada
i i
2006/5/27
page 88
i i

88 [CAP. 3: CONTAGEM

elemento do conjunto Ai , que é uma arranjo de classe p−1 dos n−1 objetos
restantes. Pelo princı́pio aditivo, temos que:

Apn = |A1 | + |A2 | + · · · + |An | = nAp−1


n−1 .

Como nossa equação (3.6) é válida para todo n ≥ 2, podemos aplicá-la para
n − 1, obtendo:
Ap−1 p−2
n−1 = (n − 1)An−2 ,

de onde vem que


Apn = n(n − 1)Ap−2
n−2 .

Repetindo este argumento sucessivamente, obtemos que

p−(p−1)
Apn = n(n − 1)(n − 2) · · · (n − (p − 2))An−(p−1)
= n(n − 1)(n − 2) · · · (n − p + 2)A1n−p+1 .

Notemos agora que A1n−p+1 = n − p + 1; logo, da igualdade acima segue-se


que

Apn = n(n − 1)(n − 2) · · · (n − p + 2)(n − p + 1)


n(n − 1)(n − 2) · · · (n − p + 2)(n − p + 1) × (n − p) · · · 1
=
(n − p) · · · 1
n!
= ,
(n − p)!

como desejávamos.

Agora vamos dar alguns exemplos de como aparecem problemas práticos


que requerem fazer este tipo de cálculo. O primeiro dele tem a ver com a
formação de palavras diferentes com um conjunto dado de leras.
Um anagrama de uma palavra é uma transposição de letras dessa palavra
para formar outra palavra. Por exemplo:

• Um anagrama de amor é Roma;

• Um anagrama de Célia é Alice;

• Um anagrama de Caterina é Natércia;

i i

i i
“Livro˙Olimpiada
i i
2006/5/27
page 89
i i

[SEC. 3.5: ARRANJOS SIMPLES 89

• Um anagrama de Elvis é Lives.

Exemplo 3.26. Quantos anagramas de p letras distintas podemos formar


com um alfabeto de 23 letras, sendo p < 23 ?

Solução. Claramente nosso problema consiste em achar todos os arranjos


de classe p de 23 objetos dados, que neste caso são as 23 letras do alfabeto.
Logo, este número é
23!
Ak23 = .
(23 − k)!

Exemplo 3.27. De quantos modos 2 pessoas podem se sentar em 5 cadeiras


que estão em fila?

Solução. Este problema é equivalente a achar o número total de arranjos de


classe 2 de 5 objetos, correspondendo as 5 cadeiras aos 5 objetos e as duas
pessoas indicando a ordem do arranjo. Logo, este número é dado por

5!
A25 = = 20.
3!
A Tabela 3.1 mostra como fazer todas as distribuições possı́veis. Na
mesma, p1 e p2 denotam as duas pessoas e c1 , c2 , · · · , c5 as 5 cadeiras em
fila. Observe que a tabela é dividida em 5 grupos de 4 filas cada um,
correspondendo o i-ésimo deles (1 ≤ i ≤ 5) a todos os arranjos possı́veis
quando a pessoa p1 se senta na cadeira ci .

Exemplo 3.28. Considere os dı́gitos 2, 3, 4, 5, 7 e 9. Supondo que a repetição


de dı́gitos não seja permitida, responda as seguintes perguntas:

(a) Quantos números de três dı́gitos podem ser formados ?

(b) Dentre os achados em (a) quantos são pares ?

(c) Dentre os achados em (a) quantos são ı́mpares ?

(d) Dentre os achados em (a) quantos são múltiplos de 5 ?

(e) Dentre os achados em (a) quantos são menores do que 400 ?

i i

i i
“Livro˙Olimpiada
i i
2006/5/27
page 90
i i

90 [CAP. 3: CONTAGEM

c1 c2 c3 c4 c5
p1 p2
p1 p2
p1 p2
p1 p2
p2 p1
p1 p2
p1 p2
p1 p2
p2 p1
p2 p1
p1 p2
p1 p2
p2 p1
p2 p1
p2 p1
p1 p2
p2 p1
p2 p1
p2 p1
p2 p1

Tabela 3.1: Cada linha da tabela indica uma forma diferente de distribuir as
duas pessoas nas 5 cadeiras.

Solução. Seja O = {2, 3, 4, 5, 7, 9} nosso conjunto de objetos

(a) A quantidade de números de três dı́gitos que podemos formar sem


repetição de algum deles é claramente o número de arranjos de classe
3 dos 6 dı́gitos de O, isto é,
6!
A36 = = 120.
3!

(b) Sabemos que em todo número par o último dı́gito é um múltiplo de


2, isto é, ele acaba em 0, 2, 4, 6 ou 8. Então, em nosso caso as únicas
possibilidades são que o número termine em 2 ou 4. Supondo que

i i

i i
“Livro˙Olimpiada
i i
2006/5/27
page 91
i i

[SEC. 3.6: COMBINAÇÕES SIMPLES 91

o último dı́gito seja 2, temos que preencher as duas casas restantes


com os dı́gitos pertencentes ao conjunto O − {2}. Assim, existem
5!
A2|O−{2}| = A25 = 3! = 20 números dos achados em (a) que finalizam
5!
em 2. De forma análoga, existem A2|O−{4}| = A25 = 3! = 20 números
dos achados em (a) que finalizam em 4. Logo, dentre os números
achados em (a) existem 20 + 20 = 40 números pares.

(c) Todo conjunto de números pode ser dividido em duas classes disjun-
tas: a classe dos números pares e a classe dos números ı́mpares que
pertencem ao mesmo. Segue-se que dentre os números achados em (a)
existem 120 − 40 = 80 números ı́mpares.

(d) Todo número múltiplo de 5 acaba em 0 ou 5; no nosso caso temos


que a única possibilidade para o último dı́gito é 5. Assim o problema
consiste em preencher as duas casas restantes com dı́gitos do conjunto
O − {5}. De onde se segue que a quantidade de números múltiplos de
5 existentes em (a) vem dada por A2|O−{5}| = A25 = 5! 3! = 20.

(e) Para obter os números menores do que 400 a casa das centenas só
poderá ser ocupada pelos dı́gitos 1, 2 ou 3. Como 1 ∈ / O, temos que
as únicas possibilidades em nosso caso são 2 ou 3. Então, supondo
que o primeiro dı́gito do número seja 2, devemos preencher duas casas
restantes com os dı́gitos pertencentes a O − {2}. De forma análoga,
5!
existem A2|O−{3}| = A25 = 3! = 20 números dos achados em (a) e que
começam com 3. Logo, dentre os números achados em (a) existem
20 + 20 = 40 menores do que 400.

3.6 Combinações Simples


O conceito de combinação simples surge naturalmente quando intentamos
responder à seguinte pergunta: de quantas formas diferentes podemos sele-
cionar p objetos dentro de n objetos dados?
Por exemplo, suponha que queremos enfeitar uma festa de aniversário
com bolas de dois tipos de cores e na loja onde as compraremos existem bolas
nas cores azul, verde e vermelha. De quantas formas distintas podemos
enfeitar nossa festa ? É claro que podemos enfeitar a festa de 3 formas

i i

i i
“Livro˙Olimpiada
i i
2006/5/27
page 92
i i

92 [CAP. 3: CONTAGEM

diferentes: com bolas em azul e verde; com bolas em azul e vermelho ou


com bolas em verde e vermelho.
Notemos que, ao contrário do caso em que trabalhamos com arranjos,
quando fazemos uma seleção de duas cores não estamos interessados na
ordem em que elas foram escolhidas.
Definição 3.29. Consideremos n objetos e p um inteiro positivo tal que
0 < p ≤ n. Uma combinação simples de classe p dos n objetos dados é uma
seleção de p objetos distintos dentre estes que diferem entre si apenas pela
natureza de cada um, isto é, o que importa ¡é ¢simplesmente quem partic-
ipa no grupo selecionado. Denotaremos por np o número de combinações
simples de classe p de n objetos.
Proposição 3.30. Seja n ≥ 1. O número total de combinações
¡ ¢ simples de
classe p de n objetos O = {o1 , o2 , . . . , on } é dado por np = p!(n−p)!
n!
.
Demonstração. Veremos a seguir que arranjos simples e combinações sim-
ples de classe p estão estreitamente relacionados. Com efeito, para cada
combinação simples formada por p objetos distintos de O podemos gerar to-
dos os arranjos simples de classe p formados por estes p objetos. Basta para
isto fazer todas as suas permutações possı́veis. Obtém-se assim p ! arranjos
simples diferentes com esses p objetos. Resumindo, para cada combinação
simples de classe p formada com p objetos diferentes de O podemos fazer
p ! arranjos simples diferentes de classe p com estes mesmos objetos; logo,
no total, teremos a seguinte relação:
µ ¶
n n!
p! = Apn = ,
p (n − p)!
de on segue-se que µ ¶
n n!
= .
p p!(n − p)!

Exemplo 3.31. De quantas formas podemos dividir um grupo 5 pessoas


em um grupo de duas e outro de três ?
¡¢
Solução. Temos 52 = 25!3! ! = 10 formas diferentes de escolher duas pessoas
do grupo. Por cada uma dessas escolhas o outro grupo de três pessoas é
automaticamente determinado; logo, temos 10 possibilidades diferentes de
fazer a divisão.

i i

i i
“Livro˙Olimpiada
i i
2006/5/27
page 93
i i

[SEC. 3.6: COMBINAÇÕES SIMPLES 93

Exemplo 3.32. De quantos modos podemos dividir 6 pessoas em:


(a) dois grupos de 3 pessoas cada?
(b) três grupos de 2 pessoas cada?
Solução.
¡ ¢ Começamos por (a). À primeira vista, parece que a resposta deve
ser n3 = 3!6!3! = 20, similarmente ao exemplo anterior. Porém, aqui há
um problema pois, devido ao fato de estarmos dividindo em grupos que
têm a mesma quantidade de pessoas, então as permutações de cada dois
grupos formados são consideradas divisões iguais; logo, devemos dividir o
resultado por 2 !, obtendo assim 10 formas diferentes de obter dois grupos
com 3 pessoas cada.
Para resolver o item (b) seguimos os seguintes passos:
• Primeiramente calcularemos o número de formas possı́veis para dividir
6 pessoas ¡em
¢ um grupo de 2 e outro grupo de 4; esta quantidade vem
dada por 62 = 4!6!2! .
• Agora dividiremos as 4 pessoas restantes em ¡ ¢um grupo de 2 e outro
grupo de 2; esta quantidade vem dada por 42 = 2!4!2! .
¡ ¢¡ ¢
Pelo princı́pio multiplicativo temos que existem 62 42 = (2!)6!
3 possibilidades

de dividir 6 pessoas em 3 grupos com duas pessoas cada. Igualmente ao


caso anterior, aqui as permutações possı́veis de cada 3 grupos formados
são consideradas iguais; logo, devemos dividir este último resultado por 3 !.
Portanto, existem 15 formas diferentes de dividir 6 pessoas em três grupos
de 2 pessoas cada.
Exemplo 3.33. Se você possui 10 amigos, de quantas maneiras você pode
jantar com 2 ou mais deles?
Solução. Esquematizamos a solução da seguinte maneira:
• Primeiramente, vamos encontrar a quantidade de ¡ maneiras
¢ pelas quais
você pode jantar com 2 amigos; isto é feito de 10
2 formas diferentes.
• Depois, vamos encontrar a quantidade de¡ maneiras
¢ pelas quais você
pode jantar com 3 amigos; isto é feito de 10
3 formas diferentes.

• Em seguida, encontramos a quantidade de¡ maneiras


¢ pelas quais você
pode jantar com 4 amigos; isto é feito de 10
4 formas diferentes.

i i

i i
“Livro˙Olimpiada
i i
2006/5/27
page 94
i i

94 [CAP. 3: CONTAGEM

• Em geral, o número de maneiras


¡ ¢ diferentes que você tem de jantar
com p amigos é dado por 10
p .

Pelo Princı́pio Aditivo, temos que a quantidade de formas diferentes que


você tem de jantar com 2 ou mais de seus amigos, é dada por:
µ ¶ µ ¶ µ ¶ µ ¶
10 10 10 10
+ + ··· + + = 1013.
2 3 9 10

Exemplo 3.34. De um grupo de 10 pessoas das quais 4 são mulheres,


quantas comissões de 5 pessoas podem ser formadas de modo que pelo menos
uma mulher faça parte?
Solução. Sendo que o grupo tem 10 pessoas e 4 destas são mulheres, segue-
se que no grupo temos 6 homens. Para formar um grupo de 5 pessoas com
pelo menos uma mulher, temos as seguintes alternativas:
• Nosso grupo é¡composto
¢¡ ¢ por uma mulher e 4 homens; neste caso poder-
emos formar 41 64 = 60 comissões de 5 pessoas.
• Nosso grupo é¡ composto
¢¡ ¢ por 2 mulheres e 3 homens; neste caso poder-
emos formar 42 63 = 120 comissões de 5 pessoas.
• Nosso grupo é¡ composto
¢¡ ¢ por 3 mulheres e 2 homens; neste caso poder-
emos formar 43 62 = 60 comissões de 5 pessoas.
• Nosso grupo é composto
¡ ¢¡ ¢ por 4 mulheres e um homem; neste caso
poderemos formar 44 61 = 6 comissões de 5 pessoas.
Pelo princı́pio aditivo temos que é possı́vel formar 246 comissões de 5 pessoas
de modo que pelo menos uma mulher faça parte.

3.7 Contagem e Probabilidades


Uma das aplicações interessantes da contagem de elementos de um conjunto
é quando desejamos estudar a probabilidade de eventos aleatórios. Por
exemplo, se lançarmos um dado de seis faces, temos os seguintes resultados
possı́veis:
Ω = {1, 2, . . . , 6}.

i i

i i
i i “Livro˙Olimpiada˙O
2006/5/27
page 95
i i

[SEC. 3.7: CONTAGEM E PROBABILIDADES 95

Se desejamos saber qual é a chance de que ocorra um número primo no


lançamento, devemos contar quantos primos aparecem em {1, 2, 3, 4, 5, 6}
e dividir por 6. Ou seja, a chance de ocorrer um número primo num
lançamento de um dado de seis faces é 3/6 = 0, 5.
Em geral, definimos então a probabilidade de um subconjunto A ⊂ Ω
como
|A|
p(A) = .
|Ω|
Também chamamos o subconjunto Ω de todos os resultados possı́veis de
espaço amostral e um subconjunto A de Ω de evento. Por exemplo, pode-
mos calcular a probabilidade de escolhermos um número par no conjunto
1, 2, 3, . . . , 15. Neste caso, o conjunto Ω está claro e é igual a Ω = {1, 2, 3, . . . , 15}.
O conjunto A é A = {2, 4, 6, . . . , 14}. Logo,

|A| 7
p(A) = = .
|Ω| 15

Assim, fica claro que a maior dificuldade para calcular a probabilidade


de um evento é contar quantos elementos pertencem a este evento e quantos
elementos pertencem ao espaço amostral. A seguir, veremos um exemplo
mais elaborado onde aplicamos a noção de combinações simples.

Exemplo 3.35. Vamos agora calcular a probabilidade de que escolhendo


um grupo de 44 pessoas, existam pelo menos duas que fazem aniversário no
mesmo dia do ano.
Podemos reescrever isso do seguinte modo: num saco existem bolas enu-
meradas com os números 1, 2, . . . , 365 (correspondentes aos dias do ano).
Retiramos a bola b1 e anotamos o número que apareceu. Devolvemos a
bola ao saco e efetuamos uma nova retirada, anotando novamente o número
que aparece. Repetindo este processo 44 vezes, obtemos uma lista com 44
números. Assim, a pergunta se transforma em : de quantos modos diferentes
podemos escolher 44 bolas enumeradas com os números 1, 2, 3, . . . , 365 com
reposição, tal que existam pelo menos duas bolas com o mesmo número?
A primeira coisa que devemos fazer é calcular o espaço amostral, de
todas as possibilidades possı́veis de resultado. Como escolhemos 44 bolas
enumeradas num saco, cada resultado possı́vel é uma lista (n1 , n2 , . . . , n44 )
com 44 números. Observe que, pelo princı́pio multiplicativo, |Ω| = 36544 ,
pois temos 365 opções para escolher n1 , 365 opções para escolher n2 , etc.

i i

i i
“Livro˙Olimpiada
i i
2006/5/27
page 96
i i

96 [CAP. 3: CONTAGEM

A segunda pergunta trata-se de saber quantos resultados são favoráveis,


ou seja, quantas são as escolhas tais que existam pelo menos duas bolas com
o mesmo número. Para isso é mais fácil contar quantas escolhas existem tais
que os 44 números são diferentes. Neste caso, devemos escolher 44 números
distintos entre¡ 365.¢ Como não importa a ordem em que eles aparecem,
são ao todo 365 44 grupos de 44 números. Logo, o número de resultados
favoráveis, isto¡ é, ¢onde existem pelo menos duas pessoas que nasceram no
mesmo dia é 365 44 de onde concluı́mos que a probabilidade de que este
evento ocorra é ¡365¢ ¡365¢
36544 − 44
p= = 1 − 4444 .
36544 365
Com ajuda de uma fórmula matemática chamada fórmula de Stirling e de uma
calculadora, obtemos que p é aproximadamente p ∼
= 0.80, como havı́amos prometido
no Capı́tulo 1.

3.8 Exercı́cios Propostos


1. De quantas maneiras podemos escolher três números distintos do con-
junto I50 = {1, 2, 3, . . . , 49, 50} de modo que sua soma seja
a) um múltiplo de 3?
b) um número par?
2. Considere o conjunto In = {1, 2, 3, . . . , n − 1, n}. Diga de quantos
modos é possı́vel formar subconjuntos de k elementos nos quais não
haja números consecutivos?
3. Considere as letras da palavra PERMUTA. Quantos anagramas de 4
letras podem ser formados, onde:
a) não há restrições quanto ao número de consoantes ou vogais?
b) o anagrama começa e termina por vogal?
c) a letra R aparece?
d) a letra T aparece e o anagrama termina por vogal?
4. Calcular a soma de todos os números de 5 algarismos distintos forma-
dos com os algarismos 1, 3, 5, 7 e 9.
5. Quantos números podem ser formados pela multiplicação de alguns
ou de todos os números 2, 2, 3, 3, 3, 5, 5, 6, 8, 9, 9?

i i

i i
“Livro˙Olimpiada
i i
2006/5/27
page 97
i i

[SEC. 3.8: EXERCÍCIOS PROPOSTOS 97

6. Dentre todos os números de sete dı́gitos, diga quantos possuem exata-


mente três dı́gitos 9 e os quatro dı́gitos restantes todos diferentes?
7. De quantas maneiras podemos distribuir 22 livros diferentes entre 5
alunos se 2 deles recebem 5 livros cada e os outros 3 recebem 4 livros
cada?
8. Quantos são os números naturais de sete dı́gitos nos quais o dı́gito 4
figura exatamente 3 vezes e o dı́gito 8 figura exatamente 2 vezes?
9. De quantas maneiras uma comissão de 4 pessoas pode ser formada,
de um grupo de 6 homens e 6 mulheres, se a mesma é composta de
um número maior de homens do que de mulheres?
10. O comprimento de uma palavra é a quantidade de caracteres que ela
possui. Encontre a quantidade de palavras de comprimento 5 que
podemos formar fazendo uso de 10 caracteres distintos, de forma que
não existam três caracteres consecutivos idênticos em cada palavra.
11. Quantos números inteiros existem entre 1 e 10.000 que não são di-
visı́veis por 3, 5 e 7?
12. Quantas são as permutações da palavra PROPOR nas quais não ex-
istem letras consecutivas iguais?
13. De quantos modos 6 casais podem sentar-se ao redor de uma mesa
circular de tal forma que marido e mulher não fiquem juntos?
14. Quantas são as permutações das letras da palavra BRASIL em que o
B ocupa o primeiro lugar, ou o R ocupa o segundo lugar, ou o L o
sexto lugar.
15. De quantas formas podemos representar o número 15 como soma de
vários números naturais?
16. Quantos quadrados perfeitos existem entre 40.000 e 640.000 que são
múltiplos simultaneamente de 3,4 e 5?
17. Oito amigos vão para o cinema assistir a um filme que custa um real.
Quatro deles possuem uma nota de um real e quatro possuem uma
nota de dois reais. Sabendo-se que o caixa do cinema não possui
nenhum dinheiro, como eles podem organizar uma fila para pagar o
filme permitindo o troco pelo caixa?

i i

i i
“Livro˙Olimpiada
i i
2006/5/27
page 98
i i

98 [CAP. 3: CONTAGEM

18. Se considerarmos todas as configurações do tabuleiro com duas torres


que não se atacam, como no Exemplo 3.2, sem distinguir as torres,
quantas configurações obteremos?

19. Continuando o problema anterior, generalize-o para 3, 4, 5, . . . torres


que não se atacam, encontrando também o número máximo de torres
que podem ser colocadas no tabuleiro de modo que duas delas não se
ataquem.

20. Tente fazer o problema anterior para cavalos de xadrez.

i i

i i
“Livro˙Olimpiada
i i
2006/5/27
page 99
i i

Capı́tulo 4

O Princı́pio da Casa dos


Pombos

Um importante instrumento para tratar problemas matemáticos relaciona-


dos à existência de elementos de conjuntos validando certas exigências é o
chamado princı́pio de Dirichlet, também conhecido como Princı́pio da Casa
dos Pombos (PCP). Este princı́pio foi usado por Dirichlet (1805-1859) para
resolver problemas na teoria dos números, entretanto ele possui um grande
número de aplicações em diversos ramos da matemática como combinatória
e geometria.
A versão mais simples do Princı́pio da Casa dos Pombos afirma o seguinte:

Se distribuı́mos N + 1 pombos em N casas, então alguma das


casas contém dois ou mais pombos.

A prova deste princı́pio é muito fácil e decorre de fazer uma simples


contagem dos pombos contidos em todas as casas depois de distribuı́dos.
Com efeito, suponhamos pelo contrário que em cada casa não existe mais do
que um pombo, então contando todos os pombos contidos nas N casas não
teremos mais do que N pombos, contradizendo isto a hipóteses de termos
N + 1 pombos distribuı́dos nas N casas (ver Figura 4.1)
Não é difı́cil detectar quando o princı́pio pode ser usado, mas a principal
dificuldade para aplicar o mesmo reside em identificar, em cada problema,
quem faz papel de pombos e quem faz papel de casas.

99

i i

i i
“Livro˙Olimpiada
i i
2006/5/27
page 100
i i

100 [CAP. 4: O PRINCÍPIO DA CASA DOS POMBOS

P1 P2 ········· PN

C1 C2 CN

PN +1

Figura 4.1: Em cada casa Cj , 1 ≤ j ≤ N , coloca-se um único pombo, denotado


por Pj . O pombo restante, denotado por PN +1 , deve ir para alguma das casas,
juntando-se ao que já se encontrava contido nela.

Nas seguintes seções discutiremos vários exemplos de diferentes naturezas,


onde o Princı́pio da Casa dos Pombos é aplicado com sucesso.

4.1 Primeiros Exemplos


Exemplo 4.1. Numa floresta crescem 1000 jaqueiras. É conhecido que uma
jaqueira não contém mais do que 600 frutos. Prove que existem 2 jaqueiras
na floresta que têm a mesma quantidade de frutos.

Solução. Temos 1000 jaqueiras, representando os pombos, e 601 casas iden-


tificadas pelos números 0, 1, 2, 3, · · · , 600. O número k associado a cada casa
significa que nela serão colocadas jaqueiras que têm exatamente k frutos.
Como 1000 > 602 = 601 + 1, o PCP nos garante que existem duas jaqueiras
com a mesma quantidade de frutos.

Exemplo 4.2. Em uma reunião há n pessoas. Mostre que existem duas
pessoas que conhecem exatamente o mesmo número de pessoas.

i i

i i
“Livro˙Olimpiada
i i
2006/5/27
page 101
i i

[SEC. 4.1: PRIMEIROS EXEMPLOS 101

Solução. Os pombos neste caso são as n pessoas. As casas são enumeradas


com os números 0, 1, 2, · · · , n − 1, indicando estes que na mesma serão colo-
cadas pessoas que têm essa quantidade de conhecidos. Notemos que uma
das casas enumeradas com 0 ou n − 1 permanece desocupada, pois a possi-
bilidade de conhecer 0 e n − 1 pessoas não acontece simultaneamente. Logo,
nas n − 1 casas restantes haverá uma ocupada por dois ou mais pombos,
depois de serem distribuı́dos. Portanto, existem no mı́nimo duas pessoas
com o mesmo número de conhecidos.

Exemplo 4.3. Dados 8 números inteiros mostre que existem dois deles cuja
diferença é divisı́vel por 7.

Solução. Consideramos os 8 números como sendo os pombos e as casas como


sendo os 7 possı́veis restos na divisão por 7. Como temos 8=7+1 números
o PCP nos diz que existem dois números dentro dos 8 dados que têm o
mesmo resto quando divididos por 7. Finalmente, observamos que se dois
números deixam o mesmo resto na divisão por 7 então a diferença entre eles
é divisı́vel por 7.

Uma forma alternativa e muito útil na qual pode-se apresentar o Princı́pio


da Casa dos Pombos é a seguinte:

Se a soma de n números naturais é igual S, então existe pelo


menos um deles que não é maior que S/n, assim como existe
pelo menos um deles que não é menor que S/n.

Exemplo 4.4. Numa familia formada por 5 pessoas a soma das idades é
de 245 anos. Prove que podem ser selecionados 3 membros da familia cuja
soma das idades não é menor que 147.
¡¢
Solução. Temos um total de 53 = 3!2! 5!
= 10 trios diferentes formados por
¡¢
membros da familia. Além disso, cada pessoa aparece exatamente em 42 =
4!
2!2! = 6 trios. Então, denotando por Ej a soma das idades dos membros de
cada trio Tj , j = 1, 2 · · · 10, temos que

E1 + E2 + · · · + E10 = 6 · 245 = 1470;


1470
conseqüentemente existe algum trio Tj ∗ tal que Ej ∗ ≥ 10 = 147.

i i

i i
“Livro˙Olimpiada
i i
2006/5/27
page 102
i i

102 [CAP. 4: O PRINCÍPIO DA CASA DOS POMBOS

4.2 Uma Versão mais Geral


A seguinte versão mais geral do PCP é bastante útil na resolução de alguns
problemas. Ela afirma o seguinte:

Se distribuı́mos N k + 1 pombos em N casas, então alguma das


casas contém pelo menos k + 1 pombos.

A prova deste enunciado mais geral é similar à anterior. Com efeito,


suponhamos pelo contrário que em cada casa não existe mais do que k
pombos, então contando todos os pombos contidos nas N casas não teremos
mais do que N k pombos, contradizendo isto a hipóteses de termos N k + 1
pombos distribuı́dos nas N casas.
Notemos que se k = 1 esta versão mais geral coincide com a versão mais
simples.
Exemplo 4.5. Num colégio com 16 salas são distribuı́das canetas nas cores
preta, azul e vermelha para realizar uma prova de concurso. Se cada sala
recebe canetas da mesma cor então prove que existem pelo menos 6 salas
que receberam canetas da mesma cor.
Solução. Fazendo a divisão com resto de 16 por 3 temos que 16 = 3 · 5 + 1.
Consideramos as 16 salas como sendo os pombos e as três cores, preto, azul
e vermelho como sendo as casas. Logo, podemos “colocar”cada sala em
uma das três cores. Assim, o PCP com N = 3 e k = 5 nos dá que existe
uma casa com pelo menos 6 pombos, ou seja, existem no mı́nimo 6 salas
que receberam canetas da mesma cor.
Exemplo 4.6. Uma equipe formada por seis alunos de matemática é se-
lecionada para representar o Brasil numa olimpı́ada internacional. Mostre
que necessariamente existem três deles que se conhecem mutuamente, ou
três deles que não se conhecem mutuamente.
Solução. Resolveremos o problema com o auxı́lio da Figura 4.2. Cada aluno
Aj , com j = 1, 2, · · · , 6, é representado por um dos vértices de um hexágono
regular. Quando dois alunos se conhecem traçamos o segmento de reta que
liga os vértices correspondentes na cor azul; caso contrário traçamos este
segmento na cor vermelha. Logo, usando este esquema, o problema equivale
a provar que sempre existe um triângulo azul ou um triângulo vermelho com
vértices no conjunto A = {A1 , A2 , · · · , A6 }.

i i

i i
“Livro˙Olimpiada
i i
2006/5/27
page 103
i i

[SEC. 4.3: APLICAÇÕES NA TEORIA DOS NÚMEROS 103

Temos 5 segmentos (pombos) incidindo no vértice A1 , cada um deles


pintado de azul ou de vermelho (estas duas cores são consideradas como as
casas). Como 5 = 2 · 2 + 1, pelo PCP temos que 3 dos 5 segmentos são
azuis ou vermelhos. Suponhamos que 3 são azuis (caso contrário o argu-
mento é similar) e denotemos estes por A1 A3 , A1 A4 e A1 A6 (ver Figura
4.2). Se algum dos segmentos A3 A4 , A3 A6 ou A4 A6 for azul então este seg-
mento junto aos que se ligam com A1 formam um triângulo azul. Por outro
lado, se nenhum deles for azul, então eles formam um triângulo vermelho,
completando isto a demonstração.
A3 A2

A4 A1

A5 A6
Figura 4.2: O triângulo A1 A2 A5 indica que os alunos A1 , A2 e A5 não se con-
hecem mutuamente e o triângulo A1 A4 A6 indica que os alunos A1 , A4 e A6 se
conhecem mutuamente.

4.3 Aplicações na Teoria dos Números


Nesta seção apresentamos alguns exemplos de aplicações do PCP na Teoria
dos Números. A primeira delas é:
Exemplo 4.7. Se n e m são números naturais, então o conjunto A =
{m + 1, m + 2, . . . , m + n} possui algum divisor de n.
Solução. Temos n números diferentes no conjunto acima. Vamos utilizar
o método de redução ao absurdo. Se não existisse nenhum múltiplo de n,
quando dividı́ssemos os números do conjunto A por n, os restos pertence-
riam ao conjunto B = {1, 2, . . . , n − 1}, que possui n − 1 elementos. Logo,

i i

i i
“Livro˙Olimpiada
i i
2006/5/27
page 104
i i

104 [CAP. 4: O PRINCÍPIO DA CASA DOS POMBOS

devem existir dois números m+i e m+j, com 1 ≤ i < j ≤ n tais que o resto
da divisão de m + i por n é o mesmo que o resto da divisão de m + j por n.
Logo, m + j − (m + i) é um múltiplo de n, o que implica que n > j − i ≥ 1
é múltiplo de n menor que n (absurdo!). Logo, deve existir algum múltiplo
de n no conjunto A.

Como conseqüência desse exemplo, podemos resolver o seguinte prob-


lema:

Exemplo 4.8. Demonstrar que todo inteiro tem um múltiplo cuja repre-
sentação decimal começa com o bloco de dı́gitos 1234567890.

Solução. Se m e n são inteiros positivos, pelo exemplo anterior um dos


número m + 1, m + 2, . . . , m + n é múltiplo de n. Assim, Dado n um inteiro
qualquer, escolhe-se m = 1234567890×10n+1 . Deste modo, todos os inteiros
m + 1, m + 2, . . . , m + n começam com 1234567890 e algum deles é múltiplo
de n.

Exemplo 4.9. Dado um número inteiro positivo n, mostre que existe um


múltiplo de n que se escreve com os algarismos 0 e 1 apenas. (Por exemplo,
se n = 3, temos 111 ou 1101, etc...)

Solução. Consideramos os n + 1 números

1, 11, 111, 1111, · · · , |111{z


· · · 1} , (4.1)
n+1−vezes

como sendo os pombos e n casas enumeradas com os números

0, 1, 2, 3, · · · , n − 1,

ou seja, com os possı́veis restos na divisão por n. Similarmente ao exemplo


anterior existem dois números na lista (4.1) que deixam o mesmo resto na
divisão por n e, portanto, a diferença entre o maior e o menor é múltiplo
de n. Obviamente a diferença entre dois números quaisquer da lista (4.1)
resulta em um número formado apenas pelos algarismos 0 e 1.

Exemplo 4.10. Prove que entre n + 1 elementos escolhidos no conjunto


{1, 2, 3, · · · 2n} existem dois que são primos relativos.

i i

i i
“Livro˙Olimpiada
i i
2006/5/27
page 105
i i

[SEC. 4.4: APLICAÇÕES GEOMÉTRICAS 105

Solução. A escolha das casas e dos pombos neste exemplo não é tão óbvia.
Os pombos representam os n+1 números escolhidos do conjunto {1, 2, · · · 2n}
e as casas são escolhidas como sendo os n conjuntos:

Cj = {2j − 1, 2j}, 1 ≤ j ≤ n.

Logo, pelo PCP, quando distribuı́mos os n + 1 números nos n conjuntos


Cj , 1 ≤ j ≤ n, dois deles ficarão juntos em algum conjunto Cj , ou seja, estes
números serão consecutivos e portanto primos entre si.
Finalizaremos esta seção com uma outra prova do Teorema de Bézout:
Exemplo 4.11. Seja d = (a, b) o m.d.c. entre os números naturais a e b.
Então, existem x e y números inteiros tais que

ax + by = d.

Solução. Denotando por m = a/d e n = b/d, podemos supor que a e b são


primos entre si. Realmente, se podemos escrever

mx + ny = 1

então, substituindo os valores de m e n na equação acima, temos que ax +


by = d.
Se (a, b) = 1, considere a sequência A = {a, 2a, . . . , ba}. Afirmamos que
existe algum número no conjunto A que deixa resto 1 quando dividido por
b. De fato, se isso não ocorresse, terı́amos b números em A deixando no
máximo b − 1 restos diferentes quando divididos por b. Logo, pelo PCP,
dois deles, digamos ia e ja com b > j > i ≤ 1, devem deixar o mesmo resto
quando divididos por b. assim, (j − i)a é divisı́vel por b. Como estamos
supondo que (a, b) = 1, temos que b deve dividir j − i > 0. Como b > j − i,
temos um absurdo.
Assim, algum dos números em a deixa resto 1 quando divididos por
b. Digamos que esse número seja ax. Logo, ax − 1 é múltiplo de b, onde
ax − 1 = by, o que encerra nossa prova.

4.4 Aplicações Geométricas


Na geometria também encontramos belas aplicações do PCP. Vejamos os
problemas a seguir para constatar isto.

i i

i i
“Livro˙Olimpiada
i i
2006/5/27
page 106
i i

106 [CAP. 4: O PRINCÍPIO DA CASA DOS POMBOS

Exemplo 4.12. Mostre que se tomamos cinco pontos quaisquer sobre √ um


quadrado de lado 1, então pelo menos dois deles distam menos que 2/2.

Solução. Vamos dividir o quadrado em quatro quadradinhos de lado 1/2,


como mostra a Figura 4.3




1
• •

Figura 4.3::

Logo, pelo PCP pelo menos dois deles devem estar no mesmo quadrad-
inho, uma vez que temos 4 quadradinhos e 5 pontos. Logo, como a maior
distância num quadrado é a diagonal, o teorema de√pitágoras nos garante
que a distância desses dois pontos é no máximo 2/2, como querı́amos
mostrar.

Exemplo 4.13. Qual é o maior número de quadradinhos de um tabuleiro


de 8 × 8 que podem ser pintados de preto, de forma tal que qualquer arranjo
de três quadradinhos como mostra a Figura 4.4 tenha pelo menos um dos
quadradinhos não pintado de preto?

Figura 4.4::

Solução. Primeiramente, pintamos o tabuleiro de 8×8 como um tabuleiro de


jogar xadrez, ou seja, 32 quadradinhos pintados de branco e 32 quadradinhos
pintados de preto (ver Figura 4.5).
Notemos que uma vez pintado o tabuleiro desta forma é satisfeita a
exigência do problema, pois nunca temos 2 quadradinhos vizinhos (quadrad-
inhos com um lado comum) pintados de preto.

i i

i i
“Livro˙Olimpiada
i i
2006/5/27
page 107
i i

[SEC. 4.4: APLICAÇÕES GEOMÉTRICAS 107

Figura 4.5::

Mostraremos agora que se pintamos 33 quadradinhos de preto então a


condição exigida no problema falha. De fato, se dividimos o tabuleiro em 16
quadrados de 2 × 2 (casas) e pintamos 33 quadradinhos de preto (pombos);
então, como 33 = 16 · 2 + 1, pela versão geral do PCP um dos 16 quadrados
de 2 × 2 contém 3 quadradinhos pintados de preto. Portanto, este último
contém um arranjo como na Figura 4.5 completamente pintado de preto.
Resumindo, o número máximo de quadradinhos que podemos pintar de
preto é 32.
Exemplo 4.14. Na região delimitada por um triângulo eqüilátero de lado
4 são marcados 10 pontos. Prove que existe√ ao menos um par destes pontos
cuja distância entre eles não e maior que 3.
Solução. Dividimos o triângulo equilátero de lado 4 em 16 triângulos equiláteros
menores de lado 1, conforme a Figura 4.6.
Agora pintamos os triângulos nas cores verde e cinza de maneira que dois
triângulos vizinhos, isto é, com um lado comum, são pintados de cores difer-
entes. Se tivéssemos dois pontos no mesmo triângulo a distância máxima
possı́vel entre eles seria 1 e o problema estaria resolvido. Se tivéssemos
pontos em triângulos vizinhos, a maior distância possı́vel entre eles seria

3 e também isto resolveria o problema. Se não tivéssemos nenhum dos
casos anteriores, não seria difı́cil ver que os 10 pontos deveriam estar situa-
dos sobre os 10 triângulos verdes, contendo cada triângulo exatamente um
ponto. Dividendo o triângulo DBE em 4 triângulos iguais de lado 3/2 pelo

i i

i i
“Livro˙Olimpiada
i i
2006/5/27
page 108
i i

108 [CAP. 4: O PRINCÍPIO DA CASA DOS POMBOS

C

E
• •


• •
• •
• •
A D B
Figura 4.6::

PCP temos que pelo menos dois dos 6 pontos contidos em DBE estão num √
destes 4 triângulos, logo a distância entre eles não é maior que 3/2 < 3.
Com isto terminamos nossa prova.

4.5 Miscelânea
Exemplo 4.15. Em cada quadradinho de um tabuleiro 3 × 3 é colocado
um dos números: −1, 0 ou 1. Prove que entre todas as somas das linhas,
colunas e diagonais do tabuleiro há duas que são iguais. Por exemplo, no
tabuleiro abaixo a soma da segunda linha é 2, que coincide com a soma da
terceira coluna.

-1 -1 1
1 0 1
0 -1 0

Solução. Seja S = a1 + a2 + a3 , onde cada a1 , a2 e a3 podem tomar val-


ores: −1, 0 e 1. Então, temos 7 valores possı́veis para S (casas), que são:
−3, −2, −1, 0, 1, 2, 3.
O tabuleiro 3 × 3 tem 3 linhas, 3 colunas e 2 diagonais, portanto, ao
somarmos os elementos de cada uma das linhas, colunas e diagonais, obter-
emos 8 números (pombos). Como existem somente 7 valores possı́veis para
estes números, pelo PCP pelo menos dois deles devem ser iguais.

i i

i i
“Livro˙Olimpiada
i i
2006/5/27
page 109
i i

[SEC. 4.5: MISCELÂNEA 109

Exemplo 4.16. Dado qualquer conjunto A formado por 10 números nat-


urais escolhidos entre 1 e 99, inclusos, demonstre que existem dois sub-
conjuntos disjuntos e não-vazios de A tal que a soma dos seus respectivos
elementos é igual.

Solução: É conhecido que A tem 210 − 1 = 1023 subconjuntos não-vazios


diferentes. A soma dos elementos de cada um deles dá uma quantidade
menor do que 1000, pois o subconjunto com no máximo 10 elementos de
maior soma possı́vel é o formado por 90, 91, · · · , 99, e nesse caso 90 +
91 + · · · + 99 = 945. Agora consideramos os pombos como sendo os 1023
subconjuntos distintos de A e as casas como sendo as somas possı́veis dos
elementos de cada um dos conjuntos. Logo, como o número de conjuntos
é maior que o número de somas possı́veis, devem existir dois conjuntos B
e C de A, de tal modo que a soma dos elementos de B é igual à soma dos
elementos de C. Se B e C são disjuntos, acabou a prova. Se não, considere
D = B − B ∩ C e E = C − B ∩ C. Logo, os conjuntos D e E são disjuntos e
a soma dos seus elementos é a mesma, pois retiramos de ambos a mesma
quantidade.

Exemplo 4.17. Dados sete números reais arbitrários, demonstre que exis-
tem dois deles, digamos x e y, tais que

x−y 1
0≤ ≤√
1 + xy 3
x−y
Solução. Primeiramente observamos que a expressão 1+xy nos faz pensar
na fórmula
tan α − tan β
tan(α − β) = . (4.2)
1 + tan α tan β
Sejam x1 , x2 , · · · , x7 os sete números selecionados arbitrariamente. Lem-
bramos que a função tangente é uma bijeção entre o intervalo (− π2 , π2 ) e os
números reais R, logo para cada xi , 1 ≤ i ≤ 7, existe um αi ∈ (− π2 , π2 ) tal
que tan(αi ) = xi . Dividimos o intervalo (− π2 , π2 ) em seis subintervalos de
comprimento π6 , como mostra o desenho abaixo.

αi1 αi2

− π2 π
π
2
6

i i

i i
“Livro˙Olimpiada
i i
2006/5/27
page 110
i i

110 [CAP. 4: O PRINCÍPIO DA CASA DOS POMBOS

Pelo PCP dois dos números αi pertencem ao mesmo subintervalo. De-


notemos os mesmos por αi1 e αi2 e suponhamos, sem perda de generalidade,
que αi1 ≤ αi2 . Então vale
π
0 ≤ αi2 − αi1 ≤ .
6
Usando o fato de que a tangente é uma função crescente e a fórmula (4.2)
temos que
π
tan(0) ≤ tan(αi2 − αi1 ) ≤ tan( ).
6
Equivalentemente,
xi2 − xi1 1
0≤ ≤√ .
1 + xi2 xi1 3

4.6 Exercı́cios
1. Seja x um número real arbitrário. Prove que entre os números

x, 2x, 3x, · · · , 101x

existe um tal que sua diferença com certo número inteiro é menor
0,011.

2. Mostre que entre nove números que não possuem divisores primos
maiores que cinco, existem dois cujo produto é um quadrado.

3. Um disco fechado de raio um contém sete pontos, cujas distâncias


entre quaisquer dois deles é maior o igual que um. Prove que o centro
do disco é um destes pontos.

4. Na região delimitada por um retângulo de largura quatro e altura três


são marcados seis pontos. Prove que existe ao menos
√ um par destes
pontos cuja distância entre eles não e maior que 5.
5. Seja a um número irracional. Prove que existem infinitos números
racionais r = p/q tais que |a − r| < 1/q 2 .

i i

i i
“Livro˙Olimpiada
i i
2006/5/27
page 111
i i

Capı́tulo 5

Indução Matemática

Imagine uma fila com infinitos dominós, um atrás do outro. Suponha que
eles estejam de tal modo distribuı́dos que, uma vez que um dominó caia,
o seu sucessor na fila também caia. O que acontece quando derrubamos o
primeiro dominó?
Apesar da simplicidade da pergunta acima ela trás em sua essência toda
a idéia usada no Método da Indução Matemática. Esse método é uma das
grandes armas do matemático moderno e tem utilidade na solução de vários
problemas, como iremos ver ao longo deste capı́tulo.

5.1 Formulação Matemática


No inı́cio do século XX o matemático Giuseppe Peano (1858-1932) estabele-
ceu os axiomas necessários que nos permitem hoje descrever com precisão
o conjunto dos números naturais denotado por N = {1, 2, 3, · · · }. O último
dos seus axiomas diz o seguinte: Seja A um subconjunto de N (A ⊂ N). Se
1 ∈ A e se, além disso, A contém todos os sucessores dos seus elementos,
então A = N.
Este axioma é conhecido como axioma de indução e serve como base
do método de demonstração por indução matemática, o qual é de grande
utilidade para estabelecer provas rigorosas em matemática.

111

i i

i i
“Livro˙Olimpiada
i i
2006/5/27
page 112
i i

112 [CAP. 5: INDUÇÃO MATEMÁTICA

5.1.1 Método de Indução Matemática


Neste capı́tulo, não assumiremos o axioma da indução. Ao invés disso, ire-
mos utilizar o Princı́pio da Boa Ordenação, enunciado no inı́cio do Capı́tulo 2,
para mostrar que o Método da Indução Finita é verdadeiro. O motivo de
fazer isto é mostrar ao leitor que os axiomas da boa ordem e de indução não
são independentes e sem nenhuma conexão. De fato, eles são equivalentes,
ou seja, se consideramos o axioma da boa ordem como sendo um postulado
podemos deduzir o axioma de indução e, reciprocamente, se consideramos o
axioma de indução como sendo um postulado podemos deduzir o axioma da
boa ordem. Aqui, P (n) representa a propriedade P em relação ao natural
n, podendo esta se verificar ou não.
Teorema 5.1 (Método de Indução Finita). Considere n0 um inteiro
não-negativo. Suponhamos que, para cada inteiro n ≥ n0 seja dada uma
proposição P (n). Se podemos provar, simultaneamente, as duas seguintes
propriedades:
(a) A proposição P (n0 ) é verdadeira;
(b) Se P (n) é verdadeira então P (n + 1) também é verdadeira, para todo
n ≥ n0 ;
então P (n) é verdadeira qualquer que seja n ≥ n0 .
A afirmação (a) é chamada de base da indução e a (b) de passo indutivo.
Demonstração. Definamos o conjunto
V := {k inteiros não-negativos; k ≥ n0 e P (k) é verdadeira}
Notemos que V é não-vazio, pois a condição (a) nos assegura que n0 ∈ V.
A prova do teorema é equivalente a mostrarmos que
V = {n0 , n0 + 1, n0 + 2, n0 + 3, · · · },
ou equivalentemente, a provarmos que o conjunto
F := {k inteiros não-negativos; k ≥ n0 e P (k) é falsa}
é vazio.
Provaremos que F é vazio por contradição; para isto suponhamos que
é não-vazio. Pelo axioma da boa ordem segue-se que existe um menor
elemento k0 ∈ F, onde P (k0 ) é falso. Observemos que,

i i

i i
“Livro˙Olimpiada
i i
2006/5/27
page 113
i i

[SEC. 5.2: PROBLEMAS CLÁSSICOS 113

• k0 ≥ n0 +1. De fato, k0 ≥ n0 , porém a possibilidade k0 = n0 contradiz


a condição (a);

• k0 − 1 ∈ V. Com efeito, P (k0 − 1) é verdadeira pois, caso contrário,


k0 −1 ∈ F e, além disto, k0 −1 < k0 , contradizendo isto a minimalidade
de k0 .

Finalmente, como P (k0 − 1) é verdadeira, segue da condição (b) que P (k0 )


também é verdadeira, o que é impossı́vel pela definição de k0 . Portanto, o
conjunto F é vazio, concluindo assim a prova.

O grande trunfo do método da indução é poder provar que uma quan-


tidade infinita de afirmações é verdadeira, simplesmente verificando que
uma quantidade finita destas afirmações é verdadeira. Vamos deixar clara
a utilidade deste método resolvendo alguns problemas.

5.2 Problemas Clássicos


Muitas descobertas em matemática são feitas baseadas na realização de
testes que nos fornecem evidências empı́ricas. Tais evidências são estu-
dadas para efetivamente verificarmos se os resultados que elas insinuam são
verdadeiros. O método de indução finita constitui uma ferramenta muito
útil na hora de desvendar a veracidade de resultados provenientes deste tipo
de estudo.
Dentro da grande gama de problemas que podem ser abordados apli-
cando o método de indução podemos distinguir três importantes grupos:

⋄ Demonstração de identidades;

⋄ Demonstração de desigualdades;

⋄ Demonstração de problemas de divisibilidade.

A seguir damos vários exemplos de como aplicar o método em problemas


referentes a cada um destes grupos.

5.2.1 Demonstrando Identidades


Começamos com os seguintes problemas clássicos:

i i

i i
“Livro˙Olimpiada
i i
2006/5/27
page 114
i i

114 [CAP. 5: INDUÇÃO MATEMÁTICA

(P1) Determinar uma fórmula para a soma dos n primeiros números pares,
isto é,
Sp (n) := 2 + 4 + 6 + · · · + 2n.

(P2) Determinar uma fórmula para a soma dos n primeiros números ı́mpares,
isto é,
Si (n) := 1 + 3 + 5 + · · · + 2n − 1.

Para induzir ambas as fórmulas, primeiro fazemos os cálculos para vários


valores de n, os quais apresentamos na seguinte tabela.

n 1 2 3 4 5 ···
Sp (n) 2=1·2 6=2·3 12 = 3 · 4 20 = 4 · 5 30 = 5 · 6 ···
Si (n) 1 = 12 4 = 22 9 = 32 16 = 42 25 = 52 ···

Os resultados na tabela sugerem que Sp (n) = n(n + 1) e que Si (n) =


n2 . Entretanto, isto não constitui por si só uma prova rigorosa destas
fórmulas, pois para poder garantir a veracidade das mesmas utilizando a
tabela terı́amos que verificar cada valor de n natural, sendo isto impossı́vel.
Provaremos agora que, de fato, as fórmulas induzidas são válidas usando o
método da indução finita.

Exemplo 5.2. Demonstre que para qualquer n ∈ N, é valida a igualdade:

2 + · · · + 2n = n(n + 1).

Solução. Definamos a proposição

P (n) : 2 + · · · + 2n = n(n + 1)

e observemos que a mesma vale para n = 1 (base da indução), de fato

P (1) : 2 = 1(1 + 1).

Agora partimos para a prova do passo indutivo:

• Hipótese: Suponhamos que P (k) é verdadeira para um certo k >


1, k ∈ N.

• Tese: Devemos mostrar que P (k + 1) também é verdadeira.

i i

i i
“Livro˙Olimpiada
i i
2006/5/27
page 115
i i

[SEC. 5.2: PROBLEMAS CLÁSSICOS 115

Com efeito, como

2 + · · · + 2k = k(k + 1),
somando 2(k + 1) a ambos lados desta igualdade, temos que

2 + · · · + 2k + 2(k + 1) = k(k + 1) + 2(k + 1)


= (k + 2)(k + 1).

Esta última igualdade afirma que P (k + 1) também é verdadeira.


O Método da Indução nos garante que P (n) é verdadeira para qualquer
n ∈ N.
Exemplo 5.3. Demonstre que para qualquer n ∈ N, é válida a igualdade:

1 + 3 + 5 + · · · + 2n − 1 = n2 .

Solução. Aqui definimos a proposição:

P (n) : 1 + 3 + 5 + · · · + 2n − 1 = n2

e notamos que a mesma é válida se tomarmos, por exemplo, n = 1 e n = 2.


De fato,

P (1) : 1 = 2 · 1 − 1 e P (2) : 1 + 3 = 1 + 2 · 2 − 1 = 4 = 22 .

Agora só resta provar o passo indutivo:


• Hipótese: Suponhamos que P (k) seja verdadeira para um certo k >
1, k ∈ N.
• Tese: Devemos mostrar que P (k + 1) também é verdadeira.
Com efeito, como
1 + 3 + 5 + · · · + 2k − 1 = k 2 ,
somando 2k + 1 a ambos lados desta igualdade, temos que

1 + 3 + 5 + · · · + 2k − 1 + 2k + 1 = k 2 + 2k + 1
= (k + 1)2 .

O Princı́pio de Indução nos garante que P (n) é verdadeira para qualquer


n ∈ N.

i i

i i
i i “Livro˙Olimpiada˙O
2006/5/27
page 116
i i

116 [CAP. 5: INDUÇÃO MATEMÁTICA

Uma conseqüência imediata do exemplo 5.2 é a fórmula para a soma dos


n primeiros números naturais, dada por

n(n + 1)
S(n) = 1 + 2 + 3 + · · · + n = . (5.1)
2
Com efeito, como
2 + 4 + · · · + 2n = n(n + 1),
então dividindo por 2 ambos os membros da igualdade acima, obtemos a
equação 5.1.
Continuando com o mesmo raciocı́nio, é natural nos perguntarmos se é
possı́vel obter uma fórmula para a soma dos n primeiros quadrados perfeitos,
ou seja, determinar Q(n) onde:

Q(n) := 12 + 22 + 32 + · · · + n2 .

Para induzir a fórmula, consideramos os valores de S(n) e Q(n) numa tabela:

n 1 2 3 4 5 6 ···
S(n) 1 3 6 10 15 21 ···
Q(n) 1 5 14 30 55 91 ···

Aparentemente são existe nenhuma relação entre S(n) e Q(n). Mas, se


considerarmos o quociente Q(n)/S(n) vejamos o que acontece:

n 1 2 3 4 5 6 ···
Q(n)/S(n) 3/3 5/3 7/3 9/3 11/3 13/3 ···

Isso nos sugere que vale a relação

Q(n) 2n + 1
= ,
S(n) 3

logo nosso candidato para valor de Q(n) é

S(n)(2n + 1) n(n + 1)(2n + 1)


Q(n) = = .
3 6
Convidamos o leitor a provar a veracidade da equação acima utilizando o método
da indução.

i i

i i
i i “Livro˙Olimpiada˙O
2006/5/27
page 117
i i

[SEC. 5.2: PROBLEMAS CLÁSSICOS 117

5.2.2 Demonstrando Desigualdades


Apresentamos agora alguns exemplos de como usar indução para provar desigual-
dades.
2
Exemplo 5.4. Prove que 3n−1 < 2n para todo n ∈ N.
2
Solução. Denotamos por P (n) a propriedade: 3n−1 < 2n . É claro que P (1) é
valida, pois 1 < 2. Agora supondo que P (n) é verdadeira temos que
2 2
3n = 3n−1 · 3 < 2n · 22n+1 = 2(n+1) ,

logo P (n + 1) também vale. Observamos que na desigualdade acima usamos o


fato de que 3 < 22n+1 para qualquer n ∈ N.

No mesmo espı́rito do problema anterior damos o seguinte exemplo:

Exemplo 5.5. Mostre que para todo número n ∈ N maior que 3 vale 2n < n!.

Demonstração. Para n = 4 a desigualdade é verificada, pois 24 = 16 < 4! = 24.


Vamos assumir como hipótese de indução que a desigualdade é válida para n ≥ 4.
Então, precisamos mostrar que a mesma vale também para n + 1. De fato, por
hipótese de indução:
2n < n! (5.2)
Como 2 < n + 1, podemos multiplicar o lado esquerdo da desigualdade em 5.2 por
2 e o lado direito por n + 1, sem alterar o sinal de desigualdade. Logo, temos que:

2n .2 = 2n+1 < n!(n + 1) = (n + 1)!,

concluindo nossa demonstração.

5.2.3 Indução em problemas de divisibilidade


Agora damos alguns exemplos de problemas de divisibilidade que podem ser
mostrados utilizando o Método da Indução:
n
X
Exemplo 5.6. Mostre que j ! é um número ı́mpar.
j=1

1
X
Solução. A afirmação é verdadeira para n = 1, pois j ! = 1, que obviamente
j=1
é ı́mpar. Partimos agora para a prova do passo indutivo:

i i

i i
i i “Livro˙Olimpiada˙O
2006/5/27
page 118
i i

118 [CAP. 5: INDUÇÃO MATEMÁTICA

k
X
• Hipótese: j ! é um número ı́mpar para k > 1.
j=1
k+1
X
• Tese: j ! é um número ı́mpar.
j=1

Com efeito,
k+1
X k
X
j!= j ! + (k + 1) !
j=1 j=1

Sendo k > 1 afirmamos que (k + 1) ! é um número par, pois, (k + 1) ! = 2 ·


[3 · · · · k · (k + 1)]. Portanto,
k+1
X
j ! = número ı́mpar + número par
j=1

= número ı́mpar.

Exemplo 5.7. Mostre que para qualquer n ∈ N, n3 + 2n é sempre divisı́vel por


3.
Solução. Para n = 1 a afirmação é válida, pois 13 + 2 · 1 = 3, que obviamente é
divisı́vel por 3.
Assumamos como hipótese indutiva que a afirmação vale para algum k ∈ N,
isto é,
Hipótese: k3 + 2k é divisı́vel por3.
Devemos mostrar que a afirmação também é verdadeira para k + 1, ou seja,
temos que provar que
Tese: (k + 1)3 + 2(k + 1) é divisı́vel por3.
Para provar isto último, usamos o fato de que
(k + 1)3 + 2(k + 1) = (k3 + 3k2 + 3k + 1) + (2k + 2);

agrupando adequadamente,

(k + 1)3 + 2(k + 1) = (k3 + 2k) + (3k2 + 3k + 3)


= (k3 + 2k) + 3(k2 + k + 1)
| {z } | {z }
múltiplo de 3 múltiplo de 3

= múltiplo de 3,

i i

i i
i i “Livro˙Olimpiada˙O
2006/5/27
page 119
i i

[SEC. 5.3: INDUÇÃO NA GEOMETRIA 119

concluindo assim a prova.

Exemplo 5.8. Mostre que a soma dos cubos de três números naturais consecu-
tivos é divisı́vel por 9.

Solução. Definamos a seguinte proposição:

p(n) : n3 + (n + 1)3 + (n + 2)3 é um múltiplo de nove.

Notemos que P (1) é válida, pois

13 + 23 + 33 = 1 + 8 + 27 = 36 = 9 · 4.

Precisamos provar agora o passo indutivo, isto é,


• Hipótese: P (k) é verdadeira para algum k ∈ N.
• Tese: P (k + 1) também é verdadeira.
Para provar isto, observamos que

(k + 1)3 + (k + 2)3 + (k + 3)3 = (k + 1)3 + (k + 2)3 + (k3 + 9k2 + 27k + 27) .


| {z }
α

Ordenando adequadamente o lado direito da última igualdade, temos que

α = k3 + (k + 1)3 + (k + 2)3 + (9k2 + 27k + 27)


= k3 + (k + 1)3 + (k + 2)3 + 9(k2 + 3k + 3)
| {z } | {z }
múltiplo de 9 múltiplo de 9

= múltiplo de 9,

completando assim nossa demonstração.

5.3 Indução na Geometria


Tratamos aqui alguns exemplos que mostram a utilidade do Método de Indução
na resolução de problemas geométricos.
Vamos começar estudando duas propriedades importantes dos polı́gonos. A
primeira delas trata da soma dos ângulos internos de um polı́gono de n lados
(n-ágono).

Exemplo 5.9. Mostre que a soma dos ângulos internos de um polı́gono de n


lados (n ≥ 3) é igual a (n − 2)π radianos.

i i

i i
i i “Livro˙Olimpiada˙O
2006/5/27
page 120
i i

120 [CAP. 5: INDUÇÃO MATEMÁTICA

A4 A4
A3

A5 A3

A1 A2 A1 A2
(a) (b)

Figura 5.1::

Solução. No caso de n = 3 a propriedade acima é muito bem conhecida. Desde


Tales de Mileto e Euclides se conhecia que a soma dos ângulos internos de um
triângulo é π radianos. Façamos mais um caso, tomando n = 4. Neste caso,
podemos dividir um quadrilátero em dois triângulos, como mostra a Figura 5.1-
(a)
Assim, a soma dos ângulos internos de um quadrilátero é 2π radianos.
Para elucidar o processo de indução e não deixar dúvidas sobre o que iremos
fazer, vamos considerar mais um polı́gono, o pentágono (n = 5). Neste caso, para
mostrar que a soma dos seus ângulos internos é (5 − 2)π = 3π radianos, iremos
dividir o pentágono A1 A2 A3 A4 A5 em um quadrilátero A1 A2 A3 A4 e um triângulo
A1 A4 A5 , como mostra a Figura 5.1-(b). Assim, a soma dos ângulos internos do
pentágono A1 A2 A3 A4 A5 é igual à soma dos ângulos internos do triângulo A1 A4 A5
(igual a π) mais a soma dos ângulos internos do quadrilátero A1 A2 A3 A4 (igual a
2π), ou seja, é igual a 3π.
Finalmente, vamos assumir como hipótese de indução que para um certo n ≥ 3
mostramos que a soma dos ângulos internos do n-ágono é (n − 2)π. Precisamos
mostrar que a soma dos ângulos internos de um n + 1-ágono é [(n + 1) − 2]π =
(n − 1)π. De fato, podemos repetir o processo acima. Vamos denominar de
A1 , A2 , . . . , An , An+1 os vértices do (n + 1)-ágono. Podemos dividi-lo no n-ágono
A1 A2 . . . An e no triângulo A1 An+1 An . Logo, a soma dos ângulos internos do
(n + 1)-ágono é (n − 2)π + π = (n − 1)π

Exemplo 5.10. Mostre que o número de diagonais de um polı́gono convexo de


n-lados é igual a n(n−3)
2
.

Solução. Observe que para n = 3 temos que existem 0 = 3.(3 − 3)/2 diagonais
num triângulo. Para n = 4, temos 2 = 4(4 − 3)/2 diagonais num quadrilátero
convexo (veja a figura 5.2).

i i

i i
i i “Livro˙Olimpiada˙O
2006/5/27
page 121
i i

[SEC. 5.3: INDUÇÃO NA GEOMETRIA 121

Vamos agora assumir como hipótese de indução que se n é um n-ágono con-


vexo então o seu número de diagonais é n(n − 3)/2 e vamos provar que a fórmula
vale para um (n + 1)-ágono convexo. De fato, denote por A1 , A2 , . . . , An , An+1
os vértices do n + 1-ágono. Podemos decompô-lo como a união do n-ágono
A1 , A2 , . . . , An e do triângulo A1 , An , An+1 . Neste caso, para contarmos as di-
agonais do (n + 1)-ágono devemos considerar os seguintes casos:
• Diagonais do n-ágono A1 , A2 , . . . , An ; por hipótese de indução, o número
dessas diagonais é n(n − 3)/2.
• n − 2 diagonais que partem do vértice An+1 mais a diagonal A1 An .
Assim, o número total de diagonais do (n + 1)-ágono é

n(n − 3) n2 − 3n + 2n − 2 n2 − n − 2 (n + 1)(n − 2)
+ (n − 2) + 1 = = = ,
2 2 2 2
como querı́amos demonstrar.

A4 A4
A3

A5 A3

A1 A2 A1 A2
(a) (b)

Figura 5.2::

Exemplo 5.11. Mostre que podemos cobrir os n2 pontos no reticulado abaixo


traçando 2n − 2 segmentos de reta sem tirar o lápis do papel.

Solução. O caso n = 3 já foi enunciado no exemplo 1.9 do Capı́tulo 1. A figura


abaixo mostra a solução, onde o caminho realizado com as 4 linhas é o seguinte:
A → B → C → D → B.
Daremos a prova do problema acima por indução. Para isso, veja que podemos
resolver o caso n = 4 continuando a solução do caso n = 3. Como paramos num
dos vértices do quadrado 3 × 3, acrescentamos mais uma linha e uma coluna para
obter um reticulado 4 × 4. Assim, conseguimos cobrir os 16 pontos utilizando

i i

i i
i i “Livro˙Olimpiada˙O
2006/5/27
page 122
i i

122 [CAP. 5: INDUÇÃO MATEMÁTICA

• • • • •

• • • • •

• • • • •

• • • • •

• • • • •
| {z }
n×n−pontos

Figura 5.3::

A
• • •

• • •

• • •
D B

Figura 5.4::

4 + 2 = 6 linhas, sem tirar o lápis do papel e cobrindo dois lados do quadrado,


como mostram as linhas descontı́nuas na Figura 5.5.

Finalmente, vamos assumir como hipótese de indução que podemos cobrir


n ≥ 2 um reticulado n×n com 2n−2 linhas, sendo que a última delas cobre um dos
lados do reticulado. Acrescentando 2n+1 pontos como mostra a figura, obtemos
um reticulado (n + 1) × (n + 1) que pode ser coberto com 2n − 2 + 2 = 2(n + 1) − 2
pontos, como querı́amos demonstrar.

i i

i i
i i “Livro˙Olimpiada˙O
2006/5/27
page 123
i i

[SEC. 5.4: MISCELÂNEA 123

C
• • • •

A
• • • •

• • • •

• • • •
D B

Figura 5.5::

5.4 Miscelânea
Nesta seção discutiremos alguns exemplos interessantes de como podemos aplicar
o método da indução aos mais variados tipos de problemas. O primeiro deles é
uma generalização do problema 1.5:
Exemplo 5.12. Um rei muito rico possui 3n moedas de ouro. Porém, uma destas
moedas é falsa e seu peso é menor que o peso das demais. Com uma balança de
2 pratos e sem nenhum peso, mostre que é possı́vel encontrar a moeda falsa com
apenas n pesagens.
Solução. Para resolver este problema, vamos utilizar o método da indução. De
fato, se n = 1, procederemos da seguinte forma: pegamos duas moedas quaisquer e
colocamos na balança, deixando uma do lado de fora. Caso a balança se equilibre,
a moeda que está do lado de fora é necessariamente a que tem menor peso. Caso
a balança se desequilibre, a que tem menor peso está na balança, no prato mais
alto. O caso n = 2 foi feito no problema 1.5.
Vamos agora assumir como hipótese de indução que dadas 3 moedas, podemos
achar a moeda mais leve com n pesagens. Vamos mostrar que para n + 1 moedas,
precisamos de n + 1 pesagens. De fato, dividiremos as 3n+1 moedas em 3 grupos,
A, B e C com 3n moedas cada. Colocamos na balança os grupos A e B. Caso
os dois grupos se equilibrem, a moeda mais leve está no grupo C. Caso o grupo
A esteja mais leve, a moeda mais leve se encontra no grupo A. De qualquer
modo, com uma pesagem conseguimos determinar em qual grupo de 3n elementos
a moeda mais leve se encontra. Por hipótese de indução, precisamos de mais n
pesagens para encontrar a moeda mais leve, totalizando n + 1 pesagens.

Exemplo 5.13. Mostre que utilizando um balde com 5 litros de capacidade e


outro com 7 litros, é possı́vel separar qualquer quantidade superior a 24 litros.

i i

i i
i i “Livro˙Olimpiada˙O
2006/5/27
page 124
i i

124 [CAP. 5: INDUÇÃO MATEMÁTICA

Solução. Novamente, faremos a prova utilizando o método da indução. Neste caso,


começaremos o processo de indução a partir de 24. De fato, podemos separar 24
litros utilizando duas vezes o balde de 7 e duas vezes o balde de 5 litros. Note que
o problema acima equivale a mostrar que
Todo número maior ou igual a 24 pode ser escrito da forma 7x + 5y,
onde x e y são números inteiros maiores ou iguais a zero.
Neste caso, escrevemos 24 como 24 = 2.7 + 2.5. Por hipótese de indução,
vamos supor que conseguimos escrever um número n ≥ 24 como n = 7x + 5y, com
x e y números inteiros maiores ou iguais a zero. Devemos mostrar que n + 1 se
escreve deste modo também. Para isso, vamos dividir a análise em dois casos:
Caso 1: y ≤ 3
Logo, x ≥ 2 pois se isso não ocorresse, terı́amos 7x + 5y ≤ 22 < 24, o que é
impossı́vel. Assim, podemos escrever:

n + 1 = 7x + 5y + 1 = 7(x − 2) + 5(y + 3),

pois x − 2 ≥ 0.
Caso 2: y ≥ 4
Neste caso, y − 4 é maior que 0. Logo, podemos escrever:

n + 1 = 7x + 5y + 1 = 7(x + 3) + 5(y − 4),

finalizando a nossa prova por indução

Finalizamos esta seção com um exemplo clássico conhecido como Torre de


Hanói.

Exemplo 5.14. A Torre de Hanói é um jogo composto por uma base com 3
hastes e n anéis colocadas num deles. Os anéis são colocados em ordem crescente
de acordo com seu tamanho (ver Figura 5.6). Os anéis podem ser deslocados de
uma haste para qualquer outra, sendo que nunca pode ser colocado um anel maior
em cima de um menor. Prove que:
(a) É possı́vel mover todos os anéis para uma das hastes que estão livres;
(b) Isto pode ser realizado usando 2n − 1 movimentos.

Solução. Resolveremos ambas questões usando indução sobre n. Primeiro verifi-


camos que as afirmações são válidas para n = 1, 2. Com efeito, se temos somente
um anel basta mover este para qualquer outra haste com um único movimento
(21 − 1 = 1). Se temos 2 anéis então movemos o menor deles para a segunda
haste, o maior para a terceira haste e, finalmente, o menor para a terceira haste,
realizando um total de 3 movimentos (22 − 1 = 3).

i i

i i
i i “Livro˙Olimpiada˙O
2006/5/27
page 125
i i

[SEC. 5.5: MÉTODO FORTE DA INDUÇÃO 125

Figura 5.6: Torre de Hanoi

Uma vez constatada a veracidade das afirmações para n = 1, 2, supomos


que as mesmas valem para n = k. Agora resta provar que também valem para
n = k + 1. De fato, se temos k + 1 anéis na primeira haste, então pela hipótese
indutiva podemos mover todos eles, exceto o maior, para a terceira haste usando
2k − 1 movimentos. Seguidamente movemos o anel restante para a segundo haste
e, finalmente, movemos os k anéis da terceira haste para a segunda haste usando
novamente 2k − 1 movimentos. Para finalizar, notamos que foram feitos (2k − 1) +
1 + (2k − 1) = 2k+1 − 1 movimentos.

5.5 Método Forte da Indução


Muitas vezes, para conseguir mostrar que a hipótese P (n + 1) é verdadeira, pre-
cisamos supor que P (k) é verdadeira para todo n0 ≤ k ≤ n. Isto é a base do
Método Forte da Indução Finita que enunciamos a seguir:

Teorema 5.15 (Método Forte da Indução Finita). Considere n0 um in-


teiro não negativo. Suponhamos que, para cada inteiro n ≥ n0 seja dada uma
proposição P (n), a qual chamaremos de “hipótese indutiva”. Se podemos provar,
simultaneamente, as duas seguintes propriedades:
(a) a proposição P (n0 ) é verdadeira;
(b) para cada inteiro não negativo k, com n0 ≤ k ≤ n, temos que cada uma
das proposições P (k) são verdadeiras, segue-se que P (n + 1) é também ver-
dadeira,
então, a proposição P (n) é verdadeira para qualquer n ≥ n0 .

Vejamos agora algumas aplicações interessantes deste método.


Utilizando o Método Forte da Indução, vamos dar agora a prova alternativa
do Teorema Fundamental da Aritmética estudado no Capı́tulo 2.

i i

i i
i i “Livro˙Olimpiada˙O
2006/5/27
page 126
i i

126 [CAP. 5: INDUÇÃO MATEMÁTICA

Exemplo 5.16 (Teorema Fundamental da Aritmética). Todo número nat-


ural N maior que 1 pode ser escrito como um produto

N = p1 · p2 · p3 · · · pm , (5.3)

onde m ≥ 1 é um número natural e os pi , 1 ≤ i ≤ m são números primos. Além


disso, a fatoração em (5.3) é única se exigirmos que p1 ≤ p2 ≤ · · · ≤ pm .

Solução. Para cada n ∈ N, n ≥ 2, definamos a proposição

P (n) : n é um número primo ou um produto de números primos.

Notemos que P (2) é verdadeira, pois 2 é um número primo


Agora plantemos o passo indutivo:
• Hipótese: P (k) é verdade para cada inteiro k tal que 2 ≤ k ≤ n.
• Tese: P (n + 1) é verdade. Em outras palavras, temos que mostrar que n + 1
é um número primo ou ele é um produto de números primos.
Faremos a prova dividindo em dois casos:

(a) Se n + 1 é um numero primo, isto mostra que P (n + 1) é verdade, e isto


acaba nossa demonstração.
(b) Se n + 1 não é um número primo, então existem α, β ∈ N com 2 ≤ α ≤ n e
2 ≤ β ≤ n tais que n + 1 = α · β.
Nossa hipótese indutiva é válida para P (α) e P (β). Isto significa que α é
um número primo ou um produto de números primos e que β é um número
primo ou um produto de números primos. Portanto, n + 1 = α · β é um
produto de números primos.

Exemplo 5.17. Dada a seguinte relação de recorrência

a0 = 8;
a1 = 10;
an = 4an−1 − 3an−2 , ∀n ≥ 2.

Mostre que sua solução é dada por

an = 7 + 3 n , ∀n ∈ Z+ .

i i

i i
i i “Livro˙Olimpiada˙O
2006/5/27
page 127
i i

[SEC. 5.5: MÉTODO FORTE DA INDUÇÃO 127

Solução. Definamos a proposição P (n) : an = 7 + 3n . P (0) é verdadeira, pois


P (0) = 7 + 30 = 7 + 1 = 8. Suponhamos que P (k) é verdadeiro para cada inteiro
k tal que 1 ≤ k ≤ n. Vamos mostrar que P (k) é verdade para k = n + 1. Com
efeito,
an+1 = 4an − 3an−1
= 4(7 + 3n ) − 3(7 + 3n−1 )
= 7 + 4 × 3n − 3 × 3n−1
¡ ¢
= 7 + 3n−1 4 × 3 − 3
¡ ¢
= 7 + 3n−1 9 = 7 + 3n−1 × 32
= 7 + 3n+1 .

Definimos a seqüência de Fibonacci como sendo a seqüência Fn tal que

F1 = 1;
F2 = 1;
Fn = Fn−1 + Fn−2 , se n ≥ 3.
A seqüência de Fibonacci adquiriu muita fama devido a suas conexões com
áreas das mais variadas na cultura humana. Ela aparece em problemas de biologia,
arquitetura, engenharia, fı́sica, quı́mica e muitos outros. Agora vamos utilizar
indução para mostrar algumas de suas propriedades:
Exemplo 5.18. Considere Fn a seqüência de Fibonacci. Mostre que
µ ¶n
7
Fn < .
4
¡ ¢n
Solução. Definamos a proposição P (n) := Fn < 47 . Para n = 1 temos que F1 =
1 < 47 , de modo que P (1) é verdadeira. Suponhamos que P (1), P (2), . . . , P (n), ∀n ≥
¡ ¢n+1
2, sejam todas verdadeiras. Mostraremos que Fn+1 < 74 . Com efeito,
Fn+1 = Fn + Fn−1
¡ ¢n ¡ ¢n−1
< 74 + 47
¡ ¢n−1 ¡ 7 ¢n−1
< 74 47 + 4
¡ ¢ ¡ ¢n−1
< 1 + 47 74 .
¡ 7
¢ ¡ 7 ¢2 ¡ 7 ¢2 ¡ 7 ¢n−1
Como 1 + 4 < 4 , segue-se que Fn+1 < 4 4
. Portanto, Fn+1 <
¡ 7 ¢n+1
4
.

i i

i i
i i “Livro˙Olimpiada˙O
2006/5/27
page 128
i i

128 [CAP. 5: INDUÇÃO MATEMÁTICA

Observação 5.19. Quando aplicamos o Método da Indução devemos tomar cer-


tos cuidados. A seguir damos um exemplo de como o método pode ser aplicado
de forma errada. Vamos mostrar a seguinte afirmação:
Afirmação: Num conjunto qualquer de n bolas , todas as bolas pos
suem a mesma cor.
Observe que nossa proposição é claramente falsa. Mas, mesmo assim, vamos dar
uma prova por indução:
Para n = 1, nossa proposição é verdadeira pois em qualquer conjunto com
uma bola, todas as bolas têm a mesma cor, pois só existe uma bola. Assuma
por hipótese de indução que a proposição é verdadeira para n e provemos que a
proposição é verdadeira para n + 1. Ora, seja A = {b1 , . . . , bn , bn+1 } o conjunto
com n+1 bolas referido. Considere os subconjuntos de B e C de A com n elementos,
construı́dos como:

B = {b1 , b2 , . . . , bn } e C = {b2 , . . . , bn+1 }

Observe que ambos os conjuntos têm n elementos. Assim, as bolas b1 , b2 , . . . , bn


do conjunto B têm a mesma cor. Do mesmo modo, as bolas do conjunto C têm
a mesma cor. Em particular, a bola bn tem a mesma cor da bola bn+1 . Assim,
todas as bolas têm a mesma cor.
Ache o erro no argumento!

5.6 Exercı́cios
1. Considere a, b ∈ R de modo que a + b > 0 e a 6= b. Mostre que

2n−1 (an + bn ) > (a + b)n , ∀n ∈ N.

2. Mostre que para qualquer n ∈ N é valida a seguinte desigualdade


1 1 1 1 √
√ + √ + √ + · · · + √ > n.
1 2 3 n

3. Mostre que para qualquer n ∈ N é valida a seguinte desigualdade


1 1 1 1 13
+ + + ··· + > .
n+1 n+2 n+3 2n 24

4. Mostre a seguinte identidade trigonométrica


(n + 1) cos nx − n cos(n + 1)x − 1
cos x + 2 cos 2x + · · · + n cos nx = .
4 sin2 x2

i i

i i
i i “Livro˙Olimpiada˙O
2006/5/27
page 129
i i

[SEC. 5.6: EXERCÍCIOS 129

5. Um torneio de xadrez tem n jogadores. Cada jogador joga uma única par-
tida contra todos os outros jogadores. Calcule o número total de partidas
do torneio.
6. Demonstre que para qualquer n ∈ N, é valida a igualdade:
· ¸2
n(n + 1)
13 + 23 + 33 . . . + n3 =
2

7. Demonstre que para qualquer n ∈ N, é valida a igualdade:

n(n + 1)(2n + 1)(3n2 + 3n − 1)


14 + 24 + 34 . . . + n4 =
30

8. Mostre que qualquer número natural n ≥ 0, 11n+2 + 122n+1 é sempre di-


visı́vel por 133.
9. Mostre que para todo n ∈ Z+ temos que 32n+1 + 2n+2 é um múltiplo de 7.
10. Mostre que para todo n ∈ Z+ temos que 32n+2 + 26n+1 é um múltiplo de
11.
11. Lembrando que a seqüência de Fibonacci define-se recursivamente por

F1 = 1;
F2 = 1;
Fn = Fn−1 + Fn−2 ,

mostre as seguintes propriedades:


n
X n
X
(a) Fi = Fn+2 − 1; (b) F2i−1 = F2n ;
i=1 i=1
Xn
(c) F2i = F2n+1 − 1; (d) Fn−1 Fn+1 − Fn2 = (−1)n ;
i=1
2n−1
X 2
(e) Fn Fn+1 − Fn−2 Fn−1 = F2n−1 ; (f) Fi Fi+1 = F2n ;
i=1
2n
X n
X
2
(g) Fi Fi+1 = F2n+1 − 1; (h) Fi2 = Fn Fn+1 .
i=1 i=1

12. Considere Fn a seqüência de Fibonacci. Mostre que


µ √ ¶n µ √ ¶n
1 1+ 5 1 1− 5
Fn = √ −√ .
5 2 5 2

i i

i i
i i “Livro˙Olimpiada˙O
2006/5/27
page 130
i i

130 [CAP. 5: INDUÇÃO MATEMÁTICA

13. Calcular o número de regiões em que o plano é dividido por n retas distintas
em cada uma das seguintes situações
(a) As n retas são concorrentes;
(b) Não existem duas retas paralelas nem três retas concorrentes.

i i

i i
i i “Livro˙Olimpiada˙O
2006/5/27
page 131
i i

Capı́tulo 6

Problemas de Olimpı́adas

Este capı́tulo está inteiramente dedicado à problemas de olimpı́adas regionais,


nacionais e internacionais. Nele você vai encontrar um amplo material suficiente
para algumas horas (ou dias!) de diversão e treinamento para olimpı́adas. Para
entender as siglas de cada competição, veja a tabela no final do capı́tulo. Para
as soluções, exercı́cios adicionais e continuação do treinamento, recomendamos
a leitura dos livros [11], [12] e [13], além da visita aos sites www.obm.org.br e
www.obmep.org.br. Agora que você já estudou bastante, mão na massa e boa
sorte!

Problema 6.1 (OIM-1988). Seja ABC um triângulo cujos lados são a, b e


c. Divide-se cada lado do triângulo em n segmentos iguais. Sejam sA (respec-
tivamente, sB e sC ) a soma dos quadrados das distâncias do vértice A a cada
um dos pontos de divisão do lado oposto a A (respectivamente, B e C). Se
S = sA + sB + SC , demonstre que

S
a2 + b2 + c2

é um número racional.

Problema 6.2 (IMO-1979). Seja P um ponto dado dentro de uma esfera. Três
vetores mutuamente perpendiculares partem de P e intersectam a esfera nos pon-
tos U, V e W . Q denota o vértice diagonalmente oposto a P no paralelepı́pedo
determinado por P U, P V e P W . Encontre o lugar geométrico de Q, para as tri-
nas de raios que partem de P .

131

i i

i i
i i “Livro˙Olimpiada˙O
2006/5/27
page 132
i i

132 [CAP. 6: PROBLEMAS DE OLIMPÍADAS

Problema 6.3 (OAM-2005). Decida se o seguinte número é primo, justifi-


cando:
(2005)4 + 242 .

Problema 6.4 (OAM-2005). Será que existe um múltiplo de 2005 que se escreve
utilizando apenas zeros e uns?

Problema 6.5 (OBM-2002). Mostre que, entre dezoito inteiros consecutivos de


três algarismos, sempre existe algum que é divisı́vel pela soma de seus algarismos.

Problema 6.6 (IMO-1983). Diga se é possı́vel escolher 1983 números inteiros


positivos e distintos, todos menores ou iguais que 105 , onde nenhum de três in-
teiros consecutivos estão em progressão aritmética? Justifique sua resposta.

Problema 6.7 (IMO-1978). Considere a seqüência de inteiros positivos distin-


tos {ak }∞
k=1 . Mostre que

n
X n
X
ak 1
2
≥ , ∀n ∈ N.
k k
k=1 k=1

Problema 6.8 (IMO-1979). Sejam p e q números naturais tais que


p 1 1 1 1 1
= 1 − + − + ··· − + .
q 2 3 4 1318 1319
Mostre que p é divisı́vel por 1979.

Problema 6.9 (IMO-1979). Considere π como sendo um plano e tome P ∈ π e


QP + P R
Q∈/ π. Encontre todos os pontos R ∈ π tal que a razão seja máxima.
QR
Problema 6.10 (IMO-1978). Sejam m e n números naturais tais que 1 ≤
m < n. Se na representação decimal de 1978m temos que seus três últimos dı́gitos
coincidem com os três últimos dı́gitos da representação decimal de 1978n , encontre
m e n de modo que m + n seja o menor valor possı́vel.

Problema 6.11 (IMO-1979). Encontre todos os números reais a para os quais


existem números reais não negativos x1 , x2 , x3 , x4 , x5 satisfazendo as relações
5
X 5
X 5
X
kxk = a, k3 xk = a2 , k5 xk = a3 .
k=1 k=1 k=1

Problema 6.12 (IMO-1981). Determine o valor máximo de m2 + n2 , onde m


e n são inteiros satisfazendo m, n ∈ {1, 2, . . . , 1981} e (n2 − mn − m2 )2 = 1.

i i

i i
i i “Livro˙Olimpiada˙O
2006/5/27
page 133
i i

133

Problema 6.13 (IMO-1981). Resonda as seguintes questões:


(a) Para quais valores de n > 2 existe um conjunto de n inteiros positivos
consecutivos de modo que o maior elemento neste conjunto seja um divisor
do mı́nimo múltiplo comum dos restantes n − 1 números?
(b) Para quais valores de n > 2 existe exatamente um conjunto com a pro-
priedade enunciada em (a)?
Problema 6.14 (IMO-1981). A função f (x, y) satisfaz
f (0, y) =y + 1,
f (x + 1, 0) =f (x, 1),
f (x + 1, y + 1) =f (x, f (x + 1, y)),
para todos os inteiros não negativos x, y. Determine f (4, 1981).
Problema 6.15 (IMO-1982). A função f (n) esta definida para todos os inteiros
positivos n e toma seus valores nos inteiros não negativos. Também para quaisquer
m e n temos que
f (m + n) − f (m) − f (n) = 0 ou 1
f (2) = 0, f (3) > 0, e f (9999) = 3333.
Ache f (1982).
Problema 6.16 (IMO-1982). Prove que se n é um inteiro positivo tal que a
equação
x3 − 3xy 2 + y 3 = n
tem uma solução inteira (x, y), então existem pelo menos outras três soluções
inteiras. Demonstre que a equação não tem solução inteira quando n = 2891.
Problema 6.17 (IMO-1982). Seja ABCDEF um hexágono regular. Sejam M
um ponto sobre a diagonal AC e N um ponto sobre a diagonal CE, de modo que
AM CN
= = r.
AC CE
Determine r no caso em que B, M e N sejam colineares.
Problema 6.18 (IMO-1985). Dado um conjunto M de 1985 inteiros positivos
distintos onde nenhum deles tem um divisor primo maior que 26, prove que M
contém pelo menos um subconjunto de quatro elementos distintos cujo produto é
a quarta potência de um inteiro.
Problema 6.19 (IMO-1977). Seja f (n) uma função definida no conjunto dos
inteiros positivos e tomando valores no mesmo conjunto. Prove que se f (n + 1) >
f (f (n)), para cada inteiro positivo n, então f (n) = n , ∀n.

i i

i i
i i “Livro˙Olimpiada˙O
2006/5/27
page 134
i i

134 [CAP. 6: PROBLEMAS DE OLIMPÍADAS

Problema 6.20 (IMO-1975). Quando 44444444 é escrito em notação decimal,


temos que a soma de seus dı́gitos é A. Agora, denote por B a soma dos dı́gitos de
A. Encontre a soma dos dı́gitos de B. (A e B foram escritos em notação decimal)

Problema 6.21 (IMO-1963). Encontre todas as raı́zes da equação


p p
x2 − p + 2 x2 − 1 = x,

onde p é um parâmetro real.

Problema 6.22 (OCM-2003). Um homem acha-se no centro de um cı́rculo. A


periferia desse cı́rculo é delimitada por uma cerca, que separa um homem de um
cachorro. Admitindo que o cachorro só pode correr ao longo da cerca, prove que o
homem pode escapar pulando a cerca sem ser mordido pelo cão se as velocidades
máximas possı́veis de serem desenvolvidas pelo cachorro e pelo homem estiverem
entre si na razão 4:1. Determine as relações entre as velocidades máximas do
cachorro e do homem para os quais o homem pode escapar.

Problema 6.23 (IMO-1997). Sejam x1 , x2 , ..., xn números reais que verificam


as condições:
n+1
|x1 + x2 + ... + xn | = 1 e |xi | ≤ , para i = 1, 2, . . . , n.
2
Demonstrar que existe uma reordenação (ou permutação) y1 , y2 , ..., yn de x1 , x2 , ..., xn
tal que
n+1
|y1 + 2y2 + · · · + nyn | ≤ .
2
Problema 6.24 (IMO-1997). Determinar todos os pares (a, b) de inteiros a ≥
1, b ≥ 1 que satisfazem a equação
2
ab = b a .

Problema 6.25 (IMO-2002). Seja n um inteiro positivo e T o conjunto de


pontos (x, y) no plano onde x e y são inteiros não negativos com x + y < n.
Cada ponto de T é pintado de vermelho ou azul. Se um ponto (x, y) é vermelho,
então todos os pontos (x′ , y ′ ) com x′ ≤ x e y ′ ≤ y também são. Um conjunto
X é um conjunto de n pontos azuis com abscissas distintas, e um conjunto Y é
um conjunto de n pontos azuis com ordenadas distintas. Prove que o número de
conjuntos X é igual ao número de conjuntos Y.

Problema 6.26 (IMO-2000). No inı́cio existem n (n ≥ 2) pulgas numa reta


horizontal, nem todas no mesmo ponto. Para um número real positivo λ define-se
um salto da seguinte maneira: Escolhem-se duas pulgas quaisquer nos pontos A e

i i

i i
i i “Livro˙Olimpiada˙O
2006/5/27
page 135
i i

135

B com o ponto A à esquerda do ponto B; A pulga que está em A salta até o ponto
C da reta, à direita de B, tal que
BC
= λ.
AB
Determine todos os valores de λ para os quais qualquer ponto M na reta e quais-
quer que sejam as posições iniciais das n pulgas, existe uma sucessão finita de
saltos que levam todas as pulgas para pontos à direita de M .

Problema 6.27 (IMO-2000). Um mágico tem cem cartões numerados de 1 a


100. Coloca-os em três caixas, uma vermelha, uma branca e uma azul, de modo
que cada caixa contém pelo menos um cartão. Uma pessoa da platéia escolhe duas
das três caixas, seleciona um cartão de cada caixa e anuncia a soma dos números
dos dois cartões que escolheu. Ao saber esta soma, o mágico identifica a caixa
da qual não se retirou nenhum cartão. De quantas maneiras podem ser colocados
todos os cartões nas caixas de modo de que este truque sempre funcione? (Duas
maneiras consideram-se diferentes se pelo menos um cartão é colocado numa caixa
diferente).

Problema 6.28 (IMO-2001). Sejam a, b, c, d inteiros com a > b > c > d > 0.
Considere que
ac + bd = (b + d + a − c)(b + d − a + c).
Prove que ab + cd não é um número primo.

Problema 6.29 (OCS-1997). Seja n um número natural, n > 3. Demonstre


que entre os múltiplos de 9 menores que 10n há mais números com a soma de seus
dı́gitos igual a 9(n − 2) que números com a soma de seus dı́gitos igual a 9(n − 1).

Problema 6.30 (OCS-1997). Considere um triângulo acutângulo ABC, e seja


X um ponto do plano do triângulo. Sejam M, N e P as projeções ortogonais de
X sobre as retas que contém as alturas do triângulo ABC. Determinar para que
posições de X o triângulo MNP é congruente a ABC. Nota: a projeção ortogonal
de um ponto X sobre uma reta l é a intersecção de l com a perpendicular a ela
que passa por X.

Problema 6.31 (OPM-2001). Um pacote com 100 bombons deve ser dividido
entre 15 garotos.
(a) Prove que existem dois garotos que receberam a mesma quantia de bombons.
(b) Qual o número mı́nimo de bombons no pacote, de modo que, depois de di-
vididos entre os 15 garotos, é possı́vel que não existam dois garotos que
receberam a mesma quantia de bombons.
Obs: Considere que algum garoto pode receber zero bombons.

i i

i i
i i “Livro˙Olimpiada˙O
2006/5/27
page 136
i i

136 [CAP. 6: PROBLEMAS DE OLIMPÍADAS

Problema 6.32 (OEMRJ-1998). Encontre todas as soluções inteiras e positi-


vas de
1 1 1
+ = ,
x y p
onde p é um número primo (cada solução é um par ordenado (x, y)). Encontre
pelo menos 5 soluções inteiras e positivas de
1 1 1
+ = ,
x y 1998

Problema 6.33 (OMRN-2000). É possı́vel colocar os inteiros 1, 2, 3, 4, ...,


239, 240 numa tabela com 15 linhas e 16 colunas, de modo que a soma dos números
em cada uma das colunas seja a mesma?

Problema 6.34 (ORMRS-1998). Mostrar que para cada n inteiro positivo:


µ ¶n
1
1+ < 3.
n
Problema 6.35 (OSM-1999). Os segmentos de reta AO, OB e OC são mutu-
amente perpendiculares. Expresse a área do triângulo ABC em termos das áreas
dos triângulos OBC, OCA e OAB.

Problema 6.36 (OCS-2004). Dada uma circunferência C e um ponto P exte-


rior a ela, traçam-se por P as duas tangentes à circunferência, sendo A e B os
pontos de tangência. Toma-se um ponto Q sobre o menor arco AB de C. Sejam
M a intersecção da reta AQ com a perpendicular a AQ traçada por P e N a in-
tersecção da reta BQ com a perpendicular a BQ traçada por P. Demonstre que,
ao variar Q no arco AB, todas as retas MN passam por um mesmo ponto.

Problema 6.37 (OM-2004). Ache todos os números naturais x, y, z que veri-


ficam simultaneamente:

xyz = 4104 e x + y + z = 77.

Problema 6.38 (OMEG-1999). Quantos retângulos diferentes constituı́dos de


um número inteiro de quadrados podem ser desenhados:
(a) Em um tabuleiro (xadrez) 8 × 8?
(b) Em um tabuleiro n × n?
Observação: consideramos dois retângulos diferentes se eles possuı́rem di-
mensões diferentes ou se ocuparem posições diferentes no tabuleiro.

Problema 6.39 (ORMSC-1999). Mostre que um número natural diferente de


1 formado somente por algarismos 1, não pode ser um quadrado perfeito.

i i

i i
i i “Livro˙Olimpiada˙O
2006/5/27
page 137
i i

137

Problema 6.40. Uma codorna é uma peça que pode se mover num quadriculado
infinito no seguinte sentido: em qualquer quadrado seu primeiro movimento é
para um quadrado vizinho (horizontal ou vertical) e depois se move n quadrados
em direção perpendicular ao primeiro movimento. Por exemplo, se n = 2, a
codorna é um cavalo do xadrez. Encontre todos os n tais que a codorna consegue
alcançar qualquer quadrado a partir de um quadrado qualquer.

Problema 6.41 (OBM-1982). Dê as soluções inteiras da equação x2 +15a = 2b .

Problema 6.42 (OBM-1981). Encontre o número de soluções inteiras, não


negativas, da equação x + y + z = n, onde n é um inteiro positivo.

Problema 6.43 (OBM-1982). Sejam x, y, z números inteiros tais que x3 +y 3 −


z 3 é múltiplo de 7. Mostre que um desses números é múltiplo de 7.

Problema 6.44 (OBM-1984). Quantos quadrados perfeitos existem entre 40.000


e 640.000 que são múltiplos simultaneamente de 3, 4 e 5?

Problema 6.45 (OBM-1986). Dado um número natural n, determine de quan-


tas maneiras esse número pode ser representado como soma de naturais consecu-
tivos.

Problema 6.46 (OBM-1984). Seja n um número inteiro maior que 1. Mostre


que 4n + n4 não é primo.

Problema 6.47 (OBM-1983). Mostre que, para todo número natural n ≥ 2,

1 1 1
1+ + +···+
2 3 n
não é inteiro.

Problema 6.48 (OBM-1981). Dado um número inteiro n, mostre que existe


um múltiplo de n que se escreve apenas com os algarismos 0 e 1. (Por exemplo,
se n = 3, temos 111 ou 1011, etc).

Problema 6.49 (IMO-1985). Seja D um inteiro diferente de 2, 5, 13. Mostre


que podemos escolher A e B pertencentes a {2, 5, 13, D} tal que AB − 1 não seja
quadrado perfeito.

i i

i i
i i “Livro˙Olimpiada˙O
2006/5/27
page 138
i i

138 [CAP. 6: PROBLEMAS DE OLIMPÍADAS

Problema 6.50 (OBM-1982). Um retângulo de lados inteiros m e n é dividido


em quadrados de lado 1. Um raio de luz entra no retângulo por um dos vértices,
na direção da bissetriz do ângulo reto, e é refletido sucessivamente nos lados do
retângulo. Quantos quadrados são atravessados pelo raio de luz?
Problema 6.51 (OBM-1981). Um clube de Matemática contém 100 membros.
Suponha que em qualquer grupo de 4 membros, existe um membro que conheça
os outros 3. Prove que existe um membro do clube que conhece todos os outros
99 membros. Qual é o número mı́nimo de tais membros? (Obs.: se o membro A
conhece o membro B, então o membro B também conhece o membro A).

Problema 6.52 (OBM-1985). Apenas 5 casais participam de uma reunião.


Após os cumprimentos, João pergunta a cada um dos outros 9 participantes:
“Quantos apertos de mão você deu?”e obtém todas as nove respostas possı́veis:
0, 1, 2, 3, 4, 5, 6, 7, 8. Qual foi a resposta da esposa do João? (Obs.: obviamente
ninguém apertou a mão do próprio conjugue.)

Problema 6.53 (OBM-1983). Quantos números, de 1 a 1983, podem ser es-


critos como soma de duas ou mais potências distintas de 3?

Problema 6.54 (OBM-1983). Os números a, b e c são reais não-negativos e p


e q são½inteiros positivos distintos. Prove que:
ap + bp = cp
Se , então a = 0 ou b = 0.
aq + bq = cq

Problema 6.55 (OBM-1985). Prove que se a, b e c são números reais tais que
a, b, c são positivos e a + b + c = 1, então a2 + b2 + c2 ≥ 1/3.

Problema 6.56. O ponto A é chamado “pseudocentro de simetria”de um con-


junto M (que contém mais de um ponto do plano) se é possı́vel remover um ponto
de M de modo que A seja o centro de simetria do conjunto resultante. Quantos
pseudocentros de simetria um conjunto finito pode ter?
Problema 6.57 (OBM-1983). Mostre que é possı́vel, usando apenas duas cores,
pintar os pontos de uma circunferência de tal forma que não exista triângulo
retângulo inscrito na circunferência com vértices em pontos da mesma cor.

Problema 6.58 (OBM-1984). Colam-se 27 cubos de madeira, todos iguais,


para montar um cubo maior. Um cupim parte do centro de um cubo pequeno,

i i

i i
i i “Livro˙Olimpiada˙O
2006/5/27
page 139
i i

139

colocado no centro de uma das faces do cubo grande. O cupim liga, sempre por
linhas retas, o centro de um dos cubos pequenos ao centro de outro cubo pequeno
com face adjacente ao primeiro. Desta forma, existe um caminho que passe ex-
atamente uma vez por cada um dos pequenos cubos e termine no centro do cubo
grande?

Problema 6.59 (OBM-1985). Dois triângulos semelhantes têm dois pares de


lados ordenadamente iguais. Qual a condição para que os dois triângulos não se-
jam congruentes?

Problema 6.60 (OBM-1979). Sejam ABCD um quadrado, M o ponto médio


de AB, N o ponto médio de BC e I a intersecção de DN e CM . Calcule a área
do triângulo N IC, tomando AB = 1.

Problema 6.61 (OAM-2004). Um número n é dito dobrado se ao escrevermos


seus dı́gitos na ordem inversa, obtemos exatamente o dobro de n. Por exemplo,
2004 não é dobrado, pois 4002 6= 2 × 2004. Mostre que não existem números
dobrados menores que 1000.

Problema 6.62. Três amigos, Kleber, Ferdinando e Adão, que nunca mentem,
se reuniram e travou-se a seguinte conversa:
- Kleber fala: estou escolhendo dois números inteiros positivos maiores que
um, cujos valores não revelo e vou dar, em segredo, a soma deles para Ferdinando
e o produto deles para o Adão.
- Ferdinando fala: o valor que me foi dado não excede 16. Adão fala: eu não
consigo achar os valores dos dois números escolhidos pelo Kleber.
- Ferdinando fala: eu já sabia que você não encontraria os valores dos dois
números.
- Adão fala: ah, então eu sei quais são os dois números.
Responda: Qual o valor que foi dado a Ferdinando ? Baseado em que Adão
fez a última afirmativa ?

Problema 6.63 (OBM-2001). Mostre que não existem dois números inteiros
a e b tais que (a + b)(a2 + b2 ) = 2001.

Problema 6.64. Considere um quadrado de lado x. Com centro em cada vértice


traçam-se quatro circunferências de raio x. Determinar a área do quadrilátero
curvilı́neo interior ao quadrado dado.

i i

i i
i i “Livro˙Olimpiada˙O
2006/5/27
page 140
i i

140 [CAP. 6: PROBLEMAS DE OLIMPÍADAS

Problema 6.65 (OBM-1981). Dados n pontos no plano, prove que existem 3


π
deles que determinam um ângulo menor ou igual a n .

Problema 6.66 (OBM-1985). Qual o algarismo das unidades do número N =


1 · 3 · 5 · 7 · · · · 1993?

Problema 6.67. Seja p o maior fator primo do número 314 + 313 − 12. Qual o
valor de p?

Problema 6.68. Prove que entre 13 números reais quaisquer y1 , . . . , y13 há dois
deles tais que: s √
yi − yj 2− 3
0≤ ≤ √ .
1 + yi yj 2+ 3

Problema 6.69 (OIM-1991). Encontre um número m de cinco algarismos dis-


tintos e não nulos que seja igual à soma de todos os números de três algarismos
distintos que se pode formar a partir dos algarismos de m.

Problema 6.70. Dado um conjunto M de 555 inteiros positivos, nenhum dos


quais tem divisor primo maior que 26, prove que M contém um subconjunto de
dois elementos distintos cujo produto é um quadrado perfeito.

Problema 6.71 (L.Putnan-1956). Demonstrar que todo inteiro tem um múltiplo


cuja representação decimal contém os 10 dı́gitos 1, 2, 3, 4, 5, 6, 7, 8, 9 e 0.

Problema 6.72 (OBM-2001). Dizemos que um número natural é legal quando


for soma de dois naturais consecutivos e também for soma de três naturais con-
secutivos.
(a) Mostre que 2001 é legal, mas 1999 e 2002 não são legais.
(b) Mostre que 20012001 é legal.

Problema 6.73 (IMO-1983). Diga se é possı́vel escolher 1983 números inteiros


positivos e distintos, todos menores ou iguais que 105 , onde nenhum de três in-
teiros consecutivos estão em progressão aritmética.

Problema 6.74. Considere a, b e c os comprimentos dos lados de um triângulo.


Mostre que
a2 b(a − b) + b2 c(b − c) + c2 a(c − a) ≥ 0.
Diga quando a igualdade ocorre.

i i

i i
i i “Livro˙Olimpiada˙O
2006/5/27
page 141
i i

141

Problema 6.75 (IMO-1984). Sejam a, b, c e d números inteiros ı́mpares tais


que 0 < a < b < c < d e ad = bc. Suponha que existam inteiros k e m tais que
a + d = 2k e b + c = 2m . Mostre que a = 1.

Problema 6.76 (IMO-1984). Sejam x, y e z números reais não negativos tais


que x + y + z = 1. Mostre que
7
0 ≤ yz + zx + xy − 2xyz ≤ .
27
√ √ √
Problema 6.77. Resolva a equação 3 x + 3 x − 16 = 3 x − 8.

Problema 6.78. Resolva o sistema de equações


(q x √
y
+ xy = √80 xy
,
x + y = 20.

Problema 6.79. Encontre todos os polinômios P (x) de modo que

P (x2 − 2x) = (P (x − 2))2 .

Problema 6.80. Determine os valores de p ∈ R tal que

x2 + 2px − 2
−2 < < 2.
x2 − 2x + 2
Problema 6.81. Mostre que para cada a > 0 temos:

a4 + 9 4
> .
10a 5
Problema 6.82 (OCOM-2004). Numa lista de 10 números inteiros positivos é
dada. É permitido escolher três termos consecutivos da lista e somar uma unidade
a cada um deles. Determine se sempre possı́vel, após uma quantidade finita de
operações deste tipo, que todos os termos da lista sejam múltiplos de quatro.

Problema 6.83 (IMO-1964). Prove que não existe n tal que 2n + 1 é divisı́vel
por 7 e ache todos os inteiros positivos n tais que 2n − 1 é divisı́vel por 7.

Problema 6.84 (OIM-1987). Encontre os f (x) tais que [f (x)]2 .f (1−x/1+x) =


64x para todo x distinto de 0, 1 e −1.

Problema 6.85. Prove que se m, n e r são inteiros positivos, não nulos, e 1 +


√ √ 2r−1
m + n 3 = (2 + 3) então é um quadrado perfeito.

i i

i i
i i “Livro˙Olimpiada˙O
2006/5/27
page 142
i i

142 [CAP. 6: PROBLEMAS DE OLIMPÍADAS

Problema 6.86 (OIM-1987). Se define a sucessão pn da seguinte maneira:


p1 = 2 e para todo maior ou igual a 2, pn e o maior divisor primo da expressão
p1 p2 ...pn + 1. Prove que pn é diferente de 5.

Problema 6.87 (OIM-1988). Considere os conjuntos de n números naturais


diferentes de zero nos quais não há três elementos em progressão aritmética.
Demonstre que em um desses conjuntos a soma dos inversos desses elementos
é máxima.

Problema 6.88 (OIM-1990). Seja f (x) = (x + b)2 − c um polinômio com b e


c números inteiros.

(a) Se p é um número primo tal que p divide a c e p2 não divide a c, demonstrar


que qualquer que seja o número inteiro n, p2 não divide a f (n).
(b) Seja um número primo, distinto de 2 que não divide a c.Se q divide a f (n)
para algum número inteiro n, demonstrar que para cada inteiro positivo r,
existe um número inteiro n′ tal que q divide a f (n′ ).

Problema 6.89 (OIM-1991). Encontrar um número inteiro N de cinco dı́gitos


diferentes e não nulas, que seja igual a soma de todos os números de três dı́gitos
distintos que se podem formar com os cinco dı́gitos de N .

Problema 6.90 (OIM-1992). Para cada inteiro positivo n, seja an o último


dı́gito do número1 + 2 + 3 + ... + n. Calcular a1 + a2 + ... + a1992 .

Problema 6.91 (OIM-1993). um número natural é capicua se ao escrevê-lo


em notação decimal, podemos lê-lo da mesma forma da esquerda para a direita
como de direita para a esquerda, por exemplo 8, 23432, 6446. Sejam x1 < x2 <
... < xi < xi+1 < ... todos os números capicuas. Para cada i seja yi = xi+1 − xi .
Quantos números primos distintos têm o conjunto {y1 , y2 , ...} ?

Problema 6.92 (OIM-1995). Determine os possı́veis valores das somas dos


dı́gitos de todos os quadrados perfeitos.

Problema 6.93 (OBMEP-2005). Capitú cortou uma folha de papel retangular


em 9 pedaços quadrados de lados 1, 4, 7, 8, 9, 10, 14, 15 e 18 centı́metros cada
um.
(a) Qual é a área da folha antes de ser cortada?
(b) Quais eram as dimensões da folha antes de ser cortada?
(c) Capitú precisa montar a folha de novo. Ajude-a, mostrando com um de-
senho, como fazer esta montagem.

i i

i i
i i “Livro˙Olimpiada˙O
2006/5/27
page 143
i i

143

Problema 6.94. Fatore o número 51985 − 1 como o produto de três inteiros, cada
um dos quais é maior que 5100 .

Problema 6.95. m caixas são dadas, cada uma contendo um certo número de
bolas. Se n < m uma operação permitida é escolher n caixas e acrescentar uma
bola em cada uma delas. Prove que
(a) Se m e n são primos entre si, então é possı́vel após um número finito de
operações deste tipo chegar a situação onde cada caixa contém a mesma
quantidade de bolas;
(b) Se m e n não são primos entre si, existem configurações iniciais das bolas
de modo que não é possı́vel chegar a situação onde todas as caixas contém
a mesma quantidade de bolas.

Problema 6.96 (IMO-1981). Determine o valor máximo de m2 + n2 , onde m


onde n são inteiros satisfazendo n, m ∈ {1, 2, . . . , 1981} e (n2 − mn − m2 )2 = 1.

Problema 6.97 (IMO-1983). É possı́vel escolher 1983 inteiros positivos distin-


tos, todos menores ou iguais a 105 , sem quaisquer três deles não sejam termos
consecutivos de uma progressão aritmética? Justifique sua resposta.

Problema 6.98 (IMO-1984). Ache um par de inteiros a, b tais que:


(a) ab(a + b) é divisı́vel por 7;
(b) (a + b)7 − a7 − b7 é divisı́vel por 77 .
Justifique sua resposta.

i i

i i
i i “Livro˙Olimpiada˙O
2006/5/27
page 144
i i

144 [CAP. 6: PROBLEMAS DE OLIMPÍADAS

Siglas
Competição Sigla
Olimpı́ada Internacional de Matemática IMO
Olimpı́ada Ibero-Americana de Matemática OIM
Olimpı́ada do Cone Sul OCS
Olimpı́ada de Maio OM
Olimpı́ada Brasileira de Matemática OBM
Olimpı́ada Brasileira de Matemática das Escolas Públicas OBMEP
Olimpı́ada Colombiana de Matemática OCOM
Olimpı́ada Alagoana de Matemática OAM
Olimpı́ada Cearense de Matemática OCM
Olimpı́ada de Matemática do Estado de Goiás OMEG
Olimpı́ada de Matemática do Estado do Rio de Janeiro OMERJ
Olimpı́ada Paraense de Matemática OPM
Olimpı́ada de Matemática do Rio Grande do Norte OCM
Olimpı́ada Regional de Matemática do Rio Grande do Sul ORMRS
Olimpı́ada Regional de Matemática de Santa Catarina ORMSC
Olimpı́adas Sergipanas de Matemática OSM

i i

i i
i i “Livro˙Olimpiada˙O
2006/5/27
page 145
i i

Referências Bibliográficas

[1] FOMIN, Dimitri & GENKIN, Sergei & ITENBERG, Ilia. Mathematical
Circles(Russian Experience). American Mathematical Society. Mathemat-
ical World, Volume 7, 1996. 272 p.
[2] LIMA, Elon L. & PINTO, Paulo C. & WAGNER, Eduardo & MORGADO,
Augusto C. Temas e Problemas. Sociedade Brasileira de Matemática.
Coleção do Professor de Matemática, 2001. 193 p.
[3] RIBENBOIM, Paulo. Números Primos: Mistérios e Recordes. Coleção
Matemática Universitária, IMPA, 2001. 280 p.
[4] OLIVEIRA SANTOS, José Plı́nio de. Introdução à Teoria dos Números.
Coleção Matemática Universitária, IMPA, 1993. 199 p.
[5] VINOGRADOV, I. Fundamentos de la Teoria de los Números. Editorial
MIR, Moscu, 1987. 207 p.
[6] OBM: Olimpı́ada Brasileira de Matemática. www.obm.org.br
[7] OBMEP: Olimpı́ada Brasileira de Matemática das Escolas Públicas.
www.obmep.org.br
[8] NIVEN, I., ZUCKERMAN, S., MONTGOMERY, L. An Introduction To
The Theory Of Numbers. John Wiley & Sons, Unided States of America,
1991, 529 p.
[9] ANDREWS, G. Number Theory. Dover Publications, Estados Unidos da
América, 1994, 259 p.
[10] MORGADO, A., CARVALHO, J., CARVALHO, P., FERNANDEZ, P.
Análise Combinatória e Probabilidade. SBM, Brasil, 1991, 343 p.
[11] WAGNER, E., MOREIRA, C. 10 Olimpı́adas Iberoamericanas de
Matemática. OEI, Espanha, 1996, 277 p.
[12] MEGA, E., WATANABE, R. Olimpı́adas Brasileiras de Matemática,
1a a 8a . Editora Núcleo, Brasil, 1988, 403 p.

145

i i

i i
i i “Livro˙Olimpiada˙O
2006/5/27
page 146
i i

146 REFERÊNCIAS BIBLIOGRÁFICAS

[13] GREITZER, S. International Mathematical Olympiads. The Mathe-


matical Association of America, Estados Unidos da América, 1978, 201 p.

i i

i i

Das könnte Ihnen auch gefallen